Final Exam

¡Supera tus tareas y exámenes ahora con Quizwiz!

You sample a population of butterflies and find that 56% are heterozygous at a particular locus. What should be the frequency of the homozygous individuals in this population? A) 0.50 B) 0.09 C) 0.70 D) 0.08

A) 0.50

Glucose-6-phosphate dehydrogenase deficiency (G6PD) is inherited as an X-linked recessive allele in humans. A woman whose father suffered from G6PD marries a man who has the disease. What proportion of their sons would have the disease? A) 1/2 B) 1/4 C) 100% D) zero

A) 1/2

If glucose is the sole energy source, what fraction of the carbon dioxide exhaled by animals is generated only by the reactions involved in oxidation of pyruvate to acetyl CoA? A) 1/3 B) all of it C) 1/6 D) 2/3

A) 1/3

What is the probability of producing the genotype AABBCC in a cross of individuals who both possess this genotype: AaBbCc? A) 1/64 B) 1/8 C) 1/4 D) 1/16

A) 1/64

Assuming independent assortment for all gene pairs, what is the probability that a cross between the following parents, AABbCc × AaBbCc, will produce an AaBbCcoffspring? A) 1/8 B) 1/2 C) 3/4 D) 1/16

A) 1/8

In a diploid cell with 5 chromosome pairs (2n = 10), how many centromeres will be found in a nucleus at G2 of the cell division cycle? A) 10 B) 5 C) 40 D) 20

A) 10

A solution with a pH of 2 has how many more protons in it than a solution with a pH of 4? A) 100 times more B) 1000 times more C) 5 times more D) 10 times more

A) 100 times more

A homozygous tomato plant with red fruit and yellow flowers was crossed with a homozygous tomato plant with golden fruit and white flowers. The F1 all had red fruit and yellow flowers. The F1 were testcrossed by crossing them to homozygous recessive individuals, and the following offspring were obtained: Red fruit and yellow flowers—41 Red fruit and white flowers—7 Golden fruit and yellow flowers—8 Golden fruit and white flowers—44 How many map units separate these genes? A) 15 B) 17.6 C) 18.1 D) 17.1

A) 15

A group of cells is assayed for DNA content immediately following mitosis and is found to have an average of 8 picograms of DNA per nucleus. How many picograms of DNA would be found in a nucleus at prophase of mitosis? A) 16 B) 4 C) 8 D) 24

A) 16

Use the figure and the following description to answer the question. In a particular plant, leaf color is controlled by gene locusD. Plants with at least one allele D have dark green leaves, and plants with the homozygous recessive ddgenotype have light green leaves. A true-breeding, dark-leaved plant is crossed with a light-leaved one, and the F1offspring is allowed to self-pollinate. The predicted outcome of the F2 is diagrammed in the Punnett square shown in the figure, where 1, 2, 3, and 4 represent the genotypes corresponding to each box within the square. Which of the boxes marked 1-4 correspond to plants with a heterozygous genotype? A) 2 and 3 B) 1, 2, and 3 C) 2, 3, and 4 D) 1

A) 2 and 3

If there are 40 centromeres in a cell at anaphase of mitosis, how many chromosomes will be found in each daughter cell following cytokinesis? A) 20 B) 40 C) 10 D) 80

A) 20

In birds, sex is determined by a ZW chromosome scheme. Males are ZZ and females are ZW. A recessive lethal allele that causes death of the embryo is sometimes present on the Z chromosome in pigeons. What would be the sex ratio in the offspring of a cross between a male that is heterozygous for the lethal allele and a normal female? A) 2:1 male to female B) 3:1 male to female C) 1:1 male to female D) 1:2 male to female

A) 2:1 male to female

The citric acid cycle. Starting with citrate, which of the following combinations of products would result from three acetyl CoA molecules entering the citric acid cycle? A) 3 ATP, 6 CO2, 9 NADH, and 3 FADH2 B) 1 ATP, 2 CO2, 3 NADH, and 1 FADH2 C) 6 ATP, 6 CO2, 3 NADH, and 12 FADH2 D) 3 ATP, 3 CO2, 3 NADH, and 3 FADH2

A) 3 ATP, 6 CO2, 9 NADH, and 3 FADH2

Two true-breeding stocks of pea plants are crossed. One parent has red, axial flowers, and the other has white, terminal flowers; all F1 individuals have red, axial flowers. The genes for flower color and location assort independently. Among the F2 offspring, what is the probability of producing plants with white axial flowers? A) 3/16 B) 1/16 C) 9/16 D) 1/4

A) 3/16

Given the following genotypes for two parents, AABBCc × AabbCc, assume that all traits exhibit simple dominance and independent assortment. What proportion of the progeny of this cross will be expected to phenotypically resemble the first parent with the genotype AABBCc? A) 3/4 B) 3/8 C) 1 D) ¼

A) 3/4

Phenylketonuria is an inherited disease caused by a recessive autosomal allele. If a woman and her husband are both carriers, what is the probability that their first child will be a phenotypically normal girl? A) 3/8 B) 1/4 C) 3/16 D) 1/16

A) 3/8

The following question refers to this table of codons. Which of the following sequences of nucleotides are possible in the template strand of DNA that would code for the polypeptide sequence Phe-Leu-Ile-Val? A) 3′-AAA-GAA-TAA-CAA-5′ B) 5′-AUG-CTG-CAG-TAT-3′ C) 5′-TTG-CTA-CAG-TAG-3′ D) 3′-AAA-AAT-ATA-ACA-5′

A) 3′-AAA-GAA-TAA-CAA-5′

What wavelength of light in the figure is most effective in driving photosynthesis? A) 420 nm B) 625 nm C) 575 nm D) 730 nm

A) 420 nm

A triploid cell contains three sets of homologous chromosomes. If a cell of a diploid species that normally has 42 chromosomes per cell is triploid, this cell would be expected to have which of the following sets of chromosomes? A) 63 chromosomes in 21 sets of 3 B) 63 chromosomes, each with 3 sister chromatids C) 21 chromosome pairs and 21 unique chromosomes D) 63 chromosomes in 31 1/2 pairs

A) 63 chromosomes in 21 sets of 3

How many unique gametes could be produced through independent assortment by an individual with the genotype AaBbCCDdEE? A) 8 B) 16 C) 4 D) 64

A) 8

Cytosine makes up 42% of the nucleotides in a sample of DNA from an organism. Approximately what percentage of the nucleotides in this sample will be thymine? A) 8% B) 58% C) 16% D) 42%

A) 8%

Which molecule is a saturated fatty acid? A) 9 B) 5 C) 1 D) 8

A) 9

Five dialysis bags, constructed of a type of membrane that is permeable to water and impermeable to sucrose, were filled with various concentrations of sucrose and then placed in separate beakers containing an initial concentration of 0.6 M sucrose solution. At 10-minute intervals, the bags were massed (weighed), and the percent change in mass of each bag was graphed. Which line in the graph represents the bag with the highest initial concentration of sucrose? A) A B) B C) C D) D

A) A

Which of the following will have the greatest ratio of surface area to volume? A) A box that is 1×1×1. B) A box that is 2×2×2. C) A box that is 1×1×2. D) A box that is 2×2×1

A) A box that is 1×1×1.

Which of the following mechanisms is used to coordinate the expression of multiple, related genes in eukaryotic cells? A) A given gene may have multiple enhancers, but each enhancer is generally associated with only that gene and no other. B) The genes are organized into a large operon, allowing them to be coordinately controlled as a single unit. C) A single repressor is able to turn off several related genes. D) Environmental signals enter the cell and bind directly to promoters.

A) A given gene may have multiple enhancers, but each enhancer is generally associated with only that gene and no other.

During the elongation phase of translation, which site in the ribosome represents the location where a codon is being read? A) A site B) E site C) P site D) the large ribosomal subunit

A) A site

Which of the following processes occurs when termination of translation takes place? A) A stop codon is reached. B) The 5′ cap is reached. C) The end of the mRNA molecule is reached. D) The poly-A tail is reached.

A) A stop codon is reached.

Which of the following molecules are products of the light reactions of photosynthesis that are utilized in the Calvin cycle? A) ATP and NADPH B) H2O and O2 C) ADP, (P)i, and NADP+ D) CO2 and glucose

A) ATP and NADPH

The light reactions of photosynthesis supply the Calvin cycle with A) ATP and NADPH. B) CO2 and ATP. C) light energy. D) H2O and NADPH.

A) ATP and NADPH.

The enzyme phosphofructokinase (PFK) catalyzes a key step in glycolysis. PFK is inhibited by high levels of which of the following molecules? A) ATP and citrate B) glucose and NAD+ C) citrate and CO2 D) AMP and ATP

A) ATP and citrate

In the absence of oxygen, yeast cells can obtain energy by fermentation, which results in the production of which of the following sets of molecules? A) ATP, CO2, and ethanol (ethyl alcohol) B) ATP, CO2, and acetyl CoA C) ATP, NADH, and ethanol D) ATP, CO2, and lactate

A) ATP, CO2, and ethanol (ethyl alcohol)

A number of systems for pumping ions across membranes are powered by ATP. Such ATP-powered pumps are often called ATPases, although they do not often hydrolyze ATP unless they are simultaneously transporting ions. Because small increases in calcium ions in the cytosol can trigger a number of different intracellular reactions, cells keep the cytosolic calcium concentration quite low under normal conditions, using ATP-powered calcium pumps. For example, muscle cells transport calcium from the cytosol into the membranous system called the sarcoplasmic reticulum (SR). If a resting muscle cell's cytosol has a free calcium ion concentration of 10-7 while the concentration in the SR is 10-2, then how is the ATPase acting? A) ATPase activity must be pumping calcium from the cytosol to the SR against the concentration gradient. B) ATPase activity must be transferring (P)i to the SR to enable this to occur. C) ATPase activity must be powering an inflow of calcium from the outside of the cell into the SR. D) ATPase activity must be opening a channel for the calcium ions to diffuse back into the SR along the concentration gradient.

A) ATPase activity must be pumping calcium from the cytosol to the SR against the concentration gradient.

What would be the result of a mutation in a maternal effect gene in a female Drosophila? A) All of her offspring will show the mutant phenotype, regardless of their genotype. B) Only her male offspring will show the mutant phenotype. C) She will not develop past the early embryonic stage. D) Only her female offspring will show the mutant phenotype.

A) All of her offspring will show the mutant phenotype, regardless of their genotype.

Misfolding of polypeptides is a serious problem in cells. Which of the following diseases are associated with an accumulation of misfolded polypeptides? A) Alzheimer's and Parkinson's B) diabetes mellitus C) Parkinson's D) Alzheimer's

A) Alzheimer's and Parkinson's

The unlettered circle at the top of the figure shows a diploid nucleus with four chromosomes that have not yet replicated. There are two pairs of homologous chromosomes, one long and the other short. One haploid set is black, and the other is gray. The circles labeled A to E show various combinations of these chromosomes. What is the correct chromosomal condition at prometaphase of mitosis? A) B B) C C) D D) E

A) B

In human blood types, Rh positive is a trait that shows simple dominance over Rh negative. A woman who has blood type A positive has a daughter who is type O positive and a son who is type B negative. Which of the following phenotypes is possible for the father? A) B positive B) O negative C) A negative D) AB negative

A) B positive

Why does the fitness of a phenotype depend on frequency-dependent selection? A) Because frequency-dependent selection acts against extreme phenotypes. B) Because the least number of alleles are at that locus. C) Because selection favors the least common phenotype. D) Because sexual selection produces dimorphism.

A) Because frequency-dependent selection acts against extreme phenotypes.

When the Bicoid protein is expressed in Drosophila, divisions between cells in the embryo are not yet fully developed. This information helps to explain which observation by Nüsslein-Volhard and Wieschaus? A) Bicoid protein diffuses throughout the embryo in a concentration gradient. B) Bicoid protein determines the dorso-ventral axis of the embryo. C) Bicoid protein serves as a transcription regulator. D) mRNA from the egg is translated into the Bicoid protein.

A) Bicoid protein diffuses throughout the embryo in a concentration gradient.

Why has C. elegans proven to be a useful model for understanding apoptosis? A) C. elegans undergoes a fixed and easy-to-visualize number of apoptotic events during its normal development. B) As C. elegans ages, its cells die progressively until the whole organism is dead. C) C. elegans has large cells wherein apoptosis is easily observed without the aid of a microscope. D) C. elegans does not naturally use apoptosis, but can be induced to do so in the laboratory.

A) C. elegans undergoes a fixed and easy-to-visualize number of apoptotic events during its normal development.

The molecular formula for glucose is C6H12O6. What would be the molecular formula for a polymer made by linking ten glucose molecules together by dehydration reactions? A) C60H102O51 B) C60H100O50 C) C60H111O51 D) C60H120O60

A) C60H102O51

Which of the following metabolic processes can occur without a net influx of energy from some other process? A) C6H12O6 + 6O2 → 6CO2 + 6H2O B) 6CO2 + 6H20 → C6H12O6 + 6O2 C) ADP + (P)i → ATP + H2O D) Amino acids → Protein

A) C6H12O6 + 6O2 → 6CO2 + 6H2O

The cAMP receptor protein (CRP) is said to be responsible for positive regulation of the lac operon because ________. A) CRP bound to the CRP-binding site stimulates the transcription of the lac operon B) CRP binds cAMP C) CRP binds to the CAP-binding site D) CRP prevents binding of the repressor to the operator

A) CRP bound to the CRP-binding site stimulates the transcription of the lac operon

Which of the following calculations require the use of the addition rule of probability? A) Calculate the probability of a child having either sickle-cell anemia or cystic fibrosis if parents are each heterozygous for both. B) Calculate the probability of children with both cystic fibrosis and polydactyly when parents are each heterozygous for both genes. C) Calculate the probability of black offspring from the cross AaBb × AaBb, where B is the symbol for black. D) Calculate the probability of each of four children having cystic fibrosis if the parents are both heterozygous.

A) Calculate the probability of a child having either sickle-cell anemia or cystic fibrosis if parents are each heterozygous for both.

You would like to lose weight. Which of the following should be your preferred food group? A) Cellulose and fructose B) Lactose and glucose C) Starch and fructose D) Sucrose and starch

A) Cellulose and fructose

In what way do the membranes of a eukaryotic cell vary? A) Certain proteins are unique to each membrane. B) Only certain membranes of the cell are selectively permeable. C) Only certain membranes are constructed from amphipathic molecules. D) Phospholipids are found only in certain membranes.

A) Certain proteins are unique to each membrane.

What is the difference between covalent bonds and ionic bonds? A) Covalent bonds involve the sharing of electrons between atoms; ionic bonds involve the electrical attraction between charged atoms. B) Covalent bonds involve the transfer of electrons between charged atoms; ionic bonds involve the sharing of electrons between atoms. C) Covalent bonds involve the sharing of electrons between atoms; ionic bonds involve the sharing of protons between charged atoms. D) Covalent bonds involve the sharing of pairs of electrons between atoms; ionic bonds involve the sharing of single electrons between atoms.

A) Covalent bonds involve the sharing of electrons between atoms; ionic bonds involve the electrical attraction between charged atoms.

Hershey and Chase set out to determine what molecule served as the unit of inheritance. They completed a series of experiments in which E. coli was infected by a T2 virus. Which molecular component of the T2 virus actually ended up inside the cell? A) DNA B) ribosome C) protein D) RNA

A) DNA

DNA contains the template needed to copy itself, but it has no catalytic activity in cells. What catalyzes the formation of phosphodiester bonds between adjacent nucleotides in the DNA polymer being formed during DNA replication? A) DNA polymerase B) RNA primers C) ribozymes D) ATP

A) DNA polymerase

What is the basis for the difference in how the leading and lagging strands of DNA molecules are synthesized? A) DNA polymerase can join new nucleotides only to the 3′ end of a pre-existing strand, and the strands are antiparallel. B) The origins of replication occur only at the 5′ end. C) Helicases and single-strand binding proteins work at the 5′ end. D) DNA ligase works only in the 3′ → 5′ direction.

A) DNA polymerase can join new nucleotides only to the 3′ end of a pre-existing strand, and the strands are antiparallel.

New biosensors, applied like a temporary tattoo to the skin, can alert serious athletes that they are about to "hit the wall" and will find it difficult to continue exercising. These biosensors monitor lactate, a form of lactic acid, released in sweat during strenuous exercise. Which of the statements below is the best explanation of why athletes would need to monitor lactate levels? A) During anaerobic respiration, lactate levels increase when muscles cells need more energy; however, muscles cells eventually fatigue, thus athletes should modify their activities to increase aerobic respiration. B) During anaerobic respiration, muscle cells receive too little oxygen and begin to convert lactate to pyruvate (pyruvic acid), thus athletes experience cramping and fatigue. C) During aerobic respiration, muscle cells cannot produce enough lactate to fuel muscle cell contractions, and muscles begin to cramp, thus athletic performance suffers. D) During aerobic respiration, muscles cells produce too much lactate, which causes a rise in the pH of the muscle cells, thus athletes must consume increased amounts of sports drinks, high in electrolytes, to buffer the pH.

A) During anaerobic respiration, lactate levels increase when muscles cells need more energy; however, muscles cells eventually fatigue, thus athletes should modify their activities to increase aerobic respiration.

Cells that are infected, damaged, or have reached the end of their functional life span often undergo "programmed cell death." This controlled cell suicide is called apoptosis. Select the appropriate description of this event on a cell's life cycle. A) During apoptosis, cellular agents chop up the DNA and fragment the organelles and other cytoplasmic components of a cell. B) Each cell organelle has protein signals that initiate the breakdown of the organelle's components, which leads to cell death. C) Apoptosis is regulated by cell surface receptors that signal when a cell has reached its density-dependent limits. D) During apoptosis, dying cells leak out their contents, including digestive enzymes that also destroy healthy cells.

A) During apoptosis, cellular agents chop up the DNA and fragment the organelles and other cytoplasmic components of a cell.

Which of the following statements describes one characteristic of each chromosome in a cell during the entire process of meiosis I? A) Each chromosome consists of two sister chromatids joined by a centromere. B) Each chromosome is paired with a homologous chromosome. C) Each chromosome consists of a single strand of DNA. D) Each chromosome is joined with its homologous pair to form a synaptonemal complex.

A) Each chromosome consists of two sister chromatids joined by a centromere.

At puberty, an adolescent female body changes in both structure and function of several organ systems, primarily under the influence of changing concentrations of estrogen and other steroid hormones. How can one hormone, such as estrogen, mediate so many effects? A) Estrogen binds to specific receptors inside many kinds of cells, each with different responses. B) Estrogen is produced in very large concentration by nearly every tissue of the body. C) Each cell responds in the same way when steroids bind to the cell surface. D) Estrogen is kept away from the surface of any cells, not able to bind it at the surface.

A) Estrogen binds to specific receptors inside many kinds of cells, each with different responses.

Which of the following statements describes the process of transformation in bacteria? A) External DNA is taken into a cell, becoming part of the cell's genome. B) A strand of DNA is created from an RNA molecule. C) Bacterial cells are infected by a phage DNA molecule. D) A strand of RNA is created from a DNA molecule.

A) External DNA is taken into a cell, becoming part of the cell's genome.

During aerobic respiration, which of the following molecules directly donates electrons to the electron transport chain at the lowest energy level? A) FADH2 B) ATP C) water D) NADH

A) FADH2

The somatic cells of a privet shrub each contain a total of 46 chromosomes. How do the chromosomes of this plant differ from the chromosomes of humans, who also have a total of 46? A) Genes of privet shrub chromosomes are significantly different than those in humans. B) Privet shrubs must be metabolically more like animals than like other shrubs. C) Privet shrub cells cannot reproduce sexually. D) Privet shrub sex cells have chromosomes that can synapse with human chromosomes in the laboratory.

A) Genes of privet shrub chromosomes are significantly different than those in humans.

Which observation suggested to Sutherland the involvement of a second messenger in epinephrine's effect on liver cells? A) Glycogen breakdown was observed only when epinephrine was administered to intact cells. B) Epinephrine was known to have different effects on many types of cells. C) Receptor studies indicated that epinephrine was a ligand. D) Glycogen breakdown was observed when epinephrine and glycogen phosphorylase were combined in a cell-free system.

A) Glycogen breakdown was observed only when epinephrine was administered to intact cells.

In research on aging (both cellular aging and organismal aging), it has been found that aged cells do not progress through the cell cycle as they had previously. Which of the following, if found in cells or organisms as they age, would provide evidence that this is related to cell signaling? A) Growth factor ligands do not bind as efficiently to receptors. B) Hormone concentrations decrease. C) cAMP levels change very frequently. D) Enzymatic activity declines.

A) Growth factor ligands do not bind as efficiently to receptors.

The atomic number of sulfur is 16. Sulfur combines with hydrogen by covalent bonding to form a compound, hydrogen sulfide. Based on the number of valence electrons in a sulfur atom, predict the molecular formula of the compound. A) H2S B) HS C) HS2 D) H4S

A) H2S

Which of the following events happens at the conclusion of meiosis I? A) Homologous chromosomes of a pair are separated from each other. B) The chromosome number per cell remains the same. C) Sister chromatids are separated. D) Four daughter cells are formed.

A) Homologous chromosomes of a pair are separated from each other.

DNA was isolated from three different cell types of the same organism, the relative DNA content for each type was determined, and the results were plotted on the graph shown in the figure below. Refer to the graph to answer the following questions. Which sample might represent a zygote? A) I B) II C) III D) either I or II E) either II or III

A) I

Refer to the drawings in the figure below of a single pair of homologous chromosomes as they might appear during various stages of either mitosis or meiosis, and answer the following question(s). Which diagram represents anaphase I of meiosis? A) I B) IV C) V D) II

A) I

Refer to the life cycles illustrated in the figure below to answer the following questions. Which of the life cycles is typical for animals? A) I only B) I and III C) II only D) III only

A) I only

For the following questions, match the key event of meiosis with the stages listed below. I. Prophase I V. Prophase II II. Metaphase I VI. Metaphase II III. Anaphase I VII. Anaphase II IV. Telophase I VIII. Telophase II Homologous chromosomes are aligned at the equator of the spindle. A) II B) I C) IV D) VI

A) II

Eukaryotic sexual life cycles show tremendous variation. Which of the following characteristics do all sexual life cycles have in common? I. Alternation of generations II. Meiosis III. Fertilization IV. Gametes V. Spores A) II, III, and IV B) I, II, III, IV, and V C) I, II, and IV D) II, IV, and V

A) II, III, and IV

In eukaryotes, genetic information is passed to the next generation by processes that include mitosis or meiosis. Which of the following explanations correctly identifies the process and supports the claim that heritable information is passed from one generation to another? A) In asexual reproduction, a single individual is the sole parent and passes copies of its genes to its offspring without the fusion of gametes. B) During mitosis, DNA replication occurs twice within the cell cycle to insure a full set of chromosomes within each of the daughter cells produced. C) Mitosis, followed by cytokinesis, produces daughter cells that are genetically different from the parent cell, thus insuring variation within the population. D) Single-celled organisms can fuse their cells, reproducing asexually through mitosis to form new cells that are not identical to the parent cell.

A) In asexual reproduction, a single individual is the sole parent and passes copies of its genes to its offspring without the fusion of gametes.

Nucleotides can be radiolabeled before they are incorporated into newly forming DNA and, therefore, can be assayed to track their incorporation. In a set of experiments, a student-faculty research team used labeled T nucleotides to study the incorporation of labeled nucleotides into a culture of lymphocytes. They found that the lymphocytes incorporated the labeled nucleotide at a significantly higher level after a pathogen was introduced into the culture. What might they conclude from this observation? A) Infection causes lymphocytes to divide more rapidly. B) Infection causes lymphocyte cultures to skip some parts of the cell cycle. C) The pathogen consumed radiolabeled nucleotides. D) Infection causes lymphocytes to increase in size.

A) Infection causes lymphocytes to divide more rapidly.

Which of the following statements is correct regarding the human X chromosomes? A) It carries genes that determine an individual's biological sex. B) It is referred to as an autosome. C) It is the same size as other chromosomes and has the same number of genes. D) It is present in every somatic cell of males and females.

A) It carries genes that determine an individual's biological sex.

Which of the following statements describes a characteristic feature of a carrier protein in a plasma membrane? A) It exhibits specificity for a particular type of molecule. B) It works against diffusion. C) It requires the expenditure of cellular energy to function. D) It has no hydrophobic regions.

A) It exhibits specificity for a particular type of molecule.

Use the following information to answer the question. In a Drosophila experiment, a cross was made between homozygous wild-type females and yellow-bodied males. All of the resulting F1s were phenotypically wild type. However, adult flies of the F2 generation (resulting from matings of the F1s) had the characteristics shown in the figure. How is the mutant allele for yellow body inherited? A) It is X-linked. B) It is inherited by X inactivation. C) It is not X-linked. D) It is Y-linked.

A) It is X-linked.

A 0.01 M solution of a substance has a pH of 2. What can you conclude about this substance? A) It is a strong acid that dissociates completely in water. B) It is a weak base. C) It is a strong base that dissociates completely in water. D) It is a weak acid.

A) It is a strong acid that dissociates completely in water.

Which of the following statements correctly describes a karyotype? A) It is an organized image of a cell's chromosomes. B) It reveals the appearance of an organism. C) It is a display of a cell's mitotic stages. D) It is a display of all of the cell types in an organism.

A) It is an organized image of a cell's chromosomes.

Which of the following statements best describes the significance of the TATA box in the promoters of eukaryotes? A) It is the recognition site for the binding of a specific transcription factor. B) It sets the reading frame of the mRNA during translation. C) It is the recognition site for ribosomal binding during transcription. D) It is the recognition site for ribosomal binding during translation.

A) It is the recognition site for the binding of a specific transcription factor.

Which of the following statements about the DNA in one of your brain cells is true? A) It is the same as the DNA in one of your liver cells. B) Each gene lies immediately adjacent to an enhancer. C) Most of the DNA codes for protein. D) The majority of genes are likely to be transcribed.

A) It is the same as the DNA in one of your liver cells.

What is the role of DNA ligase in the elongation of the lagging strand during DNA replication? A) It joins Okazaki fragments together. B) It stabilizes the unwound parental DNA. C) It unwinds the parental double helix. D) It synthesizes RNA nucleotides to make a primer.

A) It joins Okazaki fragments together.

Consider the following reaction at equilibrium: CO2 + H2O ⇌H2CO3. What would be the effect of adding additional H2O? A) It would drive the equilibrium dynamics to the right. B) It would drive the equilibrium dynamics to the left. C) Reactions in both the directions will slow down. D) Nothing would happen because the reactants and products are in equilibrium.

A) It would drive the equilibrium dynamics to the right.

The following question refers to this table of codons. What amino acid sequence will be generated, based on the following mRNA codon sequence? 5′-AUG-UCU-UCG-UUA-UCC-UUG-3′ A) Met-Ser-Ser-Leu-Ser-Leu B) Met-Arg-Glu-Arg-Glu-Arg C) Met-Glu-Arg-Arg-Glu-Leu D) Met-Ser-Leu-Ser-Leu-Ser

A) Met-Ser-Ser-Leu-Ser-Leu

Which electron carrier(s) function in the citric acid cycle? A) NADH and FADH2 B) NAD+ only C) the electron transport chain D) ADP and ATP

A) NADH and FADH2

A couple has a child with Down syndrome. The mother is 39 years old at the time of delivery. Which of the following is the most probable cause of the child's condition? A) One of the gametes in the mother most likely underwent nondisjunction during meiosis. B) The mother had a chromosomal duplication. C) One member of the couple underwent nondisjunction in somatic cell production. D) The woman inherited this tendency from her parents.

A) One of the gametes in the mother most likely underwent nondisjunction during meiosis.

One of two major forms of a human condition called neurofibromatosis (NF1) is inherited as a dominant gene, although it may range from mildly to very severely expressed. Which of the following is the best explanation for why a young, affected child is the first in her family to be diagnosed? A) One of the parents has a mild expression of the gene. B) The condition skipped a generation in the family. C) The mother carries the gene but does not express it. D) The child has one more chromosome than either of the parents.

A) One of the parents has a mild expression of the gene.

In autumn, chlorophyll is degraded in the leaves of deciduous trees. Why do the leaves change color to shades of yellow, orange, or red? A) Other pigments such as carotenoids are still present in the leaves. B) Degraded chlorophyll changes into many other colors. C) In the absence of photosynthesis, the leaves produce energy exclusively by aerobic cellular respiration. D) Sugars from sap fill the leaves prior to winter.

A) Other pigments such as carotenoids are still present in the leaves.

The citric acid cycle. If pyruvate oxidation is blocked, what will happen to the levels of oxaloacetate and citric acid in the citric acid cycle shown in the accompanying figure? A) Oxaloacetate will accumulate and citric acid will decrease. B) Both oxaloacetate and citric acid will accumulate. C) Both oxaloacetate and citric acid will decrease. D) Oxaloacetate will decrease and citric acid will accumulate.

A) Oxaloacetate will accumulate and citric acid will decrease.

Which of the following statements best describes the relationship between photosynthesis and cellular respiration? A) Photosynthesis stores energy in complex organic molecules; cellular respiration releases energy from complex organic molecules. B) Photosynthesis is catabolic; cellular respiration is anabolic. C) Cellular respiration runs the biochemical pathways of photosynthesis in reverse. D) Photosynthesis occurs only in plants; cellular respiration occurs only in animals.

A) Photosynthesis stores energy in complex organic molecules; cellular respiration releases energy from complex organic molecules.

If 14C-labeled uracil is added to the growth medium of cells, what macromolecules will be labeled? A) RNA B) polypeptides C) both DNA and RNA D) DNA

A) RNA

Why is the ΔG of ATP hydrolysis in the cell about twice as great as the ΔG of ATP hydrolysis in a test tube under standard conditions? A) Reactant and product concentrations in the test tube are different from those in the cell. B) A cell is an open system, whereas a test tube is a closed system. C) ATP hydrolysis in a test tube releases more heat than ATP hydrolysis associated with cellular metabolism. D) ATP hydrolysis in cells is catalyzed by enzymes, whereas the reaction in a test tube occurs spontaneously.

A) Reactant and product concentrations in the test tube are different from those in the cell.

Phenylketonuria (PKU) is a recessive human disorder in which an individual cannot appropriately metabolize the amino acid phenylalanine. This amino acid is not naturally produced by humans. Which of the following treatments would be most effective for people with PKU? A) Regulate the diet of the affected persons to severely limit the uptake of phenylalanine. B) Feed them the substrate that can be metabolized into phenylalanine. C) Feed the patients an excess of the missing product. D) Feed the patients the missing enzymes in a regular cycle, such as twice per week.

A) Regulate the diet of the affected persons to severely limit the uptake of phenylalanine.

In DNA replication in E. coli, the enzyme primase is used to attach a 5 to 10 base ribonucleotide strand complementary to the parental DNA strand. The RNA strand serves as a starting point for the DNA polymerase that replicates the DNA. If a mutation occurred in the primase gene, which of the following results would you expect? A) Replication would not occur on either the leading or lagging strand. B) Replication would not be affected as the enzyme primase in involved with RNA synthesis. C) Replication would only occur on the leading strand. D) Replication would only occur on the lagging strand.

A) Replication would not occur on either the leading or lagging strand.

Which one of the following statements about RNA processing is correct? A) Ribozymes may function in RNA splicing. B) Exons are cut out before mRNA leaves the nucleus. C) A primary transcript is often much shorter than the final RNA molecule that leaves the nucleus. D) RNA splicing can be catalyzed by tRNA.

A) Ribozymes may function in RNA splicing.

Which of the following statements is true about proteins? A) Some proteins form a complete 3-D structure only when they interact with their targets B) Denaturation leads to bond disruption, and the molecule turns into liquid C) Denaturation is always irreversible D) Final folded structure can reveal the steps of protein folding

A) Some proteins form a complete 3-D structure only when they interact with their targets

Which of the following statements about the 5 end of a polynucleotide strand of RNA is correct? A) The 5 end has a phosphate group attached to the number 5 carbon of ribose. B) The 5 end has a carboxyl group attached to the number 5 carbon of ribose. C) The 5 end has a hydroxyl group attached to the number 5 carbon of ribose. D) The 5 end has phosphate attached to the number 5 carbon of the nitrogenous base.

A) The 5 end has a phosphate group attached to the number 5 carbon of ribose.

If a cell were unable to produce histone proteins, which of the following results would be a likely effect on the cell? A) The cell's DNA could not be packed into its nucleus. B) Spindle fibers would not form during prophase. C) There would be an increase in the amount of DNA produced during replication. D) Amplification of other genes would compensate for the lack of histones.

A) The cell's DNA could not be packed into its nucleus.

How do cells at the completion of meiosis compare with cells that are in prophase of meiosis I? A) The cells have half the number of chromosomes and one-fourth the amount of DNA. B) The cells have half the number of chromosomes and half the amount of DNA. C) The cells have the same number of chromosomes and half the amount of DNA. D) The cells have half the amount of cytoplasm and twice the amount of DNA.

A) The cells have half the number of chromosomes and one-fourth the amount of DNA.

Which of the following statements about independent assortment or segregation is correct? A) The law of independent assortment describes the behavior of two or more genes relative to one another. B) The law of segregation is accounted for by anaphase of mitosis. C) The law of independent assortment is accounted for by observations of prophase I of meiosis. D) The law of segregation describes the behavior of two or more genes relative to one another.

A) The law of independent assortment describes the behavior of two or more genes relative to one another.

Gaucher disease is the most common of lipid storage diseases in humans. It is caused by a deficiency of an enzyme necessary for the breakdown of lipids. Using your knowledge of the structure of eukaryotic cells, identify the statement below that best explains how internal membranes and the organelles of cells may be involved in Gaucher disease. A) The lysosomes lack sufficient amounts of enzymes necessary for the metabolism of lipids. B) The mitochondria are most likely defective and do not produce adequate amounts of ATP needed for cellular energy. C) The rough endoplasmic reticulum contains too many ribosomes, which results in an overproduction of the enzyme involved in lipid breakdown. D) The Golgi apparatus produces vesicles with faulty membranes, which fail to be transported to the plasma membrane for secretion.

A) The lysosomes lack sufficient amounts of enzymes necessary for the metabolism of lipids.

H. V. Wilson worked with sponges to gain some insight into exactly what was responsible for holding adjacent cells together. He exposed two species of differently pigmented sponges to a chemical that disrupted intercellular junctions, and the cells of the sponges dissociated. Wilson then mixed the cells of the two species and removed the chemical that caused the cells to dissociate. Wilson found that the sponges reassembled into two separate species. The cells from one species did not interact or form associations with the cells of the other species. How do you explain the results of Wilson's experiments? A) The molecules responsible for cell-cell adhesion (cell junctions) differed between the two species of sponge. B) The molecules responsible for cell-cell adhesion (cell junctions) were irreversibly destroyed during the experiment. C) One cell functioned as an organizer for each organism, thereby attracting only cells of the same species. D) The two species of sponge had different enzymes that functioned in the reassembly process.

A) The molecules responsible for cell-cell adhesion (cell junctions) differed between the two species of sponge.

Imagine that you've isolated a yeast mutant that contains histones resistant to acetylation. What phenotype would you predict for this mutant? A) The mutant will show decreased levels of gene expression. B) The mutant will require galactose for growth. C) The mutant will show increased levels of gene expression. D) The mutant will grow rapidly.

A) The mutant will show decreased levels of gene expression.

Agrobacterium infects plants and causes them to form tumors. You determine that tumor formation requires a large amount of the plant's energy for tissue formation. How might this change the number of offspring a plant produces, and what is the most likely explanation for this change? A) The number of offspring should decrease because the plant will divert energy from reproduction to tumor formation. B) The number of offspring should increase because in general, illness increases the reproductive output of organisms. C) There should be no effect of infection on offspring production because energy for reproduction is independent of infection. D) The number of offspring should increase because the bacteria will provide energy for the plant.

A) The number of offspring should decrease because the plant will divert energy from reproduction to tumor formation.

Which of the following statements correctly describes any chemical reaction that has reached equilibrium? A) The rates of the forward and reverse reactions are equal. B) Both forward and reverse reactions have halted. C) The concentrations of products and reactants are equal. D) The reaction is now irreversible.

A) The rates of the forward and reverse reactions are equal.

Which of the following statements is true for all exergonic reactions? A) The reaction proceeds with a net release of free energy. B) The products have more total energy than the reactants. C) The reaction goes only in a forward direction: all reactants will be converted to products, but no products will be converted to reactants. D) A net input of energy from the surroundings is required for the reactions to proceed.

A) The reaction proceeds with a net release of free energy.

Refer to the life cycles illustrated in the figure below to answer the following questions. In a life cycle such as that shown in part III of the figure above, if the zygote's chromosome number is ten, which of the following statements will be true? A) The sporophyte's chromosome number per cell is ten and the gametophyte's is five. B) The sporophyte and gametophyte each have five chromosomes per cell. C) The sporophyte's chromosome number per cell is five and the gametophyte's is ten. D) The sporophyte and gametophyte each have ten chromosomes per cell.

A) The sporophyte's chromosome number per cell is ten and the gametophyte's is five.

Which of the following statements is true for a system at chemical equilibrium? A) The system can do no work. B) The system consumes energy at a steady rate. C) The system releases energy at a steady rate. D) The kinetic energy of the system is zero.

A) The system can do no work.

Cell membranes have distinct inside and outside faces. Which of the following statements is the most likely explanation for the membrane's asymmetrical nature? A) The two sides of a cell membrane face different environments and carry out different functions. B) Proteins only function on the cytoplasmic side of the cell membrane, which results in the membrane's asymmetrical nature. C) Since the cell membrane forms a border between one cell and another in tightly packed tissues such as epithelium, the membrane must be asymmetrical. D) Since cell membranes communicate signals from one organism to another, the cell membranes must be asymmetrical.

A) The two sides of a cell membrane face different environments and carry out different functions.

Why are BRCA1 and BRCA2 considered to be tumor-suppressor genes? A) Their normal products participate in repair of DNA damage. B) The normal genes make estrogen receptors. C) The mutant forms of either one of these prevent breast cancer. D) They block penetration of breast cells by chemical carcinogens.

A) Their normal products participate in repair of DNA damage.

When a cell is in equilibrium with its environment, which of the following processes occurs for substances that can diffuse through the plasma membrane? A) There is random movement of substances into and out of the cell. B) All movement of molecules across the plasma membrane occurs by active transport. C) There is directed movement of substances into and out of the cell. D) There is no movement of substances into or out of the cell.

A) There is random movement of substances into and out of the cell.

Which of the following statements describes a common characteristic of catabolic pathways? A) They are exergonic and provide energy that can be used to produce ATP from ADP and (P)i. B) They combine small molecules into larger, more energy-rich molecules. C) They are endergonic and release energy that can be used for cellular work. D) They require energy from ATP hydrolysis to break down polymers into monomers.

A) They are exergonic and provide energy that can be used to produce ATP from ADP and (P)i.

Protein kinases are enzymes that transfer the terminal phosphate from ATP to an amino acid residue on the target protein. Many are located on the plasma membrane as integral membrane proteins or peripheral membrane proteins. What purpose may be served by their plasma membrane localization? A) They can more readily encounter and phosphorylate other membrane proteins. B) They flip back and forth across the membrane to access target proteins on either side. C) Membrane localization lowers the activation energy of the phosphorylation reaction. D) ATP is more abundant near the plasma membrane.

A) They can more readily encounter and phosphorylate other membrane proteins.

What role do phosphatases play in signal transduction pathways? A) They inactivate protein kinases to turn off signal transduction. B) They activate protein kinases by phosphorylation. C) They transfer a phosphate group from one protein in the pathway to the next molecule in the series. D) They amplify the second messenger cAMP.

A) They inactivate protein kinases to turn off signal transduction.

Why are C4 plants able to photosynthesize with no apparent photorespiration? A) They use PEP carboxylase to initially fix CO2. B) They conserve water more efficiently. C) They exclude oxygen from their tissues. D) They do not participate in the Calvin cycle.

A) They use PEP carboxylase to initially fix CO2.

Fatty acids usually have an even number of carbons in their structures. Catabolism of fatty acids produces two-carbon fragments that are converted to acetyl CoA molecules. What is the most likely way in which these acetyl CoA molecules would be metabolized in aerobic cellular respiration? A) They would directly enter the citric acid cycle. B) They would be converted to pyruvate and then undergo pyruvate oxidation upon transport into mitochondria. C) They would directly enter the electron transport chain. D) They would directly enter the energy-yielding phase of glycolysis.

A) They would directly enter the citric acid cycle.

Which of the following statements correctly describes how Mendel accounted for the observation that traits had disappeared in the F1 generation and then reappeared in the F2 generation? A) Traits can be dominant or recessive, and the recessive traits were "hidden" by the dominant ones in the F1. B) Members of the F1 generation had only one allele for each trait, but members of the F2 had two alleles for each trait. C) New mutations were frequently generated in the F2progeny, causing traits that had been lost in the F1 to reappear in the F2. D) The mechanism controlling the appearance of traits was different between the F1 and the F2 plants.

A) Traits can be dominant or recessive, and the recessive traits were "hidden" by the dominant ones in the F1.

Which of the following statements is true about protein synthesis in prokaryotes? A) Translation can begin while transcription is still in progress. B) Prokaryotic cells have complicated mechanisms for targeting proteins to the appropriate cellular organelles. C) Unlike eukaryotes, prokaryotes require no initiation or elongation factors. D) Extensive RNA processing is required before prokaryotic transcripts can be translated.

A) Translation can begin while transcription is still in progress.

Which of the following is an example of monosomy in humans? A) Turner syndrome B) Down syndrome C) trisomy X D) Klinefelter syndrome

A) Turner syndrome

Refer to the drawings in the figure below of a single pair of homologous chromosomes as they might appear during various stages of either mitosis or meiosis, and answer the following question(s). Which diagram represents anaphase II of meiosis? A) V B) III C) I D) IV

A) V

Which of the following statements is the most current description of a gene? A) a DNA sequence that is expressed to form a functional product: either RNA or polypeptide B) a discrete unit of hereditary information that consists of a sequence of amino acids C) a DNA subunit that codes for a single complete protein D) a unit of heredity that causes formation of a phenotypic characteristic

A) a DNA sequence that is expressed to form a functional product: either RNA or polypeptide

Refer to the following figure to answer the questions below. What results from the chemical reaction in the illustration? The reactants have no charge. A) a cation with a net charge of +1 and an anion with a net charge of -1 B) a cation with a net charge of -1 and an anion with a net charge of +1 C) a cation with a net charge of -1 and an anion with a net charge of -1 D) a cation with a net charge of +1 and an anion with a net charge of +1

A) a cation with a net charge of +1 and an anion with a net charge of -1

Chemical equilibrium is relatively rare in living cells because metabolic pathways are interconnected. Which of the following statements describes an example of a reaction that may be at chemical equilibrium in a cell? A) a chemical reaction in which neither the reactants nor the products are being produced or consumed in any metabolic pathway at that time in the cell B) an exergonic reaction in which the free energy at equilibrium is higher than the energy content of the reaction at any point away from equilibrium C) an endergonic reaction in an active metabolic pathway where the energy for that reaction is supplied only by heat from the environment D) an exergonic reaction in which the entropy change in the cell is precisely balanced by an opposite entropy change in the cell's surroundings

A) a chemical reaction in which neither the reactants nor the products are being produced or consumed in any metabolic pathway at that time in the cell

The force driving simple diffusion is ________, while the energy source for active transport is ________. A) a concentration gradient; ATP hydrolysis B) transmembrane pumps; an electrochemical gradient C) a concentration gradient; ADP D) phosphorylated carrier proteins; ATP

A) a concentration gradient; ATP hydrolysis

What is a syndrome? A) a group of traits typically found in conjunction with a particular chromosomal aberration or gene mutation B) a characteristic facial appearance C) a trait that leads to cancer at some stage in life D) a specific characteristic that appears in conjunction with one specific aneuploidy

A) a group of traits typically found in conjunction with a particular chromosomal aberration or gene mutation

Which of the following descriptions best fits the class of molecules known as nucleotides? A) a nitrogenous base, a phosphate group, and a sugar B) a nitrogenous base and a sugar C) a nitrogenous base and a phosphate group D) a sugar and a purine or pyrimidine

A) a nitrogenous base, a phosphate group, and a sugar

Each bond in carbon dioxide represents ________. O = C = O A) a pair of shared electrons B) a pair of protons C) one resonating electron D) two pairs of shared electrons

A) a pair of shared electrons

A glycosidic linkage is analogous to which of the following in proteins? A) a peptide bond B) an amino group C) a β pleated sheet D) a disulfide bond

A) a peptide bond

Through a microscope, you can see a cell plate beginning to develop across the middle of a cell and nuclei forming on either side of the cell plate. This cell is most likely ________. A) a plant cell in the process of cytokinesis B) an animal cell in anaphase of mitosis C) an animal cell in the process of cytokinesis D) a plant cell in metaphase of mitosis

A) a plant cell in the process of cytokinesis

Which of the following mutations would be most likely to have a harmful effect on an organism? A) a single nucleotide insertion downstream of, and close to, the start of the coding sequence B) a single nucleotide deletion in the middle of an intron C) a single nucleotide deletion near the end of the coding sequence D) a deletion of three nucleotides near the middle of a gene

A) a single nucleotide insertion downstream of, and close to, the start of the coding sequence

If an enzyme in solution is saturated with substrate, the most effective way to obtain a faster yield of products is to A) add more of the enzyme. B) heat the solution to 90°C. C) add a noncompetitive inhibitor. D) add more substrate.

A) add more of the enzyme.

Post-translational modifications of proteins may include which of the following processes? A) addition of carbohydrates to form a glycoprotein B) addition of a poly-A tail C) addition of a 5′ cap D) removal of introns

A) addition of carbohydrates to form a glycoprotein

Exposing inner mitochondrial membranes to ultrasonic vibrations will disrupt the membranes. However, the fragments will reseal "inside out." The little vesicles that result can still transfer electrons from NADH to oxygen and synthesize ATP. After the disruption, which components involved in oxidative phosphorylation must be present for electron transfer and ATP synthesis to still occur? A) all of the electron transport proteins as well as ATP synthase B) all of the electron transport system and the ability to add CoA to acetyl groups. C) the ATP synthase system is sufficient D) the electron transport system is sufficient

A) all of the electron transport system and ATP synthase

Gene S controls the sharpness of spines in a type of cactus. Cacti with the dominant allele, S, have sharp spines, whereas homozygous recessive ss cacti have dull spines. At the same time, a second gene, N, determines whether or not cactuses have spines. Homozygous recessive nn cactuses have no spines at all. A cross between a true-breeding sharp-spined cactus and a spineless cactus would produce ________. A) all sharp-spined progeny B) 50% sharp-spined, 50% dull-spined progeny C) It is impossible to determine the phenotypes of the progeny. D) 25% sharp-spined, 50% dull-spined, 25% spineless progeny

A) all sharp-spined progeny

Altering patterns of gene expression in prokaryotes would most likely serve an organism's survival by ________. A) allowing an organism to adjust to changes in environmental conditions B) allowing environmental changes to alter a prokaryote's genome C) allowing each gene to be expressed an equal number of times D) organizing gene expression, so that genes are expressed in a given order

A) allowing an organism to adjust to changes in environmental conditions

A series of enzymes catalyze the reactions in the metabolic pathway X → Y → Z → A. Product A binds to the enzyme that converts X to Y at a position remote from its active site. This binding decreases the activity of the enzyme. With respect to the enzyme that converts X to Y, substance A functions as ________. A) an allosteric inhibitor B) the substrate C) an intermediate D) a competitive inhibitor

A) an allosteric inhibitor

Oxygen has an atomic number of 8 and, most commonly, a mass number of 16. Thus, what is the atomic mass of an oxygen atom? A) approximately 16 daltons B) approximately 8 daltons C) approximately 16 grams D) approximately 8 grams

A) approximately 16 daltons

Which of the following types of cells utilize deoxyribonucleic acid (DNA) as their genetic material but do not have their DNA encased within a nuclear envelope? A) archaean B) plant C) animal D) fungi

A) archaean

Accuracy in the translation of mRNA into the primary structure of a polypeptide depends on specificity in the ________. A) binding of the anticodon to the codon and the attachment of amino acids to tRNAs B) attachment of amino acids to rRNAs C) binding of ribosomes to mRNA D) binding of the anticodon to small subunit of the ribosome

A) binding of the anticodon to the codon and the attachment of amino acids to tRNAs

A spaceship is designed to support animal life for a multiyear voyage to the outer planets of the solar system. Plants will be grown to provide oxygen and to recycle carbon dioxide. Since the spaceship will be too far from the sun for photosynthesis, an artificial light source will be needed. Suppose a plant has a unique photosynthetic pigment and the leaves of this plant appear to be reddish yellow. What wavelengths of visible light are absorbed by this pigment? A) blue and violet B) green and yellow C) red and yellow D) green and red

A) blue and violet

How might a change of one amino acid at a site, distant from the active site of an enzyme, alter the substrate specificity of an enzyme? A) by changing the three-dimensional conformation of the enzyme B) by changing the optimum pH for the enzyme C) by changing the stability of the enzyme D) by changing the binding site for a noncompetitive inhibitor

A) by changing the three-dimensional conformation of the enzyme

Consider this pathway: epinephrine → G protein-coupled receptor → G protein→ adenylyl cyclase → cAMP The second messenger in this pathway is ________. A) cAMP B) G protein C) G protein-coupled receptor D) adenylyl cyclase

A) cAMP

According to the fluid mosaic model of cell membranes, phospholipids ________. A) can move laterally along the plane of the membrane B) have hydrophilic tails in the interior of the membrane C) frequently flip-flop from one side of the membrane to the other D) occur in an uninterrupted bilayer, with membrane proteins restricted to the surface of the membrane

A) can move laterally along the plane of the membrane

Which of the following categories includes all others in the list? A) carbohydrate B) disaccharide C) polysaccharide D) starch

A) carbohydrate

In the cells of some organisms, mitosis occurs without cytokinesis. This will result in A) cells with more than one nucleus. B) cells lacking nuclei. C) cell cycles lacking an S phase. D) cells that are unusually small.

A) cells with more than one nucleus.

Which organelle often takes up much of the volume of a plant cell? A) central vacuole B) Golgi apparatus C) lysosome D) chloroplast

A) central vacuole

Cells are ..... A) characteristic of prokaryotic and eukaryotic organisms B) only found in pairs, because single cells cannot exist independently C) limited in size to 200 and 500 micrometers in diameter D) characteristic of eukaryotic but not prokaryotic organisms

A) characteristic of prokaryotic and eukaryotic organisms

Thylakoids, DNA, and ribosomes are all components found in _____. A) chloroplasts B) mitochondria C) lysosomes D) nuclei

A) chloroplasts

A genetic test to detect predisposition to cancer would likely examine the APC gene for involvement in which type(s) of cancer? A) colorectal only B) lung and prostate C) lung and breast D) lung only

A) colorectal only

Which of the following conditions may be overcome by increasing the substrate concentration in an enzymatic reaction with a fixed amount of enzyme? A) competitive inhibition B) the need for a coenzyme C) allosteric inhibition D) noncompetitive inhibition

A) competitive inhibition

The anticodon of a particular tRNA molecule is A) complementary to the corresponding mRNA codon. B) the part of tRNA that bonds to a specific amino acid. C) complementary to the corresponding triplet in rRNA. D) catalytic, making the tRNA a ribozyme.

A) complementary to the corresponding mRNA codon.

Motor proteins provide for molecular motion in cells by interacting with what types of cellular structures? A) components of the cytoskeleton B) membrane proteins of the inner nuclear envelope C) free ribosomes and ribosomes attached to the ER D) cellulose fibers in the cell wall

A) components of the cytoskeleton

A signal transmitted via phosphorylation of a series of proteins is generally associated with which of the following events? A) conformational changes to each protein in the series B) requires binding of a hormone to a cytosol receptor. C) cannot occur in yeasts because they lack protein phosphatases. D) requires phosphorylase activity.

A) conformational changes to each protein in the series

Which of the following processes would result from a mutation that deactivates a regulatory gene of a repressible operon in an E. coli cell? A) continuous transcription of the structural gene controlled by that regulator B) continuous translation of the mRNA because of alteration of its structure C) irreversible binding of the repressor to the operator D) complete inhibition of transcription of the structural gene controlled by that regulator

A) continuous transcription of the structural gene controlled by that regulator

The phosphate transport system in bacteria imports phosphate into the cell even when the concentration of phosphate outside the cell is much lower than the cytoplasmic phosphate concentration. Phosphate import depends on a pH gradient across the membrane—more acidic outside the cell than inside the cell. In this bacterial cell, phosphate transport is an example of ________. A) cotransport B) passive diffusion C) facilitated diffusion D) active transport

A) cotransport

Activity of various enzymes at various temperatures (a) and at various pH (b). Which temperature and pH profile curves on the graphs are most likely associated with an enzyme isolated from a human stomach where conditions are strongly acid? A) curves 1 and 4 B) curves 3 and 4 C) curves 1 and 5 D) curves 2 and 4

A) curves 1 and 4

Which of the following molecules is a protein synthesized at specific times during the cell cycle that associates with a kinase to form a catalytically active complex? A) cyclin B) PDGF C) MPF D) Cdk

A) cyclin

Cotton-topped tamarins are small primates with tufts of long white hair on their heads. While studying these creatures, you notice that males with longer hair get more opportunities to mate and father more offspring. To test the hypothesis that having longer hair is adaptive in these males, you should ________. A) determine if hair length is heritable B) look for evidence of hair in ancestors of tamarins C) test whether males with shaved heads are still able to mate D) test whether other traits in these males are also adaptive

A) determine if hair length is heritable

The fact that plants can be cloned from somatic cells demonstrates that ________. A) differentiated cells retain all the genes of the zygote B) genes are lost during differentiation C) differentiation does not occur in plants D) the differentiated state is normally very unstable

A) differentiated cells retain all the genes of the zygote

A biologist doing a long-term study on a wild spider population observes increased variation in silk thickness. Which of the following could the spider population be experiencing? A) disruptive selection B) directional selection C) stabilizing selection D) genetic drift

A) disruptive selection

Cooking oil and gasoline (a hydrocarbon) are not amphipathic molecules because they ________. A) do not have a polar or charged region B) have hydrophobic and hydrophilic regions C) are highly reduced molecules D) do not have a nonpolar region

A) do not have a polar or charged region

The relationship between catabolism and anabolism is most similar to the relationship between which of the following pairs of terms? A) exergonic; endergonic B) work; free energy C) free energy; entropy D) exergonic; spontaneous

A) exergonic; endergonic

Which of the following processes occurs in a plant's sexual life cycle? A) gametophytes produce gametes by mitosis B) sporophytes produce gametes by mitosis C) gametophytes produce gametes by meiosis D) sporophytes produce gametes by meiosis

A) gametophytes produce gametes by mitosis

Comparisons of Neanderthal DNA revealed that there are more similarities to non-African DNA than reference sequences from West Africans. Additionally, scientists found that Neanderthal DNA is as closely related to East Asians as to Europeans. This indicates that interbreeding occurred before human migration further east. What process of population genetics generated these results? A) gene flow B) nonrandom mating C) adaptive evolution D) gene drift

A) gene flow

If the nucleotide variability of a locus equals 0%, what is the gene variability and number of alleles at that locus? A) gene variability = 0%; number of alleles = 1 B) gene variability > 0%; number of alleles = 2 C) gene variability = 0%; number of alleles = 0 D) gene variability = 0%; number of alleles = 2

A) gene variability = 0%; number of alleles = 1

There are 25 individuals in population 1, all with genotype AA, and there are 40 individuals in population 2, all with genotype aa. Assume that these populations are located far from each other and that their environmental conditions are very similar. Based on the information given here, the observed genetic variation most likely resulted from A) genetic drift. B) directional selection. C) gene flow. D) nonrandom mating.

A) genetic drift.

What is the final result of mitosis in a human? A) genetically identical 2n somatic cells B) genetically different 2n somatic cells C) genetically identical 1n somatic cells D) genetically identical 2n gamete cells

A) genetically identical 2n somatic cells

Yeast cells that have defective mitochondria incapable of respiration will be able to grow by catabolizing which of the following carbon sources for energy? A) glucose B) cholesterol C) amino acids D) fatty acids

A) glucose

In plant cells, the middle lamella ________. A) glues adjacent cells together B) connects the cytoplasm of adjacent cells C) prevents dehydration of adjacent cells D) prevents excessive uptake of water by plant cells

A) glues adjacent cells together

Which metabolic pathway is common to both fermentation and cellular respiration of a glucose molecule? A) glycolysis B) the citric acid cycle C) the electron transport chain D) reduction of pyruvate to lactate

A) glycolysis

In Drosophila melanogaster, vestigial wings are determined by a recessive allele of a gene that is linked to a gene with a recessive allele that determines black body color. T. H. Morgan crossed black-bodied, normal-winged females and gray-bodied, vestigial-winged males. The F1 were all gray bodied, normal winged. The F1 females were crossed to homozygous recessive males to produce testcross progeny. Morgan calculated the map distance to be 17 map units. Which of the following information is correct about the testcross progeny? A) gray-bodied, normal-winged flies plus black-bodied, vestigial-winged flies = 17% of the total B) black-bodied, vestigial-winged flies = 17% of the total C) black-bodied, normal-winged flies plus gray-bodied, vestigial-winged flies = 17% of the total D) black-bodied, normal-winged flies = 17% of the total

A) gray-bodied, normal-winged flies plus black-bodied, vestigial-winged flies = 17% of the total

Amoebae move by crawling over a surface (cell crawling), which involves ________. A) growth of actin filaments to form bulges in the plasma membrane B) assembly of microtubule extensions that vesicles can follow in the direction of movement C) reinforcement of the pseudopod with intermediate filaments D) localized contractions driven by myosin and microtubules

A) growth of actin filaments to form bulges in the plasma membrane

At what stage of the cell cycle is the cyclin component of MPF destroyed? A) in late M B) in early G2 C) in late G1 D) at mid-S phase

A) in late M

Where are proteins produced other than on ribosomes free in the cytosol or ribosomes attached to the endoplasmic reticulum? A) in mitochondria B) in the Golgi apparatus C) in the nucleolus D) in lysosomes

A) in mitochondria

The best experimental design ________. A) includes a large sample size and a control, and alters only one condition between the controls and the experimental condition B) includes a large sample size for each condition C) alters only one condition between the controls and the experimental condition D) includes a control

A) includes a large sample size and a control, and alters only one condition between the controls and the experimental condition

Which of the following is the most plausible explanation for why an animal cell would be unable to reduce the Ca2+ concentration in its cytosol compared with the extracellular fluid? A) insufficient ATP levels in the cytosol B) inactivation of calcium-gated ion channels in the cell membrane C) insufficient levels of protein kinase in the cell D) excessive transport of calcium from the cytosol into the endoplasmic reticulum

A) insufficient ATP levels in the cytosol

Scientists have found that extracellular matrix components may induce specific gene expression in embryonic tissues such as the liver and testes. For this to happen, there must be direct communication between the extracellular matrix and the developing cells. Which kind of transmembrane protein would most likely be involved in this kind of induction? A) Integrins B) Collagens C) Actins D) Fibronectins

A) integrins

Lipid-soluble signaling molecules, such as aldosterone, cross the membranes of all cells but affect only target cells because A) intracellular receptors are present only in target cells. B) only target cells retain the appropriate DNA segments. C) only in target cells is aldosterone able to initiate the phosphorylation cascade that turns genes on. D) only target cells have enzymes that break down aldosterone.

A) intracellular receptors are present only in target cells.

Diffusion of ions across membranes through specific ion channels is driven by ________. A) ion electrochemical gradients B) ion concentration gradients only C) electrical gradients only D) active transport pumps

A) ion electrochemical gradients

According to the central dogma, what is the intermediate molecule involved in the flow of information in a cell that should go in the blank? DNA → ________ → Proteins A) mRNA B) rRNA C) tRNA D) mtDNA

A) mRNA

Which of the following macromolecules leave the nucleus of a eukaryotic cell through pores in the nuclear membrane? A) mRNA B) DNA C) phospholipids D) amino acids

A) mRNA

How are bacteria able to change their patterns of protein synthesis so quickly in response to environmental changes? A) mRNAs that are produced are short-lived and degraded within a few minutes of being synthesized. B) mRNA is stored for later use. C) Operons are activated in the presence of transcription factors. D) mRNA have long lifespans, allowing the bacteria to use them many times for translation.

A) mRNAs that are produced are short-lived and degraded within a few minutes of being synthesized.

The two homologs of a pair move toward opposite poles of dividing cell during ... A) meiosis I. B) fertilization. C) meiosis II. D) mitosis.

A) meiosis I.

At which stage of mitosis are chromosomes usually photographed in the preparation of a karyotype? A) metaphase B) anaphase C) prophase D) interphase

A) metaphase

The pedigree in the figure shows the transmission of a trait in a particular family. Based on this pattern of transmission, the trait is most likely ________. A) mitochondrial B) sex-linked dominant C) sex-linked recessive D) autosomal dominant

A) mitochondrial

Cytochalasin D is a drug that prevents actin polymerization. A cell treated with cytochalasin D will still be able to carry out which of the following processes? A) move vesicles within a cell B) contract muscle fibers C) extend pseudopodia D) divide in two

A) move vesicles within a cell

In E. coli, to repair a thymine dimer by nucleotide excision repair, in which order do the necessary enzymes act? A) nuclease, DNA polymerase, DNA ligase B) nuclease, DNA polymerase, RNA primase C) helicase, DNA polymerase, DNA ligase D) DNA ligase, nuclease, helicase

A) nuclease, DNA polymerase, DNA ligase

Refer to the information and figure below to answer the following questions. A certain (hypothetical) organism is diploid, has either blue or orange wings as the consequence of one of its genes on chromosome 12, and has either long or short antennae as the result of a second gene on chromosome 19, as shown in the figure. A certain female's number 12 chromosomes both have the blue gene and number 19 chromosomes both have the long gene. As cells in her ovaries undergo meiosis, which of the following combinations of genes and chromosomes will her eggs have? A) one chromosome 12 with one blue gene and one chromosome 19 with one long gene B) either two number 19 chromosomes with long genes or two with short genes C) either one blue or one orange gene in addition to either one long or one short gene D) either two number 12 chromosomes with blue genes or two with orange genes

A) one chromosome 12 with one blue gene and one chromosome 19 with one long gene

Refer to the information and figure below to answer the following questions. A certain (hypothetical) organism is diploid, has either blue or orange wings as the consequence of one of its genes on chromosome 12, and has either long or short antennae as the result of a second gene on chromosome 19, as shown in the figure. If a female of this species has one chromosome 12 with a blue gene and another chromosome 12 with an orange gene, and has both number 19 chromosomes with short genes, which of the following combinations of genes will her eggs have? A) one half blue short and one half orange short gene eggs B) three fourths blue short and one fourth orange short gene eggs C) only blue short gene eggs D) only orange short gene eggs

A) one half blue short and one half orange short gene eggs

Which of the following processes includes all the others? A) passive transport B) osmosis C) diffusion of a solute across a membrane D) transport of an ion down its electrochemical gradient

A) passive transport

In photosynthetic cells, synthesis of ATP by chemiosmosis occurs during ________. A) photosynthesis and respiration B) photosynthesis only C) photosynthesis, respiration, and fermentation D) respiration only

A) photosynthesis and respiration

Which of the following inheritance patterns describes the ability of a single allele to have multiple phenotypic effects? A) pleiotropy B) multiple alleles C) epistasis D) incomplete dominance

A) pleiotropy

A localized group of organisms that belong to the same species is called a ________. A) population B) family C) community D) ecosystem

A) population

What is the oxidizing agent in the following reaction? Pyruvate + NADH + H- Lactate + NAD- A) pyruvate B) NADH C) oxygen D) lactate

A) pyruvate

If individuals tend to mate within a subset of the population, there is ________. A) random mating B) no genetic drift C) no selection D) no gene flow

A) random mating

Which of the following frequently imposes a limit on cell size? A) ratios of surface area to volume B) the volume of the endomembrane system C) the number of mitochondria in the cytoplasm D) the absence of a nucleus

A) ratios of surface area to volume

Which of the following activities would be inhibited by a drug that specifically blocks the addition of phosphate groups to proteins? A) receptor tyrosine kinase activity B) adenylyl cyclase activity C) binding of G proteins to G protein-coupled receptors D) ligand-gated ion channel signaling pathways

A) receptor tyrosine kinase activity

In the initial step in carbon fixation, a molecule of CO2 is attached to RuBP to produce a six-carbon molecule, which is immediately split to produce two molecules of 3-phosphoglycerate. After phosphorylation and reduction produces glyceraldehyde 3-phosphate (G3P), what more needs to occur to complete the Calvin cycle? A) regeneration of RuBP B) regeneration of ATP from ADP C) addition of a pair of electrons from NADPH D) regeneration of NADP+

A) regeneration of RuBP

Which of the following processes occurs during the Calvin cycle? A) regeneration of the CO2 acceptor B) production of ATP C) release of oxygen D) reduction of NADPH

A) regeneration of the CO2 acceptor

Which of the following does not occur during the Calvin cycle? A) release of oxygen B) oxidation of NADPH C) carbon fixation D) regeneration of the CO2 acceptor

A) release of oxygen

What is the function of the enzyme topoisomerase in DNA replication? A) relieving strain in the DNA ahead of the replication fork caused by the untwisting of the double helix B) elongating new DNA at a replication fork by adding nucleotides to the existing chain C) building RNA primers using the parental DNA strand as a template D) reattaching the hydrogen bonds between the base pairs in the double helix

A) relieving strain in the DNA ahead of the replication fork caused by the untwisting of the double helix

Which of the following events occurs during interphase of the cell cycle? A) replication of the DNA B) condensation of the chromosomes C) spindle formation D) separation of the spindle poles

A) replication of the DNA

Which of the following molecules is a protein produced by a regulatory gene? A) repressor B) promoter C) operon D) inducer

A) repressor

Which of the following macromolecules enter the nucleus of a eukaryotic cell through pores in the nuclear membrane? A) ribosomal proteins B) phospholipids C) rRNA D) mRNA

A) ribosomal proteins

Which of the following is present in a prokaryotic cell? A) ribosome B) mitochondrion C) chloroplast D) nuclear envelope

A) ribosome

Which structure is the site of the synthesis of proteins that may be exported from the cell? A) rough ER B) plasmodesmata C) Golgi vesicles D) free cytoplasmic ribosomes

A) rough ER

The mitotic spindle plays a critical role in which of the following processes? A) separation of sister chromatids B) dissolving the nuclear membrane C) triggering the compaction and condensation of chromosomes D) splitting of the cell (cytokinesis) following mitosis

A) separation of sister chromatids

Which of the following amino acids are most frequently phosphorylated by protein kinases in the cytoplasm during signal transduction? A) serine and threonine B) glycine and glutamic acid C) tyrosines D) glycine and histidine

A) serine and threonine

The temperature at which an alligator's egg is incubated will determine the sex of the offspring. The dependent and the independent variables in this experiment are ________. A) sex of the baby alligator and temperature respectively B) number of offspring and temperature in the incubator respectively C) size of the incubator and size of the baby alligator respectively D) temperature and sex of the baby alligator respectively

A) sex of the baby alligator and temperature respectively

HIV's genome of RNA includes the code for reverse transcriptase (RT), an enzyme that acts early in infection to synthesize a DNA genome off of an RNA template. The HIV genome also codes for protease (PR), an enzyme that acts later in infection by cutting long viral polyproteins into smaller, functional proteins. Both RT and PR represent potential targets for antiretroviral drugs. Drugs called nucleoside analogs (NA) act against RT, whereas drugs called protease inhibitors (PI) act against PR. Which mechanism produces variation for evolution by shuffling existing alleles? A) sexual reproduction B) rapid reproduction C) mutation D) changes in chromosome numbers

A) sexual reproduction

A compound contains hydroxyl groups as its predominant functional group. Therefore, this compound ________. A) should dissolve in water B) will not form hydrogen bonds with water C) should dissolve in a nonpolar solvent D) lacks an asymmetric carbon and is probably a fat or lipid

A) should dissolve in water

Which of the following are least likely to diffuse through the phospholipid bilayer of a cell membrane? A) small ions B) carbon dioxide C) small hydrophobic molecules D) large hydrophobic molecules

A) small ions

Which one of the following conditions would allow gene frequencies to change by chance? A) small populations B) large population C) gene flow D) mutation

A) small populations

In his work with pneumonia-causing bacteria and mice, Griffith found that A) some substance from pathogenic cells was transferred to nonpathogenic cells, making them pathogenic. B) heat-killed pathogenic cells caused pneumonia. C) the polysaccharide coat of bacteria caused pneumonia. D) the protein coat from pathogenic cells was able to transform nonpathogenic cells.

A) some substance from pathogenic cells was transferred to nonpathogenic cells, making them pathogenic.

Martin Wikelski and L. Michael Romero (Body size, performance and fitness in Galápagos marine iguanas,Integrative and Comparative Biology 43 [2003]:376-86) measured the snout-to-vent (anus) length of Galápagos marine iguanas and observed the percent survival of different-sized animals, all of the same age. The graph shows the log snout-vent length (SVL, a measure of overall body size) plotted against the percent survival of these different size classes for males and females. Examine the figure. What type of selection for body size appears to be occurring in these marine iguanas? A) stabilizing selection B) disruptive selection C) directional selection D) You cannot determine the type of selection from the above information.

A) stabilizing selection

Which of the following processes would be most directly affected if a thylakoid membrane is punctured so that the interior of the thylakoid is no longer separated from the stroma? A) synthesis of ATP B) reduction of NADP+ C) flow of electrons from photosystem II to photosystem I D) splitting of water

A) synthesis of ATP

In the polymerization of DNA, a phosphodiester bond is formed between a phosphate group of the nucleotide being added and which of the following atoms or molecules of the last nucleotide in the polymer? A) the 3' OH B) a nitrogen from the nitrogen-containing base C) C6 D) the 5' phosphate

A) the 3' OH

If a particular operon encodes enzymes for making an essential amino acid and is regulated like the trp operon, then A) the amino acid acts as a corepressor. B) the amino acid turns on transcription of the operon. C) the amino acid inactivates the repressor. D) the repressor is active in the absence of the amino acid.

A) the amino acid acts as a corepressor.

What is the major structural difference between starch and glycogen? A) the amount of branching that occurs in the molecule B) whether glucose is in the α or β form C) the types of monosaccharide subunits in the molecules D) the type of glycosidic linkages in the molecule

A) the amount of branching that occurs in the molecule

The complexity and variety of organic molecules is due to ________. A) the chemical versatility of carbon atoms B) the diverse bonding patterns of nitrogen C) the variety of rare elements in organic molecules D) their interaction with water

A) the chemical versatility of carbon atoms

The decline of MPF activity at the end of mitosis is due to A) the degradation of cyclin. B) decreased synthesis of Cdk. C) the accumulation of cyclin. D)the destruction of the protein kinase Cdk.

A) the degradation of cyclin.

The partial negative charge in a molecule of water occurs because ________. A) the electrons shared between the oxygen and hydrogen atoms spend more time around the oxygen atom nucleus than around the hydrogen atom nucleus B) the oxygen atom donates an electron to each of the hydrogen atoms C) the oxygen atom has two pairs of electrons in its valence shell that are not neutralized by hydrogen atoms D) one of the hydrogen atoms donates an electron to the oxygen atom

A) the electrons shared between the oxygen and hydrogen atoms spend more time around the oxygen atom nucleus than around the hydrogen atom nucleus

Which of the following events takes place in the electron transport chain? A) the harnessing of energy from high-energy electrons derived from glycolysis and the citric acid cycle B) substrate-level phosphorylation C) the breakdown of glucose into six carbon dioxide molecules D) the breakdown of an acetyl group to carbon dioxide

A) the harnessing of energy from high-energy electrons derived from glycolysis and the citric acid cycle

In a plant, which of the following reactions produce molecular oxygen (O2)? A) the light reactions alone B) the light reactions and the Calvin cycle C) neither the light reactions nor the Calvin cycle D) the Calvin cycle alone

A) the light reactions alone

Taxol is an anticancer drug extracted from the Pacific yew tree. In animal cells, Taxol prevents microtubule depolymerization. Thus, Taxol stops mitosis by interfering with which of the following structures or processes? A) the mitotic spindle B) centriole duplication C) chromosome condensation D) cytokinesis

A) the mitotic spindle

In an RNA sample, ________. A) the number of purine may or may not equal the number of and pyrimidine B) the number of purine always equals the number of pyrimidine C) the number of thiamine may or may not equal the number of adenine D) the number of thiamine always equals the number of uracil

A) the number of purine may or may not equal the number of and pyrimidine

How does the primary transcript in the nucleus of a prokaryotic cell compare to the functional mRNA? A) the primary transcript is the same size as the mRNA B) the primary transcript and the mRNA both contain introns C) the primary transcript is larger than the mRNA D) the primary transcript is smaller than the mRNA

A) the primary transcript is the same size as the mRNA

Within a cell, the amount of protein made using a given mRNA molecule depends partly on A) the rate at which the mRNA is degraded. B) the degree of DNA methylation. C) the types of ribosomes present in the cytoplasm. D) the number of introns present in the mRNA.

A) the rate at which the mRNA is degraded.

In the structural organization of many eukaryotic genes, individual exons may be related to which of the following? A) the various domains of the polypeptide product B) the sequence of the intron that immediately precedes each exon C) the number of polypeptides making up the functional protein D) the number of start sites for transcription

A) the various domains of the polypeptide product

A defect in which of the following intercellular junctions would allow partially digested material to leak passively between the cells of the small intestine into the abdominal cavity? A) tight junctions B) plasmodesmata C) desmosomes D) gap junctions

A) tight junctions

In cats, black fur color is determined by an X-linked allele; the other allele at this locus determines orange color. The heterozygote is tortoiseshell. What kinds of offspring would you expect from the cross of a black female and an orange male? A) tortoiseshell females; black males B) tortoiseshell females; tortoiseshell males C) black females; orange males D) orange females; black males

A) tortoiseshell females; black males

Which process is most directly driven by light energy? A) transfer of energy from pigment molecule to pigment molecule B) ATP synthesis C) reduction of NADP+ molecules D) creation of a pH gradient by pumping protons across the thylakoid membrane

A) transfer of energy from pigment molecule to pigment molecule

One possible result of chromosomal breakage is for a fragment to join a nonhomologous chromosome. What is this type of chromosomal alteration called? A) translocation B) deletion C) duplication D) inversion

A) translocation

Sex determination in mammals is due to the SRY gene found on the Y chromosome. Which of the following situations could allow a person with an XX karyotype to develop a male phenotype? A) translocation of SRY to an X chromosome B) the presence of an extra autosomal chromosome C) the presence of one normal and one shortened (deleted) X D) the loss of the SRY gene from an autosome

A) translocation of SRY to an X chromosome

Cells require which of the following to form cilia or flagella? A) tubulin B) actin C) laminin D) intermediate filaments

A) tubulin

The molecule illustrated in the figure... A) will be liquid at room temperature B) is a saturated fatty acid C) is a carbohydrate D) stores genetic information

A) will be liquid at room temperature

A recessive allele on the X chromosome is responsible for red-green color blindness in humans. A woman with normal vision whose father is color blind marries a color-blind male. What is the probability that this couple's first son will be color blind? A) ½ B) ¼ C) 2/3 D) 3/4

A) ½

Maltose is a disaccharide that can easily be digested into glucose molecules. The glycosidic linkage between glucose molecules in maltose is ________. A) α 1—4 B) β 1—2 C) β 1—4 D) α 1—2

A) α 1—4

Theodor W. Engelmann illuminated a filament of algae with light that passed through a prism, thus exposing different segments of algae to different wavelengths of light. He added aerobic bacteria and then noted in which areas the bacteria congregated. He noted that the largest groups were found in the areas illuminated by the red and blue light. Which of the following statements describes a relationship that Engelmann's experiment helped to determine? A. the relationship between wavelength of light and the rate of photosynthesis B. the relationship between wavelength of light and the rate of aerobic respiration C. the relationship between wavelength of light and the amount of heat released D. the relationship between carbon dioxide concentration and the rate of photosynthesis

A. the relationship between wavelength of light and the rate of photosynthesis

Anopheles mosquitoes, which carry the malaria parasite, cannot live above elevations of 5,900 feet. In addition, oxygen availability decreases with higher altitude. Consider a hypothetical human population that is adapted to life on the slopes of Mt. Kilimanjaro in Tanzania, a country in equatorial Africa. Mt. Kilimanjaro's base is about 2,600 feet above sea level and its peak is 19,341 feet above sea level. If the incidence of the sickle-cell allele in the population is plotted against altitude (feet above sea level), which of the following distributions is most likely, assuming little migration of people up or down the mountain?

B)

Nitrogen (N) normally forms three covalent bonds with a valence of five. However, ammonium has four covalent bonds, each to a different hydrogen (H) atom (H has a valence of one). What do you predict to be the charge on ammonium? A) +2 B) +1 C) -1 D) -2

B) +1

In a Hardy-Weinberg population with two alleles, A and a, that are in equilibrium, the frequency of allele a is 0.2. What is the frequency of individuals that are heterozygous for this allele? A) 0.16 B) 0.32 C) 0.04 D) 0.020

B) 0.32

Three lab groups carried out an experiment to identify the concentration of sucrose in six solutions. Each unknown contained one of the following sucrose concentrations: 0.0 M, 0.2 M, 0.4 M, 0.6 M, 0.8 M, and 1.0 M. Cubes of sweet potato (1 cm3) were soaked for 24 hours in each solution and weighed to determine the change in mass. Each data entry represents the average of three sample replicates expressed as percent change in mass following a 24-hour soak in the unknown solutions. Unknown Percent change in mass Percent change in mass Percent change in mass Group 1 Group 2 Group 3 A 6.6 7.8 7.5 B 3.1 3.7 2.9 C -2.7 -3.5 -2.5 D 0.7 0.5 1.1 E -11.6 -12.3 -12.6 F -5.2 -6.2 -4.9 Based on the data provided, the intracellular molarity of dissolved solutes in sweet potato cells is approximately ________. A) 0.8 M B) 0.4 M C) 0.2 M D) 0.6 M

B) 0.4 M

What proportion of the population is probably heterozygous (Aa) for this trait? A) 0.25 B) 0.50 C) 0.75 D) 0.05

B) 0.50

Use the figure and the following description to answer the question. In a particular plant, leaf color is controlled by gene locusD. Plants with at least one allele D have dark green leaves, and plants with the homozygous recessive ddgenotype have light green leaves. A true-breeding, dark-leaved plant is crossed with a light-leaved one, and the F1offspring is allowed to self-pollinate. The predicted outcome of the F2 is diagrammed in the Punnett square shown in the figure, where 1, 2, 3, and 4 represent the genotypes corresponding to each box within the square. Which of the boxes marked 1-4 correspond to plants that will be true-breeding? A) 2 and 3 only B) 1 and 4 only C) 1 only D) 1, 2, 3, and 4

B) 1 and 4 only

When an ionic compound such as sodium chloride (NaCl) is placed in water, the component atoms of the NaCl crystal dissociate into individual sodium ions (Na+) and chloride ions (Cl-). In contrast, the atoms of covalently bonded molecules (e.g., glucose, sucrose, glycerol) do not generally dissociate when placed in aqueous solution. Which of the following solutions would be expected to contain the greatest number of solute particles (molecules or ions)? A) 1 liter of 1.0 M glucose B) 1 liter of 1.0 M NaCl C) 1 liter of 1.0 M NaCl and 1 liter of 1.0 M glucose will contain equal numbers of solute particles. D) 1 liter of 0.5 M NaCl

B) 1 liter of 1.0 M NaCl

In some jacana species, males take care of the eggs and young, and females compete among themselves for territories that contain one to several males. Female jacanas are significantly larger than males. Which of these statements would you predict to be true of this bird species? 1. Male jacana fitness is primarily limited by ability to take care of eggs and raise young. 2. Female jacana fitness is limited by the number of males in her territory with which a female mates. 3. Variation in reproductive success should be greater in male jacanas than in females. 4. Variation in reproductive success should be greater in female jacanas than in males. 5. Males and females have equal variation in reproductive success. A) 1 and 3 B) 1, 2, and 4 C) 2 and 4 D) 5

B) 1, 2, and 4

Skin color in a certain species of fish is inherited via a single gene with four different alleles. One fish of this type has alleles 1 and 3 (S1S3), and its mate has alleles 2 and 4 (S2S4). If each allele confers a unit of color darkness such that S1 has one unit, S2 has two units, and so on, then what proportion of their offspring would be expected to have five units of color? A) 1/4 B) 1/2 C) 0 D) 1/8

B) 1/2

Suppose two individuals with the genotype AaBbCc are mated. Assuming that the genes are not linked, what fraction of the offspring are expected to be homozygous recessive for the three traits? A) 1/8 B) 1/64 C) 1/16 D) 1/4

B) 1/64

Measurements show that the pH of a particular lake is 4.0. What is the hydroxide ion concentration of the lake? A) 10.0 M B) 10-10 M C) 10-7 M D) 10-4 M

B) 10-10 M

Substrate-level phosphorylation accounts for approximately what percentage of the ATP formed by the reactions of glycolysis? A) 2% B) 100% C) 38% D) 0%

B) 100%

Which of the following combinations of molecules illustrated could be linked to form a nucleotide? A) 1, 2, and 11 B) 11, 12, and 13 C) 5, 9, and 10 D) 3, 7, and 8

B) 11, 12, and 13

Use the figure to answer the question. The figure shows a simple metabolic pathway. According to Beadle and Tatum's hypothesis, how many genes are necessary for this pathway? A) 1 B) 2 C) 3 D) It cannot be determined from the pathway.

B) 2

What is the maximum number of covalent bonds that an oxygen atom with atomic number 8 can make with hydrogen? A) 4 B) 2 C) 6 D) 1

B) 2

In a diploid cell with 5 chromosome pairs (2n = 10), how many sister chromatids will be found in a nucleus at prophase of mitosis? A) 40 B) 20 C) 10 D) 5

B) 20

Given the following genotypes for two parents, AABBCc× AabbCc, assume that all traits exhibit simple dominance and independent assortment. What proportion of the progeny of this cross will be expected to phenotypically resemble the first parent with the genotype AABBCc? A) 3/8 B) 3/4 C) 1 D) 1/4

B) 3/4

How many grams of the compound in the figure are required to make 1 liter of a 0.5 M solution? (Note: The atomic masses, in daltons, are approximately 12 for carbon, 1 for hydrogen, and 16 for oxygen.) A) 90 B) 30 C) 60 D) 120

B) 30

A part of an mRNA molecule with the following sequence is being read by a ribosome: 5′-CCG-ACG-3′ (mRNA). The following charged transfer RNA molecules (with their anticodons shown in the 3′ to 5′ direction) are available. Two of them can correctly match the mRNA so that a dipeptide can form. Which of the following anticodons in the first tRNA to bind will complement this mRNA? A) 3′-UGC-5′ B) 3′-GGC-5′ C) 5′-UGC-3′ D) 5′-GGC-3′

B) 3′-GGC-5′

Refer to the figure. Which of the triplets below is a possible anticodon for a tRNA that transports proline to a ribosome? A) 3′-CCG-5′ B) 3′-GGC-5′ C) 3′-CCC-5′ D) 3′-GUG-5′

B) 3′-GGC-5′

Skin color in a certain species of fish is inherited by a single gene with four different alleles. How many different types of gametes would be possible in this organism? A) 8 B) 4 C) 16 D) 2

B) 4

Use the following information to answer the question. A plantlike organism on the planet Pandora can have three recessive genetic traits: bluish leaves, due to an allele (a) of geneA; a feathered stem, due to an allele (b) of gene B; and hollow roots due to an allele (c) of gene C. The three genes are linked and recombine. A geneticist performed a testcross with an organism that had been found to be heterozygous for the three recessive traits, and she was able to identify progeny of the following phenotypic distribution (+ = wild type): Which of the following are the phenotypes of the parents in this cross? A) 2 and 5 B) 4 and 8 C) 1 and 6 D) 3 and 7

B) 4 and 8

Rank the following one-base point mutations with respect to their likelihood of affecting the structure of the corresponding polypeptide (from most likely to least likely). 1. insertion mutation deep within an intron 2. substitution mutation at the third position of a codon in an exon 3. substitution mutation at the second position of a codon in an exon 4. deletion mutation within the first exon of the gene A) 3, 1, 4, 2 B) 4, 3, 2, 1 C) 1, 2, 3, 4 D) 2, 1, 4, 3

B) 4, 3, 2, 1

Homo sapiens have 23 pairs of chromosomes. This implies that ________. A) 23 double-stranded DNA molecules are present in each somatic cell B) 46 double-stranded DNA molecules are present in each somatic cell C) 23 single-stranded DNA molecules are present in each somatic cell D) several hundreds of genes are present on DNA but not on the chromosomes

B) 46 double-stranded DNA molecules are present in each somatic cell

You have a freshly prepared 0.1 M sucrose (molecular mass 342) solution, which means ________. A) 6.02 × 1022 of sucrose molecules are present in the solution B) 6.02 × 1022 of sucrose molecules or 34.2 g of sucrose is present in the solution C) 34.2 g of sucrose is present in the solution D) 6.02 × 1023 of sucrose molecules are present in the solution

B) 6.02 × 1022 of sucrose molecules or 34.2 g of sucrose is present in the solution

The atomic number of nitrogen is 7. Nitrogen-15 has a greater mass number than nitrogen-14 because the atomic nucleus of nitrogen-15 contains ________. A) 7 neutrons B) 8 neutrons C) 9 protons D) 15 protons

B) 8 neutrons

We can represent atoms by listing the number of protons, neutrons, and electrons—for example, 2p+, 2n0, 2e- for helium. Which of the following represents the 18O isotope of oxygen? A) 7p+, 2n0, 9e- B) 8p+, 10n0, 8e- C) 10p+, 8n0, 9e- D) 9p+, 9n0, 9e-

B) 8p+, 10n0, 8e-

Gene S controls the sharpness of spines in a type of cactus. Cacti with the dominant allele, S, have sharp spines, whereas homozygous recessive ss cacti have dull spines. At the same time, a second gene, N, determines whether or not cacti have spines. Homozygous recessivenn cacti have no spines at all. If cacti heterozygous for both traits, SsNn, were allowed to self-pollinate, the offspring would segregate into which of the following phenotype ratios? A) 1 sharp-spined:1 dull-spined:1 spineless B) 9 sharp-spined:3 dull-spined:4 spineless C) 3 sharp-spined:1 spineless D) 1 sharp-spined:2 dull-spined:1 spineless

B) 9 sharp-spined:3 dull-spined:4 spineless

Which of the following properties is associated with a cyclin-dependent kinase (Cdk)? A) A Cdk is an enzyme that catalyzes the attachment of kinetochores to microtubules. B) A Cdk is an enzyme that attaches phosphate groups to other proteins. C) The number of Cdk molecules increases during the S and G2phases and decrease during M. D) A Cdk is inactive, or "turned off," in the presence of a cyclin.

B) A Cdk is an enzyme that attaches phosphate groups to other proteins.

Which of the following statements about a G protein signaling pathway is true? A) A G protein-coupled receptor bound to GTP is in its active state. B) A G protein bound to GTP is in its active state. C) A G protein bound to GDP is in its active state. D) Hydrolysis of bound GTP by a G protein activates the G protein.

B) A G protein bound to GTP is in its active state.

Which of the following processes occurs in eukaryotic gene expression? A) RNA polymerase requires tRNA to elongate the molecule. B) A cap is added to the 5′ end of the mRNA. C) RNA polymerase binds to the terminator sequence. D) mRNA, tRNA, and rRNA are translated.

B) A cap is added to the 5′ end of the mRNA.

Which of the following statements correctly describes what happens to a chromosome after a nonreciprocal translocation occurs? A) A duplication of part of the chromosome occurs. B) A chromosome transfers a fragment but receives none in return. C) Nondisjunction of pairs of homologous occurs. D) A deletion of part of the chromosome occurs.

B) A chromosome transfers a fragment but receives none in return.

Generally, only female cats have the tortoiseshell phenotype for fur color. Which of the following statements explains this phenomenon? A) Multiple crossovers on the Y chromosome prevent orange pigment production. B) A male inherits only one allele of the X-linked gene controlling hair color. C) Only males can have Barr bodies. D) The Y chromosome has a gene blocking orange coloration.

B) A male inherits only one allele of the X-linked gene controlling hair color.

Which of the following statements supports the one gene-one enzyme hypothesis? A) Sickle-cell anemia results in normal hemoglobin. B) A mutation in a single gene can result in a defective protein. C) Alkaptonuria results when individuals lack multiple enzymes involved in the catalysis of homogentisic acid. D) Multiple antibody genes can code for different related proteins, depending on the splicing that takes place post-transcriptionally.

B) A mutation in a single gene can result in a defective protein.

Which of the following is NOT one of Charles Darwin's observations? A) Species generally are adapted to their environments. B) A population avoids competition by producing only as many offspring as can successfully reproduce on their own. C) Individuals in a population vary in their traits. D) Many of the traits in an individual are heritable.

B) A population avoids competition by producing only as many offspring as can successfully reproduce on their own.

Which of the following effects can occur because of the high surface tension of water? A) Sweat can evaporate from the skin, helping to keep people from overheating. B) A raft spider can walk across the surface of a small pond. C) Organisms can resist temperature changes, although they give off heat due to chemical reactions. D) Lakes cannot freeze solid in winter, despite low temperatures.

B) A raft spider can walk across the surface of a small pond.

If photosynthesizing green algae are provided with CO2 containing heavy oxygen (18O), which of the following molecules produced by the algae (refer to the accompanying figure) will fail to contain 18O in later biochemical analyses? A) ribulose bisphosphate (RuBP) B) ADP C) 3-phosphoglycerate D) glyceraldehyde 3-phosphate (G3P)

B) ADP

Which one of the following structures, if missing, would usually prevent translation from starting? A) poly-A tail B) AUG codon C) 5′ cap D) exon

B) AUG codon

Which statement about variation is true? A) All phenotypic variation is the result of genotypic variation. B) All new alleles are the result of nucleotide variability. C) All genetic variation produces phenotypic variation. D) All nucleotide variability results in neutral variation.

B) All new alleles are the result of nucleotide variability.

For a science fair project, two students decided to repeat the Hershey and Chase experiment, with modifications. They decided to radioactively label the nitrogen of the DNA, rather than the phosphate. They reasoned that each nucleotide has only one phosphate and two to five nitrogen atoms. Thus, labeling the nitrogen atoms would provide a stronger signal than labeling the phosphates. Why won't this experiment work? A) There is no radioactive isotope of nitrogen. B) Amino acids (and thus proteins) also have nitrogen atoms; thus, the radioactivity would not distinguish between DNA and proteins. C) Radioactive nitrogen has a half-life of 100,000 years, and the material would be too dangerous for too long. D) Although there are more nitrogens in a nucleotide, labeled phosphates actually have 16 extra neutrons; therefore, they are more radioactive.

B) Amino acids (and thus proteins) also have nitrogen atoms; thus, the radioactivity would not distinguish between DNA and proteins.

Which of the following statements is an important consequence of the first law of thermodynamics for a living organism? A) The energy content of an organism is constant. B) An organism ultimately must obtain all of the necessary energy for life from its environment. C) Organisms grow by converting energy into organic matter. D) The entropy of an organism decreases with time as the organism grows in complexity.

B) An organism ultimately must obtain all of the necessary energy for life from its environment.

Density-dependent inhibition is explained by which of the following processes? A) As cells become more numerous, the level of waste products increases, which slows metabolism and inhibits growth. B) As cells become more numerous, the cell surface proteins of one cell contact the adjoining cells, and they signal each other to stop dividing. C) As cells become more numerous, they begin to squeeze against each other, restricting their size. D) As cells become more numerous, the protein kinases they produce begin to compete with each other, such that the proteins produced by one cell essentially cancel those produced by its neighbor.

B) As cells become more numerous, the cell surface proteins of one cell contact the adjoining cells, and they signal each other to stop dividing.

For the following questions, match the labeled component of the cell membrane in the figure with its description. Which component in the accompanying figure plays a critical role in cell-cell recognition? A) A B) B C) C D) E

B) B

Refer to the accompanying figure and the molecules labeled A, B, C, D, and E. If the carbon atom of each of the incoming CO2 molecules is labeled with a radioactive isotope of carbon, which organic molecules will be radioactively labeled after one cycle? A) C only B) B, C, D, and E C) C, D, and E only D) B and C only

B) B, C, D, and E

Which domains of life are classified as prokaryotes? A) Bacteria and Protista B) Bacteria and Archaea C) Bacteria and Eukarya D) Archaea and Fungi

B) Bacteria and Archaea

Which of the following order is correct in terms of the hierarchy of the organization? A) Biosphere → Ecosystem → Population → Community → Organism B) Biosphere → Ecosystem →Community → Population → Organism C) Ecosystem → Biosphere → Population → Community → Organism D) Ecosystem → Community → Biosphere → Population → Organism

B) Biosphere → Ecosystem →Community → Population → Organism

The molecular formula for glucose is C6H12O6. What would be the molecular formula for a molecule made by linking three glucose molecules together by dehydration reactions? A) C18H36O18 B) C18H32O16 C) C6H10O5 D) C18H30O15

B) C18H32O16

What would be the expected effect on plants if the atmospheric CO2 concentration was doubled? A) All plants would experience increased rates of photosynthesis. B) C3 plants would have faster growth; C4 plants would be minimally affected. C) C4 plants would have faster growth; C3 plants would be minimally affected. D) C3 plants would have faster growth; C4 plants would have slower growth.

B) C3 plants would have faster growth rates; C4 plants would be minimally affected.

Which of the following molecules is polar? C3H7OH C2H5COOH A) C2H5COOH is not polar, but C3H7OH is polar. B) C3H7OH and C2H5COOH are both polar molecules. C) C2H5COOH is polar, but C3H7OH is not polar. D) Neither C2H5COOH or C3H7OH is polar.

B) C3H7OH and C2H5COOH are both polar molecules.

If a horticulturist breeding gardenias succeeds in having a single plant with a particularly desirable set of traits, which of the following would be her most probable and efficient route to establishing a line of such plants? A) Backtrack through her previous experiments to obtain another plant with the same traits. B) Clone the plant. C) Breed this plant with another plant with much weaker traits. D) Force the plant to self-pollinate to obtain an identical one.

B) Clone the plant.

You have just sequenced a new protein found in mice and observe that sulfur-containing cysteine residues occur at regular intervals. What is the significance of this finding? A) Cysteine causes bends, or angles, to occur in the tertiary structure of proteins. B) Cysteine residues are involved in disulfide bridges that help form tertiary structure. C) It will be important to include cysteine in the diet of the mice. D) Cysteine residues are required for the formation of α helices and β pleated sheets.

B) Cysteine residues are involved in disulfide bridges that help form tertiary structure.

In E. coli, which enzyme catalyzes the elongation of a new DNA strand in the 5' → 3' direction? A) helicase B) DNA polymerase III C) primase D) DNA ligase

B) DNA polymerase III

Use the data in the accompanying table to answer the question. Minutes Spent in Cell Cycle Phases Cell Type G1 S G2 M Beta 18 24 12 16 Delta 100 1 0 0 Gamma 18 48 14 20 The data in the table were obtained from a study of the length of time spent in each phase of the cell cycle by cells of three eukaryotic organisms designated beta, delta, and gamma. What is the best explanation for the data associated with delta cells? A) Delta cells contain no DNA. B) Delta cells are in the G0 phase. C) Delta cells divide in the G1 phase. D) Delta cells contain no RNA.

B) Delta cells are in the G0 phase.

A researcher discovered a species of moth that lays its eggs on oak trees. Eggs are laid at two distinct times of the year: early in spring when the oak trees are flowering and in midsummer when flowering is past. Caterpillars from eggs that hatch in spring feed on oak flowers and look like oak flowers. But caterpillars that hatch in summer feed on oak leaves and look like oak twigs. How does the same population of moths produce such different-looking caterpillars on the same trees? To answer this question, the biologist caught many female moths from the same population and collected their eggs. He put at least one egg from each female into eight identical cups. The eggs hatched, and at least two larvae from each female were maintained in one of the four temperature and light conditions listed below. In each of the four environments, one of the caterpillars was fed oak flowers, the other oak leaves. Thus, there were a total of eight treatment groups (4 environments × 2 diets). Which of the following is a testable hypothesis that would explain the differences in caterpillar appearance observed in this population? A) Winter causes ugly caterpillar and trees. B) Differences in diet trigger the development of different types of caterpillars. C) The longer day lengths of summer trigger the development of twig-like caterpillars. D) Differences in air pressure, due to differences in elevation, trigger the development of different types of caterpillars.

B) Differences in diet trigger the development of different types of caterpillars.

In every case, caterpillars that feed on oak flowers look like oak flowers. In every case, caterpillars that were raised on oak leaves looked like twigs. These results support which of the following hypotheses? A) Differences in air pressure, due to elevation, trigger the development of different types of caterpillars. B) Differences in diet trigger the development of different types of caterpillars. C) The longer day lengths of summer trigger the development of twig-like caterpillars. D) The differences are genetic. A female will either produce all flower-like caterpillars or all twig-like caterpillars.

B) Differences in diet trigger the development of different types of caterpillars.

Which of the following characteristics is most likely to be associated with an enzyme that catalyzes two different chemical reactions? A) The enzyme contains α-helices and β-pleated sheets. B) Either the enzyme has two distinct active sites or the substrates involved in the two reactions have very similar structures. C) The enzyme is subject to competitive inhibition and allosteric regulation. D) The enzyme is composed of at least two subunits.

B) Either the enzyme has two distinct active sites or the substrates involved in the two reactions have very similar structures.

Which of the following statements describes the first law of thermodynamics? A) The entropy of the universe is decreasing. B) Energy cannot be created or destroyed. C) Energy cannot be transferred or transformed. D) The entropy of the universe is constant.

B) Energy cannot be created or destroyed.

Which of the following investigators was (were) responsible for the discovery that in DNA from any species, the amount of adenine equals the amount of thymine, and the amount of guanine equals the amount of cytosine? A) Matthew Meselson and Franklin Stahl B) Erwin Chargaff C) Alfred Hershey and Martha Chase D) Oswald Avery, Maclyn McCarty, and Colin MacLeod

B) Erwin Chargaff

Which of the following is not true of RNA processing? A) RNA splicing can be catalyzed by spliceosomes. B) Exons are cut out before mRNA leaves the nucleus. C) Nucleotides may be added at both ends of the RNA. D) Ribozymes may function in RNA splicing.

B) Exons are cut out before mRNA leaves the nucleus.

Cell A has half as much DNA as cells B, C, and D in a mitotically active tissue. Cell A is most likely in ... A) G2. B) G1. C) metaphase. D) prophase.

B) G1.

Measurements of the amount of DNA per nucleus were taken on a large number of cells from a growing fungus. The measured DNA levels ranged from 3 to 6 picograms per nucleus. In which stage of the cell cycle did the nucleus contain 6 picograms of DNA? A) S B) G2 C) G0 D) G1

B) G2

Characteristics are transmitted from parents to offspring. ________ are the units of inheritance. A) RNA B) Genes C) Proteins D) DNA

B) Genes

Which of the following statements correctly describes Archibald Garrod's hypothesis for how "inborn errors of metabolism" such as alkaptonuria occur? A) Metabolic enzymes require vitamin cofactors, and affected individuals have significant nutritional deficiencies. B) Genes dictate the production of specific enzymes, and affected individuals have genetic defects that cause them to lack certain enzymes. C) Certain metabolic reactions are carried out by ribozymes, and affected individuals lack key splicing factors. D) Enzymes are made of DNA, and affected individuals lack DNA polymerase.

B) Genes dictate the production of specific enzymes, and affected individuals have genetic defects that cause them to lack certain enzymes.

Gray seed color in peas is dominant to white. Assume that Mendel conducted a series of experiments where plants with gray seeds were crossed among themselves, and the following progeny were produced: 302 gray and 98 white. What is the most probable genotype of each parent? A) GG × gg B) Gg × Gg C) GG × Gg D) gg × Gg

B) Gg × Gg

Which of the following sequences correctly represents the flow of electrons during photosynthesis? A) NADPH → O2 → C O2 B) H2O → NADPH → Calvin cycle C) NADPH → chlorophyll → Calvin cycle D) NADPH → electron transport chain → O2

B) H2O → NADPH → Calvin cycle

Which of the following statements correctly describes the structure of chromatin? A) Euchromatin is not transcribed, whereas heterochromatin is transcribed. B) Heterochromatin is highly condensed, whereas euchromatin is less compact. C) Heterochromatin is composed of DNA, whereas euchromatin is made of DNA and RNA. D) Both heterochromatin and euchromatin are found in the cytoplasm.

B) Heterochromatin is highly condensed, whereas euchromatin is less compact.

Which of the following statements best distinguishes hypotheses from theories in science? A) Theories are proved true; hypotheses are often contradicted by experimental results. B) Hypotheses usually are relatively narrow in scope; theories have broad explanatory power. C) Theories are hypotheses that have been proved. D) Hypotheses are guesses; theories are correct answers.

B) Hypotheses usually are relatively narrow in scope; theories have broad explanatory power.

Which of the following are qualities of any good scientific hypothesis? I. It is testable. II. It is falsifiable. III. It produces quantitative data. IV. It produces results that can be replicated. A) III only B) I and II C) I only D) III and IV

B) I and II

In the figure, G1 is represented by which numbered part(s) of the cycle? A) IV B) I and V C) II D) III

B) I and V

Which of the following processes correctly describes alternative RNA splicing? A) It is a mechanism for increasing the rate of translation. B) It can allow the production of proteins of different sizes and functions from a single mRNA. C) It increases the rate of transcription. D) It can allow the production of similar proteins from different RNAs.

B) It can allow the production of proteins of different sizes and functions from a single mRNA.

Which of the following statements about the electron transport chain is true? A) It is driven by ATP hydrolysis. B) It consists of a series of redox reactions C) It occurs in the cytoplasm of both prokaryotic and eukaryotic cells. D) It includes a series of hydrolysis reactions associated with mitochondrial membranes.

B) It consists of a series of redox reactions

Which of the following statements correctly describes the function of a signal peptide? A) It terminates translation of messenger RNA. B) It helps target a protein to the ER. C) It signals the initiation of transcription. D) It directs an mRNA molecule into the cisternal space of the ER.

B) It helps target a protein to the ER.

An organism is discovered that thrives in both the presence and absence of oxygen. Interestingly, as oxygen is removed from the organism's environment, the rate of sugar consumption increases while the growth rate decreases. What do these observations suggest about the likely identity of this organism? A) It is an obligate anaerobic organism. B) It is a facultative anaerobic organism. C) It is an unremarkable eukaryotic organism. D) It is a photosynthetic organism.

B) It is a facultative anaerobic organism.

When chemical, transport, or mechanical work is done by an organism, what happens to the heat generated? A) It is used to power yet more cellular work. B) It is lost to the environment. C) It is used to generate ADP from nucleotide precursors. D) It is captured to store energy as more ATP.

B) It is lost to the environment.

When ATP releases some energy, it also releases inorganic phosphate. What happens to the inorganic phosphate in the cell? A) It is secreted as waste. B) It may be used to form a phosphorylated intermediate. C) It enters the nucleus to be incorporated in a nucleotide. D) It is used only to regenerate more ATP.

B) It may be used to form a phosphorylated intermediate.

P680+ is said to be the strongest biological oxidizing agent. Given its function, why is this necessary? A) It is the receptor for the most excited electron in either photosystem of photosynthesis. B) It obtains electrons from the oxygen atom in a water molecule, so it must have a stronger attraction for electrons than oxygen has. C) It is the molecule that transfers electrons to plastoquinone (Pq) of the electron transfer system. D) It transfers its electrons to reduce NADP+ to NADPH.

B) It obtains electrons from the oxygen atom in a water molecule, so it must have a stronger attraction for electrons than oxygen has.

Which of the following statements is true concerning the accompanying figure? A) It represents a C3 photosynthetic system. B) It represents a C4 photosynthetic system. C) It represents a CAM photosynthetic system. D) It represents an adaptation that maximizes photorespiration.

B) It represents a C4 photosynthetic system.

Which of the following is true of natural selection? A) It requires genetic variation. B) It requires genetic variation, results in descent with modification, and involves differential reproductive success. C) It results in descent with modification. D) It involves differential reproductive success.

B) It requires genetic variation, results in descent with modification, and involves differential reproductive success.

Which of the following properties is associated with a protein that will be secreted from a eukaryotic cell? A) Its signal sequence must target it to the plasma membrane, where it causes exocytosis. B) Its signal sequence must target it to the ER, after which it goes to the Golgi. C) Its signal sequence must be cleaved off before the polypeptide can enter the ER. D) It must be translated by a ribosome that remains free within the cytosol.

B) Its signal sequence must target it to the ER, after which it goes to the Golgi.

Which of the following statements is true for lipids? A) Lipids are true polymers. B) Lipids mix poorly with water. C) Lipids are true polymers and mix poorly with water. D) Waxes and pigments are not lipids.

B) Lipids mix poorly with water.

Which of the following statements describes a major difference between meiosis II and mitosis in a diploid animal? A) Crossing over of chromosomes takes place in meiosis II. B) Meiosis II occurs in a haploid cell, while mitosis occurs in diploid cells. C) Sister chromatids separate in mitosis, and homologous chromosomes separate in meiosis II. D) Homologous chromosomes align on the metaphase plate in meiosis II.

B) Meiosis II occurs in a haploid cell, while mitosis occurs in diploid cells.

Which of the following statements correctly describes the meaning of the chromosome theory of inheritance as expressed in the early 20th century? A) Natural selection acts on certain chromosome combinations rather than on genes. B) Mendelian genes are at specific loci on the chromosome and, in turn, segregate during meiosis. C) No more than a single pair of chromosomes can be found in a healthy normal cell. D) Individuals inherit particular chromosomes attached to genes.

B) Mendelian genes are at specific loci on the chromosome and, in turn, segregate during meiosis.

Once researchers identified DNA as the molecule responsible for transmitting heritable traits, they asked how information was transferred from the DNA in the nucleus to the site of protein synthesis in the cytoplasm. Which of the following statements correctly describes the mechanism of information transfer in eukaryotes that accomplishes this task? A) Transfer RNA takes information from DNA directly to a ribosome, where protein synthesis takes place. B) Messenger RNA is transcribed from a single gene and transfers information from the DNA in the nucleus to the cytoplasm, where protein synthesis takes place. C) Histone proteins in the chromosomes transfer information from the nucleus to the ribosome, where protein synthesis takes place. D) DNA from a single gene is replicated and transferred to the cytoplasm, where it serves as a template for protein synthesis.

B) Messenger RNA is transcribed from a single gene and transfers information from the DNA in the nucleus to the cytoplasm, where protein synthesis takes place.

The restriction enzymes of bacteria protect the bacteria from successful attack by bacteriophages, whose genomes can be degraded by the restriction enzymes. The bacterial genomes are not vulnerable to these restriction enzymes because bacterial DNA is methylated. This situation selects for bacteriophages whose genomes are also methylated. As new strains of resistant bacteriophages become more prevalent, this in turn selects for bacteria whose genomes are not methylated and whose restriction enzymes instead degrade methylated DNA. Over the course of evolutionary time, what should occur? A) Methylated DNA should become fixed in the gene pools of bacteriophages. B) Methylated and nonmethylated strains should be maintained among both bacteria and bacteriophages, with ratios that vary over time. C) Nonmethylated DNA should become fixed in the gene pools of bacteriophages. D) Methylated DNA should become fixed in the gene pools of bacterial species.

B) Methylated and nonmethylated strains should be maintained among both bacteria and bacteriophages, with ratios that vary over time.

Genomic imprinting is generally due to the addition of methyl (-CH3) groups to C nucleotides and chemical histone changes to silence a given gene. If this depends on the sex of the parent who transmits the gene, which of the following statements must be true? A) Genes required for early development stages must not be imprinted. B) Methylation must be reversible in ovarian and testicular cells. C) Methylation of this kind must occur more in males than in females. D) The imprints are transmitted only to gamete-producing cells.

B) Methylation must be reversible in ovarian and testicular cells.

Molybdenum has an atomic number of 42. Several common isotopes exist, with mass numbers from 92-100. Which of the following can be true? A) Molybdenum atoms can have between 50 and 58 electrons. B) Molybdenum atoms can have between 50 and 58 neutrons. C) Isotopes of molybdenum have different numbers of electrons. D) Molybdenum atoms can have between 50 and 58 protons.

B) Molybdenum atoms can have between 50 and 58 neutrons.

Rate of an enzyme-catalyzed reaction as a function of varying reactant concentration, with the concentration of enzyme constant In the figure, why does the reaction rate plateau at higher reactant concentrations? A) Feedback inhibition by product occurs at high reactant concentrations. B) Most enzyme molecules are occupied by substrate at high reactant concentrations. C) The reaction nears equilibrium at high reactant concentrations. D) The activation energy for the reaction increases with reactant concentration.

B) Most enzyme molecules are occupied by substrate at high reactant concentrations.

Which of the following statements about NAD+ is true? A) In the absence of NAD+, glycolysis can still function. B) NAD+ is reduced to NADH during glycolysis, pyruvate oxidation, and the citric acid cycle. C) NAD+ has more chemical energy than NADH. D) NAD+ can donate electrons for use in oxidative phosphorylation.

B) NAD+ is reduced to NADH during glycolysis, pyruvate oxidation, and the citric acid cycle.

Which molecule is the final electron acceptor for electrons from photosystem I? A) carbon dioxide B) NADP+ C) chlorophyll in photosystem II D) oxygen

B) NADP+

Researchers studying a small milkweed population note that some plants produce a toxin and other plants do not. They identify the gene responsible for toxin production. The dominant allele (T) codes for an enzyme that makes the toxin, and the recessive allele (t) codes for a nonfunctional enzyme that cannot produce the toxin. Heterozygotes produce an intermediate amount of toxin. The genotypes of all individuals in the population are determined (see chart) and used to determine the actual allele frequencies in the population. Is this population in Hardy-Weinberg equilibrium? A) Yes. B) No; there are more homozygotes than expected. C) No; there are more heterozygotes than expected. D) More information is needed to answer this question.

B) No; there are more homozygotes than expected.

If an enzyme is added to a solution where its substrate and product are in equilibrium, what will occur? A) Additional substrate will be formed. B) Nothing; the reaction will stay at equilibrium. C) The free energy of the system will change. D) The reaction will change from endergonic to exergonic.

B) Nothing; the reaction will stay at equilibrium.

If cell X enters meiosis, and nondisjunction of one chromosome occurs in one of its daughter cells during meiosis II, how will this affect the gametes at the completion of meiosis? A) Half of the gametes descended from cell X will be n + 1, and half will be n - 1. B) One-quarter of the gametes descended from cell X will be n + 1, one-quarter will be n - 1, and half will be n. C) Two of the four gametes descended from cell X will be haploid, and two will be diploid. D) All the gametes descended from cell X will be diploid.

B) One-quarter of the gametes descended from cell X will be n + 1, one-quarter will be n - 1, and half will be n.

How might a single base substitution in the sequence of a gene affect the amino acid sequence of a protein encoded by the gene? A) The amino acid sequence would be substantially altered, because the reading frame would change with a single base substitution. B) Only a single amino acid could change, because the reading frame would be unaffected. C) All amino acids following the substitution would be affected, because the reading frame would be shifted. D) It is not possible for a single base substitution to affect protein structure, because each codon is three bases long.

B) Only a single amino acid could change, because the reading frame would be unaffected.

Three lab groups carried out an experiment to identify the concentration of sucrose in six solutions. Each unknown contained one of the following sucrose concentrations: 0.0 M, 0.2 M, 0.4 M, 0.6 M, 0.8 M, and -1.0 M. Cubes of sweet potato (1 cm3) were soaked for 24 hours in each solution and weighed to determine the change in mass. Each data entry represents the average of three sample replicates expressed as percent change in mass following a 24-hour soak in the unknown solutions. From the data given, which statement most accurately describes what is occurring in response to a particular unknown solution? Unknown Percent change in mass Percent change in mass Percent change in mass Group 1 Group 2 Group 3 A 6.6 7.8 7.5 B 3.1 3.7 2.9 C -2.7 -3.5 -2.5 D 0.7 0.5 1.1 E -11.6 -12.3 -12.6 F -5.2 -6.2 -4.9 A) Unknown solution C represents a sucrose concentration slightly higher than the molarity of sweet potato cells, thus water is transported out of the cells. B) Osmosis of water molecules from unknown solution B likely caused the increase in mass observed. C) Unknown solution E contains the highest concentration of sucrose, and the change in mass is due to the active transport of sucrose out of the cell. D) Passive transport of sucrose out of the potato cells explains the change in mass observed for unknown solution F.

B) Osmosis of water molecules from unknown solution B likely caused the increase in mass observed.

Which of the following molecules is released by platelets in the vicinity of an injury? A) Cdk B) PDGF C) cyclin D) MPF

B) PDGF

How is plant cell cytokinesis different from animal cell cytokinesis? A) Plant cells divide after metaphase but before anaphase; animal cells divide after anaphase. B) Plant cells deposit vesicles containing cell wall building blocks on the metaphase plate; animal cells form a cleavage furrow. C) The structural proteins of plant cells separate the two cells; in animal cells, a cell membrane separates the two daughter cells. D) The contractile filaments found in plant cells are structures composed of carbohydrates; the cleavage furrow in animal cells is composed of contractile proteins.

B) Plant cells deposit vesicles containing cell wall building blocks on the metaphase plate; animal cells form a cleavage furrow.

Proto-oncogenes can change into oncogenes that cause cancer. Which of the following best explains the presence of these potential time bombs in eukaryotic cells? A) Proto-oncogenes are genetic "junk." B) Proto-oncogenes normally help regulate cell division. C) Proto-oncogenes are mutant versions of normal genes. D) Proto-oncogenes first arose from viral infections.

B) Proto-oncogenes normally help regulate cell division.

In eukaryotes, there are several different types of RNA polymerase. Which type is involved in transcription of mRNA for a globin protein? A) RNA polymerase III B) RNA polymerase II C) RNA polymerase I D) primase

B) RNA polymerase II

After the first replication was observed in their experiments testing the nature of DNA replication, Meselson and Stahl could be confident of which of the following conclusions? A) Replication is semi-conservative. B) Replication is not conservative. C) Replication is neither dispersive nor conservative. D) Replication is not dispersive.

B) Replication is not conservative.

You have a cube of modeling clay in your hands. Which of the following changes to the shape of this cube of clay will decrease its surface area relative to its volume? A) Stretch the cube into a long, shoebox shape. B) Round the clay up into a sphere. C) Flatten the cube into a pancake shape. D) Pinch the edges of the cube into small folds.

B) Round the clay up into a sphere.

Increased atmospheric CO2 concentrations might have what effect on seawater? A) Seawater will become more alkaline, and carbonate concentrations will decrease. B) Seawater will become more acidic, and carbonate concentrations will decrease. C) Seawater will become more acidic, and carbonate concentrations will increase. D) There will be no change in the pH of seawater, because carbonate will turn to bicarbonate.

B) Seawater will become more acidic, and carbonate concentrations will decrease.

How does natural selection apply to sexual reproduction as opposed to asexual reproduction? A) Sexual reproduction results in the greatest number of new mutations. B) Sexual reproduction results in many new gene combinations, some of which will lead to differential reproduction. C) Sexual reproduction allows the greatest number of offspring to be produced. D) Sexual reproduction utilizes far less energy than asexual reproduction.

B) Sexual reproduction results in many new gene combinations, some of which will lead to differential reproduction.

Which of the following provides the best evidence that cell-signaling pathways evolved early in the history of life? A) Most signals in all types of cells are received by cell surface receptors. B) Signal transduction molecules identified in distantly related organisms are similar. C) Bacteria and yeast cells signal each other in a process called quorum sensing. D) Cell-signaling pathways are seen in "primitive" cells such as bacteria and yeast.

B) Signal transduction molecules identified in distantly related organisms are similar.

The receptors for steroid hormones are located inside the cell instead of the membrane surface like most other signal receptors. How do steroids gain access to their receptors? A) Both steroid hormones and their receptors are produced by the same cells. B) Steroid hormones are lipid soluble, so they can readily diffuse through the lipid bilayer of the cell membrane. C) Steroid hormones first bind to a steroid activator and this complex is transported across the cell membrane by a steroid transport protein. D) Steroid hormone receptors undergo conformational changes that relocate them on the membrane surface.

B) Steroid hormones are lipid soluble, so they can readily diffuse through the lipid bilayer of the cell membrane.

In a metabolic pathway, succinate dehydrogenase catalyzes the conversion of succinate to fumarate. The reaction is inhibited by malonic acid, a substance that resembles succinate but cannot be acted upon by succinate dehydrogenase. Increasing the amount of succinate molecules to those of malonic acid reduces the inhibitory effect of malonic acid. Which of the following statements correctly describes the role played by molecules described in the reaction? A) Succinate dehydrogenase is the enzyme, and malonic acid is the substrate in the reaction. B) Succinate is the substrate, and fumarate is the product in the reaction. C) Fumarate is the product, and malonic acid is a noncompetitive inhibitor in the reaction. D) Succinate dehydrogenase is the enzyme, and fumarate is the substrate in the reaction.

B) Succinate is the substrate, and fumarate is the product in the reaction.

What would you expect to happen if MPF (maturation-promoting factor) is introduced into immature frog oocytes that are arrested in G2? A) The cells would begin DNA synthesis. B) The cells would enter mitosis. C) The cells would enter G0. D) The cells would remain arrested in G2.

B) The cells would enter mitosis.

Once a cell enters mitosis, the molecules that activate division must be turned off. What happens to MPF during mitosis? A) The Cdk component of MPF is degraded. B) The cyclin component of MPF is degraded. C) It is phosphorylated by a Cdk, which inactivates it. D) It is completely degraded.

B) The cyclin component of MPF is degraded.

Map units on a linkage map cannot be relied upon to calculate physical distances on a chromosome for which of the following reasons? A) Physical distances between genes change during the course of the cell cycle. B) The frequency of crossing over varies along the length of the chromosome. C) The gene order on the chromosomes is slightly different in every individual. D) The relationship between recombination frequency and map units is different in every individual.

B) The frequency of crossing over varies along the length of the chromosome.

Mendel crossed yellow-seeded and green-seeded pea plants and then allowed the offspring to self-pollinate to produce an F2 generation. The results were as follows: 6,022 yellow and 2,001 green (8,023 total). Which of the following statements correctly describes the relationship of the allele for green seeds to the allele for yellow seeds? A) The two alleles are codominant. B) The green allele is recessive to the yellow allele. C) The two alleles exhibit incomplete dominance. D) The green allele is dominant to the yellow allele.

B) The green allele is recessive to the yellow allele.

According to the lac operon model proposed by Jacob and Monod, what is predicted to occur if the operator is removed from the operon? A) Galactosidase permease would be produced, but would be incapable of transporting lactose. B) The lac operon would be transcribed continuously. C) Only lacZ would be transcribed. D) Only lacY would be transcribed.

B) The lac operon would be transcribed continuously.

Why are hydrocarbons insoluble in water? A) The majority of their bonds are polar covalent carbon-to-hydrogen linkages. B) The majority of their bonds are nonpolar covalent carbon-to-hydrogen linkages. C) They exhibit considerable molecular complexity and diversity. D) They are less dense than water.

B) The majority of their bonds are nonpolar covalent carbon-to-hydrogen linkages.

Which of the following statements correctly describes the difference between ATP and the nucleotides used during DNA synthesis? A) ATP is found only in human cells; the nucleotides are found in all animal and plant cells. B) The nucleotides have the sugar deoxyribose; ATP has the sugar ribose. C) ATP contains three high-energy bonds; the nucleotides have two. D) The nucleotides have two phosphate groups; ATP has three phosphate groups.

B) The nucleotides have the sugar deoxyribose; ATP has the sugar ribose.

Biological systems use free energy based on empirical data that all organisms require a constant energy input. The first law of thermodynamics states that energy can be neither created nor destroyed. For living organisms, which of the following statements is an important consequence of this first law? A) The entropy of an organism decreases with time as the organism grows in complexity. B) The organism must ultimately obtain all necessary energy for life from its environment. C) The energy content of an organism is constant except for when its cells are dividing. D) Organisms are unable to transform energy from the different states in which it can exist.

B) The organism must ultimately obtain all necessary energy for life from its environment.

Which of the following events are associated with chemiosmosis in chloroplasts? A) The pH of the stroma increases and ATP is synthesized. B) The pH of the thylakoid space increases and ATP is synthesized. C) The pH of the cytoplasm outside the chloroplast decreases and ATP is synthesized. D) The pH of the stroma decreases and ATP is hydrolyzed.

B) The pH of the thylakoid space increases and ATP is synthesized.

Use the following information to answer the question. A plantlike organism on the planet Pandora can have three recessive genetic traits: bluish leaves, due to an allele (a) of geneA; a feathered stem, due to an allele (b) of gene B; and hollow roots due to an allele (c) of gene C. The three genes are linked and recombine. A geneticist performed a testcross with an organism that had been found to be heterozygous for the three recessive traits, and she was able to identify progeny of the following phenotypic distribution (+ = wild type): What is the greatest benefit of having used a testcross for this experiment? A) The homozygous recessive parents are obvious to the naked eye. B) The phenotypes of the progeny reveal the allelic content of the gamete from the heterozygous parent. C) The homozygous parents are the only ones whose crossovers make a difference. D) All of the progeny will be heterozygous.

B) The phenotypes of the progeny reveal the allelic content of the gamete from the heterozygous parent.

What does it mean to say that a signal is transduced? A) The signal enters the cell directly and binds to a receptor inside. B) The physical form of the signal changes as it passes from the cell membrane to the ultimate intracellular target. C) The signal is amplified, such than even a single molecule evokes a large response. D) The signal triggers a sequence of phosphorylation events inside the cell.

B) The physical form of the signal changes as it passes from the cell membrane to the ultimate intracellular target.

Researchers found a strain of E. coli bacteria that had mutation rates one hundred times higher than normal. Which of the following statements correctly describes the most likely cause of these results? A) The DNA polymerase was unable to add bases to the 3′ end of the growing nucleic acid chain. B) The proofreading mechanism of DNA polymerase was not working properly. C) There were one or more base pair mismatches in the RNA primer. D) The single-strand binding proteins were malfunctioning during DNA replication.

B) The proofreading mechanism of DNA polymerase was not working properly.

Imagine that there are 25 different species of protists living in a tide pool. Some of these species reproduce both sexually and asexually, and some of them can reproduce only asexually. The pool gradually becomes infested with disease-causing viruses and bacteria. Which species are more likely to thrive in the changing environment? A) Sexually and asexually reproducing species are equally likely to thrive. B) The sexually reproducing species is likely to thrive. C) Neither species will be able to thrive. D) The asexually reproducing species is likely to thrive.

B) The sexually reproducing species is likely to thrive.

Which of the following statements describes a likely effect of a drug designed that inhibits the cellular response to testosterone? A) The cytoplasmic levels of cAMP would decrease. B) The transcription of certain genes would decrease. C) The cytosolic calcium concentration would increase. D) The activity of G proteins would decrease.

B) The transcription of certain genes would decrease.

What does a recombination frequency of 50% indicate? A) Abnormal meiosis has occurred. B) The two genes are likely to be located on different chromosomes. C) All of the offspring have combinations of traits that match one of the two parents. D) The genes are located on sex chromosomes.

B) The two genes are likely to be located on different chromosomes.

A large population of laboratory animals has been allowed to breed randomly for a number of generations. After several generations, 25% of the animals display a recessive trait (aa), the same percentage as at the beginning of the breeding program. The rest of the animals show the dominant phenotype, with heterozygotes indistinguishable from the homozygous dominants. What is the most reasonable conclusion that can be drawn from the fact that the frequency of the recessive trait (aa) has not changed over time? A) There has been sexual selection favoring allele a. B) The two phenotypes are about equally adaptive under laboratory conditions. C) The genotype AA is lethal. D) There has been a high rate of mutation of allele A to allele a.

B) The two phenotypes are about equally adaptive under laboratory conditions.

What is the most reasonable conclusion that can be drawn from the fact that the frequency of the recessive trait (aa) has not changed over time? A) There has been sexual selection favoring allele a. B) The two phenotypes are about equally adaptive under laboratory conditions. C) The genotype AA is lethal. D) There has been a high rate of mutation of allele A to allele a.

B) The two phenotypes are about equally adaptive under laboratory conditions.

What is the relationship between the wavelength of light and the quantity of energy per photon? A) They have a direct, linear relationship. B) They are inversely related. C) They are logarithmically related. D) They are separate phenomena.

B) They are inversely related.

Why are carbohydrates and fats frequently considered high-energy foods? A) They contain many oxygen atoms. B) They contain many electrons associated with hydrogen atoms. C) They contain no nitrogen atoms. D) They are strong oxidizing molecules.

B) They contain many electrons associated with hydrogen atoms.

What is a primary function of transcription factors? A) formation of mating complexes B) they control gene expression. C) secretion of substances that inhibit foreign bacteria D) digestion of unwanted parasite populations

B) They control gene expression.

Carotenoids are often found in foods that are considered to have antioxidant properties in human nutrition. What related function do they have in plants? A) They serve as accessory pigments to increase light absorption. B) They protect against oxidative damage from excessive light energy. C) They shield the sensitive chromosomes of the plant from harmful ultraviolet radiation. D) They reflect orange light and enhance red light absorption by chlorophyll.

B) They protect against oxidative damage from excessive light energy.

Yeast cells of mating type a are genetically engineered to produce only mating factor α instead of the normal mating factor a. The gene for the mating factor receptor was unaltered. How will these engineered cells behave in terms of mating? A) They will only mate with normal mating type α cells. B) They will mate with each other or with normal mating type a cells, but not with normal mating type α cells. C) They will only mate each other and not with normal mating type a or α cells. D) They will only mate with normal mating type a cells.

B) They will mate with each other or with normal mating type a cells, but not with normal mating type α cells.

The enzyme phosphofructokinase (PFK) catalyzes a key step in glycolysis. About 10% of Springer spaniels suffer from canine PFK deficiency. Given its critical role in glycolysis, which of the following conditions would be a likely consequence for dogs afflicted with this disorder? A) They would carry out elevated levels of oxidative phosphorylation. B) They would be lethargic and readily tire from exercise. C) They would die as embryos. D) They would have elevated blood-glucose levels, which may result in a high incidence of diabetes.

B) They would be lethargic and readily tire from exercise.

Nucleotides can be radiolabeled before they are incorporated into newly forming DNA and, therefore, can be assayed to track their incorporation. In a set of experiments, a student—faculty research team used labeled T nucleotides and introduced these into the culture of dividing human cells at specific times. Which of the following questions might be answered by using the method described? A) How many cells are produced by the culture per hour? B) What is the length of the S phase of the cell cycle? C) How many picograms of DNA are made per cell cycle? D) When do spindle fibers attach to chromosomes?

B) What is the length of the S phase of the cell cycle?

Hershey and Chase used a DNA-based virus for their work. How might the results have been different if they had used an RNA virus? A) With an RNA virus, the protein shell would have been radioactive in both samples. B) With an RNA virus, radioactive RNA would have been in the final pellet. C) With an RNA virus, radioactive protein would have been in the final pellet. D) With an RNA virus, neither sample would have had a radioactive pellet.

B) With an RNA virus, radioactive RNA would have been in the final pellet.

You are suffering from Streptococcus throat infection. You share the following with the bacteria that is responsible for your condition. A) You have nothing in common. B) You both are made up of cells. C) You both belong to the same domain. D) You both have genetic material in your nucleus.

B) You both are made up of cells.

Which of the following phrases correctly defines what one map unit is? A) 1 nanometer of distance between two genes B) a 1% frequency of recombination between two genes C) the recombination frequency between two genes assorting independently D) the physical distance between two linked genes

B) a 1% frequency of recombination between two genes

Zinc, an essential trace element for most organisms, is present in the active site of the enzyme carboxypeptidase. The zinc most likely functions as ________. A) a noncompetitive inhibitor of the enzyme B) a cofactor necessary for enzyme activity C) an allosteric activator of the enzyme D) a coenzyme derived from a vitamin

B) a cofactor necessary for enzyme activity

Which of the following types of mutation, resulting in an error in the mRNA just after the AUG start of translation, is likely to have the most serious effect on the polypeptide product? A) a substitution of the first nucleotide of a GGG codon B) a deletion of two nucleotides C) a deletion of a codon D) a substitution of the third nucleotide in an ACC codon

B) a deletion of two nucleotides

If an adult person has a faulty version of the human analog to ced-4 of the nematode, which of the following is most likely to result? A) activation of a developmental pathway found in the worm but not in humans B) a form of cancer in which there is insufficient apoptosis C) formation of molecular pores in the mitochondrial outer membrane D) excess skin loss

B) a form of cancer in which there is insufficient apoptosis

The SRY gene is best described as ________. A) an autosomal gene that is required for the expression of genes on the X chromosome B) a gene present on the Y chromosome that triggers male development C) a gene present on the X chromosome that triggers female development D) an autosomal gene that is required for the expression of genes on the Y chromosome

B) a gene present on the Y chromosome that triggers male development

A cleavage furrow is ________. A) a ring of vesicles forming a cell plate B) a groove in the plasma membrane between daughter nuclei C) the separation of divided prokaryotes D) the space that is created between two chromatids during anaphase

B) a groove in the plasma membrane between daughter nuclei

Not all intercellular signals require transduction. Which one of the following signals would be processed without transduction? A) a signal that binds to a receptor in the cell membrane B) a lipid-soluble signal C) a signal that binds to the extracellular matrix D) a signal that is weakly bound to a nucleotide

B) a lipid-soluble signal

Hemoglobin is ________. A) a quaternary protein with two polypeptides B) a quaternary protein with four polypeptides C) a tertiary protein with two polypeptides D) a tertiary protein with four polypeptides

B) a quaternary protein with four polypeptides

Which of the following characteristics would you expect of a eukaryotic organism that lacks the enzyme telomerase? A) a high probability of somatic cells becoming cancerous B) a reduction in chromosome length in gametes C) an inability to repair thymine dimers D) an inability to produce Okazaki fragments

B) a reduction in chromosome length in gametes

A series of enzymes catalyze the reactions in the metabolic pathway X → Y → Z → A. Product A binds to the enzyme that converts X to Y at a position remote from its active site. This binding decreases the activity of the enzyme. What is substance X? A) an intermediate B) a substrate C) the product D) an allosteric inhibitor

B) a substrate

The oxygen consumed during cellular respiration is directly involved in which of the following processes or events? A) glycolysis B) accepting electrons at the end of the electron transport chain. C) the citric acid cycle D) the oxidation of pyruvate to acetyl CoA

B) accepting electrons at the end of the electron transport chain

Which one of the following is formed by the removal of a carbon (as CO2) from a molecule of pyruvate? A) ATP B) acetyl CoA C) water D) citrate

B) acetyl CoA

In the presence of oxygen, the three-carbon compound pyruvate can be catabolized in the citric acid cycle. First, however, the pyruvate (1) loses a carbon, which is given off as a molecule of CO2, (2) is oxidized to form a two-carbon compound called acetate, and (3) is bonded to coenzyme A. Which of the following sets of products result from these reactions? A) acetyl CoA, FADH2, and CO2 B) acetyl CoA, NADH, and CO2 C) acetyl CoA, NAD+, ATP, and CO2 D) acetyl CoA, O2, and ATP

B) acetyl CoA, NADH, and CO2

Protein kinase is an enzyme that functions in which of the following ways? A) as a receptor for various signal molecules B) activates or inactivates other proteins by adding a phosphate group to them C) activates a G protein D) as a second messenger molecule

B) activates or inactivates other proteins by adding a phosphate group to them

Which of the following processes generally requires protein phosphorylation? A) activation of steroid hormone receptors B) activation of receptor tyrosine kinases C) activation of G protein-coupled receptors D) activation ligand-gated ion channels

B) activation of receptor tyrosine kinases

The evolution of one species into two or more species as a result of different populations becoming reproductively isolated from each other is best termed as ________. A) prototype B) adaptive radiation C) creationism D) natural selection

B) adaptive radiation

Some carbon skeletons have different numbers and locations of double bonds to ________. A) increase its solubility in water B) add molecular complexity and diversity that characterize living matter C) be more flexible that makes the molecule stronger D) stay in its liquid state

B) add molecular complexity and diversity that characterize living matter

In the formation of biofilms, such as those forming on unbrushed teeth, cell signaling serves which function? A) formation of mating complexes B) aggregation of bacteria that can cause cavities C) secretion of substances that inhibit foreign bacteria D) digestion of unwanted parasite populations

B) aggregation of bacteria that can cause cavities

Metaphase is characterized by ________. A) separation of the centromeres B) alignment of chromosomes on the equator of the cell C) cytokinesis D) separation of sister chromatids

B) alignment of chromosomes on the equator of the cell

What kinds of cells carry out ATP synthesis by chemiosmosis? A) only eukaryotic cells, both plant and animal, using either oxygen or other electron acceptors B) all respiring cells, both prokaryotic and eukaryotic, using either oxygen or other electron acceptors C) all cells, both prokaryotic and eukaryotic, exclusively using oxygen as the electron acceptor D) only animal cells in mitochondria, exclusively using oxygen as the electron acceptor

B) all respiring cells, both prokaryotic and eukaryotic, using either oxygen or other electron acceptors

For a protein to be an integral membrane protein, it would have to be ________. A) hydrophilic B) amphipathic, with at least one hydrophobic region C) hydrophobic D) exposed on only one surface of the membrane

B) amphipathic, with at least one hydrophobic region

Which kind of metabolic poison would most directly interfere with glycolysis? A) an agent that reacts with NADH and oxidizes it to NAD+ B) an agent that closely mimics the structure of glucose but is not metabolized C) an agent that binds to pyruvate and inactivates it D) an agent that reacts with oxygen and depletes its concentration in the cell

B) an agent that closely mimics the structure of glucose but is not metabolized

Atoms have no electric charge because they have ________. A) an equal number of protons and neutrons B) an equal number of protons and electrons C) uncharged neutrons in their nuclei D) an equal number of charged and uncharged subatomic particles

B) an equal number of protons and electrons

Which of the following processes might produce a human zygote with 45 chromosomes? A) failure of the egg nucleus to be fertilized by the sperm B) an error in meiotic anaphase occurring in either an egg or sperm C) incomplete cytokinesis during spermatogenesis after meiosis I D) failure of an egg to complete meiosis II

B) an error in meiotic anaphase occurring in either an egg or sperm

In comparison to eukaryotes, prokaryotes ________. A) do not have membranes B) are smaller C) are more structurally complex D) are larger

B) are smaller

In yeast signal transduction, a yeast cell releases a mating factor, which ________. A) acts back on the same cell that secreted the mating factor, changing its development B) binds to receptors on the membranes of other types of yeast cells C) passes through the membranes of neighboring cells, binds to DNA, and initiates transcription D) diffuses through the membranes of distant cells, causing them to produce factors that initiate long-distance migrations

B) binds to receptors on the membranes of other types of yeast cells

A gardener is concerned that her greenhouse is getting too hot from too much light and seeks to shade her plants with colored translucent plastic sheets, the color of which allows passage of only that wavelength. What color should she use to reduce overall light energy but still maximize plant growth? A) green B) blue C) orange D) Any color will work equally well.

B) blue

How does a noncompetitive inhibitor decrease the rate of an enzyme-catalyzed reaction? A) by decreasing the free-energy change of the reaction catalyzed by the enzyme B) by binding to an allosteric site, thus changing the shape of the active site of the enzyme C) by binding to the substrate, thus changing its shape so that it no longer binds to the active site of the enzyme D) by binding to the active site of the enzyme, thus preventing binding of the normal substrate

B) by binding to an allosteric site, thus changing the shape of the active site of the enzyme

How do steroid hormones produce their effects in cells? A) by promoting the degradation of specific mRNAs B) by binding to intracellular receptors and promoting transcription of specific genes C) by binding to the regulatory gene in an operon D) by activating translation of certain mRNAs

B) by binding to intracellular receptors and promoting transcription of specific genes

Many organisms spend most of their life cycle in the diploid state. If meiosis produces haploid cells, how is the diploid number restored for these types of organisms? A) through the transcription of DNA to RNA B) by fertilization C) by DNA replication D) by synapsis of the homologous pairs of chromosomes during prophase of meiosis I

B) by fertilization

Compared to C3 plants, C4 plants ________. A) have higher rates of photorespiration B) can continue to fix CO2 even at lower CO2 concentrations and higher oxygen concentrations C) do not use rubisco for carbon fixation D) make a four-carbon compound, oxaloacetate, which is then delivered to the citric acid cycle in mitochondria

B) can continue to fix CO2 even at lower CO2 concentrations and higher oxygen concentrations

Referring to the accompanying figure, where would elevated concentrations of oxygen inhibit carbon fixation reactions? A) cell I only B) cell II only C) both cell I and cell II D) neither cell I nor cell II

B) cell II only

What is the name of the microtubule-organizing center found in animal cells as an identifiable structure present during all phases of the cell cycle? A) centromere B) centrosome C) centriole D) kinetochore

B) centrosome

Genetic drift produces variation for evolution when ________. A) a population has heritable traits better suited to the environment B) chance events cause allele frequencies to fluctuate unpredictably C) sudden change in environment drastically reduces the gene pool D) a gene pool decreases because a smaller group establishes a new population

B) chance events cause allele frequencies to fluctuate unpredictably

Which polysaccharide is an important component in the structure of many animals and fungi? A) cellulose B) chitin C) amylose D) amylopectin

B) chitin

Which of the following types of molecules lack hydrophilic domains? A) transmembrane proteins B) cholesterol C) peripheral membrane proteins D) integral membrane proteins

B) cholesterol

Which of the following molecules helps to "turn off" genes in a cell? A) inducer B) corepressor C) operon D) promoter

B) corepressor

In some cells, there are many ion electrochemical gradients across the plasma membrane even though there are usually only one or two proton pumps present in the membrane. The gradients of the other ions are most likely accounted for by ________. A) pores in the plasma membrane B) cotransport proteins C) ion channels D) passive diffusion across the plasma membrane

B) cotransport proteins

You are maintaining a small population of fruit flies in the laboratory by transferring the flies to a new culture bottle after each generation. After several generations, you notice that the viability of the flies has decreased greatly. Recognizing that small population size is likely to be linked to decreased viability, the best way to reverse this trend is to ________. A) change the temperature at which you rear the flies B) cross your flies with flies from another lab C) reduce the number of flies that you transfer at each generation D) transfer only the largest flies

B) cross your flies with flies from another lab

Activity of various enzymes at various temperatures (a) and at various pH (b) Which curves on the graphs may represent the temperature and pH profiles of an enzyme taken from a bacterium that lives in a mildly alkaline hot springs at temperatures of 70°C or higher? A) curves 3 and 4 B) curves 3 and 5 C) curves 2 and 5 D) curves 1 and 5

B) curves 3 and 5

As a research scientist, you measure the amount of ATP and NADPH consumed by the Calvin cycle in one hour. You find that 30,000 molecules of ATP were consumed, but only 20,000 molecules of NADPH were consumed. What is the source of the extra ATP molecules? A) photosystem I B) cyclic electron flow C) linear electron flow D) photosystem II

B) cyclic electron flow

A researcher found a method she could use to manipulate and quantify phosphorylation and methylation in embryonic cells in culture. In one set of experiments, she succeeded in increasing acetylation of histone tails in the chromatin of the cells. Which of the following results would she most likely see in these cells? A) inactivation of the selected genes B) decreased chromatin condensation C) decreased binding of transcription factors D) increased chromatin condensation

B) decreased chromatin condensation

Which of the following is the most predictable outcome of increased gene flow between two populations? A) increased genetic difference between the two populations B) decreased genetic difference between the two populations C) lower average fitness in both populations D) higher average fitness in both populations

B) decreased genetic difference between the two populations

A decrease in entropy is associated with which type of reaction? A) hydrolysis B) dehydration C) depolymerization D) catabolic

B) dehydration

The activation of receptor tyrosine kinases is characterized by A) dimerization and IP3 binding. B) dimerization and phosphorylation. C) GTP hydrolysis. D) a phosphorylation cascade.

B) dimerization and phosphorylation.

Use the figure to answer the question. Refer to the metabolic pathway illustrated. If A, B, and C are all required for growth, a strain that is mutant for the gene-encoding enzyme A would be able to grow on medium supplemented with which of the following nutrient(s)? A) nutrients A and C B) either nutrient B or C C) nutrient C only D) nutrient A only

B) either nutrient B or C

Forms of the Ras protein found in tumors usually cause which of the following events to occur? A) DNA replication to stop B) excessive cell division C) cell-to-cell adhesion to be nonfunctional D) cell division to cease

B) excessive cell division

The figure illustrates the energy states associated with the reaction A + B ↔ C + D. Which of the following terms best describes the forward reaction in the figure? A) exergonic, ∆G > 0 B) exergonic, ∆G < 0 C) endergonic, ∆G > 0 D) endergonic, ∆G < 0

B) exergonic, ∆G < 0

Muscle cells differ from nerve cells mainly because they A) contain different genes. B) express different genes. C) have unique ribosomes. D) use different genetic codes.

B) express different genes.

Abnormal chromosomes are frequently found in malignant tumors. Errors such as translocations may place a gene in close proximity to different control regions. Which of the following events might then occur to make the cancer worse? A) failure of the cancer cells to multiply B) expression of inappropriate gene products C) an increase in nondisjunction D) a decrease in mitotic frequency

B) expression of inappropriate gene products

Cystic fibrosis is a genetic disorder in homozygous recessives that causes death during the teenage years. If 9 in 10,000 newborn babies have the disease, what are the expected frequencies of the dominant (A1) and recessive (A2) alleles according to the Hardy-Weinberg equation? A) f(A1) = 0.9997, f(A2) = 0.0003 B) f(A1) = 0.9700, f(A2) = 0.0300 C) f(A1) = 0.9800, f(A2) = 0.0200 D) f(A1) = 0.9604, f(A2) = 0.0392

B) f(A1) = 0.9700, f(A2) = 0.0300

If the cell whose nuclear material is shown in the figure continues toward completion of mitosis, which of the following events would occur next? A) spindle fiber formation B) formation of telophase nuclei C) synthesis of chromatids D) nuclear envelope breakdown

B) formation of telophase nuclei

The Dunkers are a religious group that moved from Germany to Pennsylvania in the mid-1700s. They do not marry with members outside their own immediate community. Today, the Dunkers are genetically unique and differ in gene frequencies, at many loci, from all other populations including those in their original homeland. Which of the following mechanisms likely explains the genetic uniqueness of this population? A) mutation and natural selection B) founder effect and genetic drift C) population bottleneck and Hardy-Weinberg equilibrium D) heterozygote advantage and stabilizing selection

B) founder effect and genetic drift

The restriction enzymes of bacteria protect the bacteria from successful attack by bacteriophages, whose genomes can be degraded by the restriction enzymes. The bacterial genomes are not vulnerable to these restriction enzymes because bacterial DNA is methylated. This situation selects for bacteriophages whose genomes are also methylated. As new strains of resistant bacteriophages become more prevalent, this in turn selects for bacteria whose genomes are not methylated and whose restriction enzymes instead degrade methylated DNA. The outcome of the conflict between bacteria and bacteriophages at any point in time results from ________. A) evolutionary imbalance B) frequency-dependent selection C) heterozygote advantage D) neutral variation

B) frequency-dependent selection

When imbalances occur in the sex ratio of sexual species that have two sexes (that is, other than a 50:50 ratio), the members of the minority sex often receive a greater proportion of care and resources from parents than do the offspring of the majority sex. This is most clearly an example of ________. A) sexual selection B) frequency-dependent selection C) stabilizing selection D) balancing selection

B) frequency-dependent selection

lasmodesmata in plant cells are most similar in function to which of the following structures in animal cells? A) desmosomes B) gap junctions C) extracellular matrix D) tight junctions

B) gap junctions

Over time, the movement of people on Earth has steadily increased. This has altered the course of human evolution by increasing ________. A) nonrandom mating B) gene flow C) genetic drift D) geographic isolation

B) gene flow

Which of the following sequences describes the path by which electrons travel downhill energetically in aerobic respiration? A) glucose → pyruvate → ATP → oxygen B) glucose → NADH → electron transport chain →oxygen C) glucose → pyruvate → electron transport chain →NADH → ATP D) food → glycolysis → citric acid cycle → NADH →ATP

B) glucose → NADH → electron transport chain →oxygen

Halobacterium has a photosynthetic membrane that appears purple. Its photosynthetic action spectrum is the inverse of the action spectrum for green plants (see figure). That is, the Halobacterium action spectrum has a peak where the green plant action spectrum has a trough. What wavelengths of light do the Halobacterium photosynthetic pigments absorb? A) red and yellow B) green and yellow C) violet and blue D) blue and red

B) green and yellow

A scientist, who wants to study the effects of nitrogen on wheat plants, sets up an experiment with 4 groups of wheat plants: group A gets 20 pounds per acre, group B gets 40 pounds per acre, group C gets 60 pounds per acre, and group D gets 0 pounds per acre. Which of the following is the control group? A) group C B) group D C) group A D) group B

B) group D

Testosterone and estradiol are male and female sex hormones, respectively, in many vertebrates. In what way(s) do these molecules differ from each other? Testosterone and estradiol ________. A) are cis-trans isomers but have the same molecular formula B) have different functional groups attached to the same carbon skeleton C) are structural isomers but have the same molecular formula D) are enantiomers of the same organic molecule

B) have different functional groups attached to the same carbon skeleton

A woman is found to have 47 chromosomes, including three X chromosomes. Which of the following statements describes her expected phenotype? A) a female with masculine characteristics such as facial hair B) healthy female of slightly above-average height C) a sterile female D) an apparent male who is sterile

B) healthy female of slightly above-average height

Low humidity in the atmosphere on a hot day ________. A) does not help in cooling because skin absorbs water from the atmosphere B) helps in cooling because water evaporates from the skin faster C) does not help in cooling because water evaporates from the skin faster D) helps in cooling because skin absorbs water from the atmosphere

B) helps in cooling because water evaporates from the skin faster

Thermal energy of the water in a bathtub is ________ than in a freshly brewed coffee pot because of its ________. A) lower; low density B) higher; greater volume C) lower; low temperature D) higher; high kinetic energy

B) higher; greater volume

In a nucleosome, the DNA is wrapped around A) ribosomes. B) histones. C) polymerase molecules. D) a thymine dimer.

B) histones.

Mutations in which of the following genes lead to transformations in the identity of entire body parts? A) segmentation genes B) homeotic genes C) inducers D) egg-polarity genes

B) homeotic genes

Several epidemic microbial diseases of earlier centuries incurred high death rates because they resulted in severe dehydration due to vomiting and diarrhea. Today they are usually not fatal because we have developed which of the following types of treatments? A) intravenous feeding techniques B) hydrating drinks with high concentrations of salt and glucose C) antiviral medications that are efficient and work well with most viruses D) medications to slow blood loss

B) hydrating drinks with high concentrations of salt and glucose

Viagra causes dilation of blood vessels and increased blood flow to the penis, facilitating erection. Viagra acts by inhibiting which of the following events? A) formation of cGMP from GTP B) hydrolysis of cGMP to GMP C) hydrolysis of GTP to GDP D) dephosphorylation of cGMP

B) hydrolysis of cGMP to GMP

Scientists isolate cells in various phases of the cell cycle. They find a group of cells that have 1.5 times more DNA than G1 phase cells. The cells of this group are... A) between the G1 and S phases in the cell cycle B) in the S phase of the cell cycle C) in the M phase of the cell cycle D) in the G2 phase of the cell cycle

B) in the S phase of the cell cycle

Homozygotes with two sickle-cell alleles are selected against because of mortality. However, heterozygotes for sickle-cell allele experience little effects of sickle allele and are more likely to survive malaria. The net effect of this exposure produced evolutionary change in areas where malaria is endemic by ________. A) increasing the number of infected mosquitoes B) increasing sickle-cell allele frequency C) distortion of red blood cells D) causing sickle-cell allele

B) increasing sickle-cell allele frequency

Natural selection changes allele frequencies because some ________ survive and reproduce better than others. A) loci B) individuals C) species D) alleles

B) individuals

Most repressor proteins are allosteric. Which of the following molecules binds with the repressor to alter its conformation and therefore affect its function? A) transcription factor B) inducer C) cAMP D) promoter

B) inducer

Once a peptide bond has been formed between the amino acid attached to the tRNA in the P site and the amino acid associated with the tRNA in the A site, what process occurs next? A) reading of the next codon of mRNA B) initiation C) translocation D) The codon-anticodon hydrogen bonds holding the tRNA in the A site are broken.

B) initiation

Besides the ability of some cancer cells to over proliferate, which of the following situations might logically result in a tumor? A) failure of cells to enter S phase B) lack of appropriate cell death C) changes in the order of cell cycle stages D) inability to form spindles

B) lack of appropriate cell death

You briefly expose bacteria undergoing DNA replication to radioactively labeled nucleotides. When you centrifuge the DNA isolated from the bacteria, the DNA separates into two classes. One class of labeled DNA includes very large molecules (thousands or even millions of nucleotides long), and the other includes short stretches of DNA (several hundred to a few thousand nucleotides in length). Which two classes of DNA do these different samples represent? A) Okazaki fragments and RNA primers B) leading strands and Okazaki fragments C) leading strands and RNA primers D) lagging strands and Okazaki fragments

B) leading strands and Okazaki fragments

In a human karyotype, chromosomes are arranged in 23 pairs. If we choose one of these pairs, such as pair 14, which of the following do the two chromosomes of the pair have in common? A) length and position of the centromere only B) length, centromere position, staining pattern, and traits coded for by their genes C) They have nothing in common except that they are X-shaped. D) length, centromere position, and staining pattern only

B) length, centromere position, staining pattern, and traits coded for by their genes

A bacterium engulfed by a white blood cell through phagocytosis will be digested by enzymes contained in ________. A) vacuoles B) lysosomes C) secretory vesicles D) Golgi vesicles

B) lysosomes

Crossing over of chromosomes normally takes place during which of the following processes? A) mitosis and meiosis II B) meiosis I C) mitosis D) meiosis II

B) meiosis I

The M phase checkpoint ensures that all chromosomes are attached to the mitotic spindle. If this does not happen, cells would most likely be arrested in ________. A) G2 B) metaphase C) prophase D) telophase

B) metaphase

Cyanide binds to at least one molecule involved in producing ATP. If a cell is exposed to cyanide, most of the cyanide will be found within the... A) ribosomes. B) mitochondria. C) peroxisomes. D) lysosomes.

B) mitochondria.

Which structure is common to plant and animal cells? A) centriole B) mitochondrion C) central vacuole D) chloroplast

B) mitochondrion

Asexual reproduction occurs during which of the following processes? A) the exchange of chromosomes between organisms of different species B) mitosis C) meiosis D) fertilization

B) mitosis

You are asked to indicate the type and number of atoms in a molecule. Which representation would work best? A) space-filling model B) molecular formula C) structural formula D) ball-and-stick model

B) molecular formula

Inheritance patterns cannot always be explained by Mendel's models of inheritance. If a pair of homologous chromosomes fails to separate during meiosis I, select the choice that shows the chromosome number of the four resulting gametes with respect to the normal haploid number (n)? A) n + 1; n + 1; n; n B) n + 1; n + 1; n - 1; n - 1 C) n + 1; n - 1; n - 1; n - 1 D) n + 1; n - 1; n; n

B) n + 1; n + 1; n - 1; n - 1

In 1983, a population of dark-eyed junco birds became established on the campus of the University of California, San Diego (UCSD), which is located many miles from the junco's normal habitat in the mixed-coniferous temperate forests in the mountains. Juncos have white outer tail feathers that the males display during aggressive interactions and during courtship displays. Males with more white in their tail are more likely to win aggressive interactions, and females prefer to mate with males with more white in their tails. Females have less white in their tails than do males, and display it less often. (Pamela J. Yeh. 2004. Rapid evolution of a sexually selected trait following population establishment in a novel habitat.Evolution 58[1]:166-74.) The UCSD campus male junco population tails are about 36% white, whereas the tails of males from nearby mountain populations are about 40-45% white. The founding stock of UCSD birds was likely from the nearby mountain populations because some of those birds overwinter on the UCSD campus each year. Population sizes on the UCSD campus have been reasonably large, and there are significant habitat differences between the UCSD campus and the mountain coniferous forests; UCSD campus has a more open environment (making birds more visible) and a lower junco density (decreasing intraspecific competition) than the mountain forests. Given this information, which of the following evolutionary mechanisms do you think is most likely responsible for the difference between the UCSD and mountain populations? A) gene flow B) natural selection C) genetic drift D) mutation

B) natural selection

A proficient engineer can easily design skeletal structures that are more functional than those currently found in the forelimbs of such diverse mammals as horses, whales, and bats. The actual forelimbs of these mammals do not seem to be optimally arranged because ________. A) though we may not consider the fit between the current skeletal arrangements and their functions excellent, we should not doubt that natural selection ultimately produces the best design B) natural selection is generally limited to modifying structures that were present in previous generations and in previous species C) natural selection has not had sufficient time to create the optimal design in each case, but will do so given enough time D) in many cases, phenotype is determined by genotype and the environment

B) natural selection is generally limited to modifying structures that were present in previous generations and in previous species

Suppose a cell has the following molecules and structures: enzymes, DNA, ribosomes, plasma membrane, and mitochondria. It could be a cell from ________. A) an animal, but not a plant B) nearly any eukaryotic organism C) a plant, but not an animal D) a bacterium, but not a eukaryote

B) nearly any eukaryotic organism

Refer to the following figure (first three rows of the periodic table) to answer the questions below. Which pair of elements would likely have similar valency and thus similar chemical behavior? A) hydrogen and helium B) nitrogen and phosphorus C) carbon and nitrogen D) sodium and chlorine

B) nitrogen and phosphorus

At the beginning of this century, there was a general announcement regarding the sequencing of the human genome and the genomes of many other multicellular eukaryotes. Many people were surprised that the number of protein-coding sequences was much smaller than they had expected. Which of the following types of DNA make up the rest of the human genome? A) DNA that consists of histone coding sequences B) non-protein-coding DNA that is transcribed into several kinds of small RNAs with biological function C) non-protein-coding DNA that serves as binding sites for reverse transcriptase D) DNA that is translated directly without being transcribed

B) non-protein-coding DNA that is transcribed into several kinds of small RNAs with biological function

A man who is a dwarf with achondroplasia and normal vision marries a color-blind woman of normal height. The man's father was six feet tall, and both the woman's parents were of average height. Dwarfism caused by achondroplasia is autosomal dominant, and red-green color blindness is X-linked recessive. How many of their daughters might be expected to be color-blind with achondroplasia? A) half B) none C) three out of four D) one out of four

B) none

E. coli cells grown on 15N medium are transferred to 14N medium and allowed to grow for two more generations (two rounds of DNA replication). DNA extracted from these cells is centrifuged. What density distribution of DNA would you expect in this experiment? A) one intermediate-density band B) one low-density and one intermediate-density band C) one high-density and one low-density band D) one high-density and one intermediate-density band

B) one low-density and one intermediate-density band

Semiconservative replication involves a template. What is the template? A) an RNA molecule B) one strand of the DNA molecule C) DNA polymerase D) single-stranded binding proteins

B) one strand of the DNA molecule

Which of the following types of cells are affected most by telomere shortening? A) only animal cells B) only eukaryotic cells C) cells in prokaryotes and eukaryotes D) only prokaryotic cells

B) only eukaryotic cells

What essential information does the product of the bicoid gene in Drosophila provide during development? A) orientation of the dorsal-ventral axis B) orientation of the anterior-posterior axis C) orientation of the left-right axis D) segmentation

B) orientation of the anterior-posterior axis

Which of the following processes is driven by chemiosmosis? A) ATP hydrolysis B) oxidative phosphorylation C) substrate-level phosphorylation D) reduction of NAD+ to NADH

B) oxidative phosphorylation

The final electron acceptor of the electron transport chain that functions in aerobic oxidative phosphorylation is A) water. B) oxygen. C) NAD+. D) pyruvate.

B) oxygen.

Which of the following is a type of local signaling in which a cell secretes a signal molecule that affects neighboring cells? A) synaptic signaling B) paracrine signaling C) autocrine signaling D) hormonal signaling

B) paracrine signaling

Which of the following types of molecules are hydrophilic and therefore excluded from the hydrophobic portion of the phospholipid bilayer? A) cholesterol B) peripheral membrane proteins C) transmembrane proteins D) integral membrane proteins

B) peripheral membrane proteins

An inhibitor of which of the following enzymes could be used to block the release of calcium from the endoplasmic reticulum? A) phosphodiesterase B) phospholipase C C) serine/threonine kinases D) adenylyl cyclase

B) phospholipase C

Among the newly discovered small noncoding RNAs, one type helps to reestablish ethylation patterns during gamete formation and blocks expression of some transposons. Which of the following types of RNA is responsible for this function? A) siRNA B) piRNA C) lncRNA D) miRNA

B) piRNA

The structural level of a protein least affected by a disruption in hydrogen bonding is the A) secondary level. B) primary level. C) quaternary level. D) tertiary level.

B) primary level.

You disrupt all hydrogen bonds in a protein. What level of structure will be preserved? A) quaternary structure B) primary structure C) secondary structure D) tertiary structure

B) primary structure

Which of the following results would most likely be an immediate result of a growth factor binding to its receptor? A) adenylyl cyclase activity B) protein kinase activity C) phosphorylase activity D) cAMP activity

B) protein kinase activity

Which of the following does not occur during mitosis? A) separation of sister chromatids B) replication of the DNA C) spindle formation D) condensation of the chromosomes

B) replication of the DNA

Anchorage dependence of animal cells in vitro or in vivo depends on which of the following? A) attachment of spindle fibers to centrioles B) response of the cell cycle controls to signals from the plasma membrane C) the binding of cell-surface phospholipids to those of adjoining cells D) response of the cell cycle controls to the binding of cell-surface phospholipids

B) response of the cell cycle controls to signals from the plasma membrane

Which of the following is the most common pathway taken by a newly synthesized protein that will be secreted by a cell? A) rough ER → Golgi → transport vesicle → nucleus B) rough ER → Golgi → transport vesicle → plasma membrane C) rough ER → lysosome → transport vesicle → plasma membrane D) Golgi → rough ER → lysosome → transport vesicle → plasma membrane

B) rough ER → Golgi → transport vesicle → plasma membrane

Which of the following would be most appropriate method to observe the three-dimensional structure and organization of microvilli on an intestinal cell? A) transmission electron microscopy B) scanning electron microscopy C) standard light microscopy D) a hand lens (magnifying glass)

B) scanning electron microscopy

Which of the following processes facilitates the fastest way for organisms to adapt to a changing environment? A) mitosis B) sexual reproduction C) asexual reproduction D) mutation

B) sexual reproduction

When a neuron responds to a particular neurotransmitter by opening gated ion channels, the neurotransmitter is serving as which part of the signal pathway? A) transducer B) signal molecule C) response molecule D) relay molecule

B) signal molecule

A research team is working on the design of a new drug for the treatment of lung cancer. To be most effective, this drug must specifically enter the cytoplasm of lung cells while not entering the cells of other tissues. Which of the following characteristics would likely enhance the specificity of this drug? A) the relative hydrophobicity of the drug molecule B) similarity of the drug molecule to other molecules normally transported lung cells C) phospholipid composition of lung cell plasma membranes D) specificity of the drug molecule for binding to the extracellular matrix of lung cells

B) similarity of the drug molecule to other molecules normally transported lung cells

Which of the following types of molecules help to hold the DNA strands apart while they are being replicated? A) ligase B) single-strand DNA binding proteins C) primase D) DNA polymerase

B) single-strand DNA binding proteins

What is the maximum number of hydrogen atoms that can be covalently bonded in a molecule containing two carbon atoms? A) four B) six C) eight D) two

B) six

To act as an effective coolant in a car's radiator, a substance has to have the capacity to absorb a great deal of heat. You have a reference book with tables listing the physical properties of many liquids. In choosing a coolant for your car, which table would you check first? A) heat of vaporization B) specific heat C) pH D) density at room temperature

B) specific heat

The oxygen released by photosynthesis is produced by which of the following processes? A) chemiosmosis B) splitting water molecules C) the electron transfer system of photosystem I D) the electron transfer system of photosystem II

B) splitting water molecules

A friend of yours calls to say that his car would not start this morning. He asks for your help. You say that you think the battery must be dead. If so, then jump-starting the car from a good battery will solve the problem. In doing so, you are ________. A) comparing multiple hypotheses for why the car will not start B) stating a hypothesis and using that hypothesis to make a testable prediction C) testing a theory for why the car will not start D) making observations to inspire a theory for why the car will not start

B) stating a hypothesis and using that hypothesis to make a testable prediction

In mitochondria, an electron transport chain pumps protons from the matrix into the intermembrane space, whereas in chloroplasts, an electron transport chain pumps protons from the ________. A) matrix to the stroma B) stroma to the thylakoid space C) thylakoid space to the matrix D) thylakoid space to the stroma

B) stroma to the thylakoid space

Which branch of biology is concerned with the naming and classifying of organisms? A) genomics B) taxonomy C) evolution D) informatics

B) taxonomy

FtsZ is a bacterial cytoskeletal protein that forms a contractile ring involved in binary fission. Its function is analogous to ________. A) the mitotic spindle of eukaryotic cells B) the cleavage furrow of eukaryotic animal cells C) the cell plate of eukaryotic plant cells D) the microtubule-organizing center of eukaryotic cells

B) the cleavage furrow of eukaryotic animal cells

Which of the following results would occur if the repressor of an inducible operon were mutated so that it could not bind the operator? A) the irreversible binding of the repressor to the promoter B) the continuous transcription of the operon's genes C) the reduced transcription of the operon's genes D) the overproduction of cAMP receptor protein (CRP)

B) the continuous transcription of the operon's genes

In the term trace element, the adjective trace means that A) the element enhances health but is not essential for the organism's long-term survival. B) the element is required in very small amounts. C) the element can be used as a label to trace atoms through an organism's metabolism. D) the element is very rare on Earth.

B) the element is required in very small amounts.

What are telomeres? A) enzymes that elongate the DNA strand during replication B) the ends of linear chromosomes C) the sites of origin of DNA replication D) the structures that hold two sister chromatids together

B) the ends of linear chromosomes

The difference between an aldose sugar and a ketose sugar is ________. A) the number of carbon atoms B) the position of the carbonyl group C) the position of the hydroxyl groups D) the ring form and the linear chain

B) the position of the carbonyl group

Cell differentiation always involves A) the selective loss of certain genes from the genome. B) the production of tissue-specific proteins. C) transcription of the myoD gene. D) the movement of cells.

B) the production of tissue-specific proteins.

Which action could produce a carbonyl group? A) the addition of a thiol to a hydroxyl B) the replacement of the -OH of a carboxyl group with hydrogen C) the addition of a hydroxyl to a phosphate D) the replacement of the nitrogen of an amine with oxygen

B) the replacement of the -OH of a carboxyl group with hydrogen

No two people are genetically identical, except for identical twins. The main source of genetic variation among humans is A) environmental effects. B) the reshuffling of alleles in sexual reproduction. C) new mutations that occurred in the preceding generation. D) genetic drift.

B) the reshuffling of alleles in sexual reproduction.

Which of the following aspects of enzyme structure is best described by a clasping handshake analogy? A) the specific manner in which an enzyme is denatured by low pH B) the specific manner in which an enzyme binds substrate C) the specific manner in which an enzyme interacts with water D) the specific manner in which an enzyme folds to form secondary and tertiary structures

B) the specific manner in which an enzyme binds substrate

Which of the following best demonstrates the unity among all organisms? A) natural selection B) the structure and function of DNA C) descent with modification D) emergent properties

B) the structure and function of DNA

In an ethane (C2H6) molecule, each carbon atom is bonded to ________ hydrogen atoms. A) four B) three C) two D) six

B) three

A fat (or triacylglycerol) would be formed as a result of a dehydration reaction between... A) one molecule of 5 and three molecules of 9 B) three molecules of 9 and one molecule of 10 C) one molecule of 5 and three molecules of 10 D) one molecule of 9 and three molecules of 10

B) three molecules of 9 and one molecule of 10

In E. coli, what is the function of DNA polymerase III? A) to seal together the broken ends of DNA strands B) to add nucleotides to the 3' end of a growing DNA strand C) to degrade damaged DNA molecules D) to unwind the DNA helix during replication

B) to add nucleotides to the 3' end of a growing DNA strand

Which of the following processes occurs in prokaryotes but not in eukaryotes? A) post-transcriptional splicing B) transcription and translation occur simultaneously C) gene splicing D) translation in the absence of a ribosome

B) transcription and translation occur simultaneously

In glycolysis, for each molecule of glucose oxidized to pyruvate, ________. A) two molecules of ATP are used, and two molecules of ATP are produced B) two molecules of ATP are used, and four molecules of ATP are produced C) four molecules of ATP are used, and two molecules of ATP are produced D) two molecules of ATP are used, and six molecules of ATP are produced

B) two molecules of ATP are used, and four molecules of ATP are produced

Which of the following molecules is a protein maintained at steady levels throughout the cell cycle that requires cyclin to become catalytically active? A. MPF B. Cdk C. Cyclin D. PDGF

B. Cdk

The beginning of anaphase is indicated by which of the following processes? A. Loss of kinetochores from the chromatids. B. Enzymatic cleavage of cohesion C. Attachment of sister chromatids to each other by cohesion. D. Disappearance of the nuclear membrane

B. Enzymatic cleavage of cohesion

Which of the following statements describes a key component of the induced fit hypothesis of enzyme catalysis? A. A competitive inhibitor can outcompete the substrate for the active site. B. The binding of the substrate changes the shape of the enzyme's active site. C. Some enzymes change their structure when activators bind to the enzyme. D. The active site creates a microenvironment ideal for the reaction.

B. The binding of the substrate changes the shape of the enzyme's active site.

Where are the proteins of the electron transport chain located? A. mitochondrial matrix B. mitochondrial inner membrane C. mitochondrial intermembrane space D. mitochondrial outer membrane

B. mitochondrial inner membrane

During a laboratory experiment, you discover that an enzyme-catalyzed reaction has a ∆Gof -20 kcal/mol. If you double the amount of enzyme in the reaction, what will be the ∆Gfor the new reaction? A) -10 kcal/mol B) +20 kcal/mol C) -20 kcal/mol D) -40 kcal/mol

C) -20 kcal/mol

In pea plants, the tall phenotype is dominant to the dwarf phenotype. If a heterozygous pea plant is crossed with a homozygous tall pea plant, what is the probability that the offspring will be dwarf in size? A) 1/2 B) 1 C) 0 D) 1/4

C) 0

A fruit fly population has a gene with two alleles, A1 and A2. Tests show that 70% of the gametes produced in the population contain the A1 allele. If the population is in Hardy-Weinberg equilibrium, what proportion of the flies carry both A1 and A2? A) 0.49 B) 0.7 C) 0.42 D) 0.21

C) 0.42

The solutions in the two arms of this U-tube are separated by a membrane that is permeable to water and glucose but not to sucrose. Side A is half-filled with a solution of 2 M sucrose and 1 M glucose. Side B is half-filled with 1 M sucrose and 2 M glucose. Initially, the liquid levels on both sides are equal. A U-shaped tube with water-permeable membrane at the center bottom of the U. The left side, A, is filled with 2 M sucrose and 1 M glucose. The right side, B, is filled with 1 M sucrose and 2 M glucose. The two sides are level with each other. When the system illustrated above reaches equilibrium, the sugar concentrations on both sides of the U-tube will be... A) 1 M sucrose, 1 M glucose B) 1 M sucrose, 2 M glucose C) 1.5 M sucrose, 1.5 M glucose D) 2 M sucrose, 1 M glucose

C) 1.5 M sucrose, 1.5 M glucose

What is the hydroxyl ion (OH-) concentration of a solution of pH 8? A) 8 × 10-6 M B) 8 M C) 10-8 M D) 10-6 M

C) 10-8 M

A dietary Calorie equals 1 kilocalorie. One kilocalorie equals ________. A) 10,000 calories, or the amount of heat required to raise the temperature of 1 kg of water by 1°F B) 1000 calories, or the amount of heat required to raise the temperature of 1 g of water by 1°C C) 1000 calories, or the amount of heat required to raise the temperature of 1 kg of water by 1°C D) 1000 calories, or the amount of heat required to raise the temperature of 100 g of water by 100°C

C) 1000 calories, or the amount of heat required to raise the temperature of 1 kg of water by 1°C

A slice of pizza has 500 kcal. If we could burn the pizza and use all the heat to warm a 50-L container of cold water, what would be the approximate increase in the temperature of the water? (Note: A liter of cold water weighs about 1 kg.) A) 5°C B) 100°C C) 10°C D) 50°C

C) 10°C

How many heme groups are present in three hemoglobin protein molecules? A) 9 B) 4 C) 12 D) 3

C) 12

Which of the following molecules is the pentose sugar found in RNA? A) 1 B) 13 C) 12 D) 6

C) 12

How many molecules of water are released during the polymerization of a 20 monomer-long cellulose molecule? A) 20 B) 10 C) 19 D) 40

C) 19

When Mendel crossed yellow-seeded and green-seeded pea plants, all the offspring were yellow-seeded. When he took these F1 yellow-seeded plants and crossed them to green-seeded plants, what genotypic ratio was expected? A) 1:1:1:1 B) 3:1 C) 1:1 D) 1:2:1

C) 1:1

Which of the following molecules act as building blocks (monomers) of polypeptides? A) 1, 4, and 6 B) 7, 8, and 13 C) 2, 7, and 8 D) 11, 12, and 13

C) 2, 7, and 8

In rabbits, the homozygous genotype LCLC has normal legs, LCLc results in deformed legs, and LcLc results in very short legs. The genotype FBFB produces black fur,FBFb brown fur, and FbFb white fur. If a cross is made between brown rabbits with deformed legs and white rabbits with deformed legs, what percentage of the offspring would be expected to have deformed legs and white fur? A) 33% B) 100% C) 25% D) 50%

C) 25%

Approximately how many molecules of ATP are produced from the complete oxidation of one molecule of glucose (C6H12O6) in aerobic cellular respiration? A) 4 B) 2 C) 30-32 D) 18-24

C) 30-32

If one strand of a DNA molecule has the sequence of bases 5′-ATTGCA-3′, the mRNA synthesized following the template will be ________. A) 5′-UGCAAU-3′ B) 5′-TAACGT-3′ C) 3′-UAACGU-5′ D) 5′-TGCAAT-3′

C) 3′-UAACGU-5′

Of the following human aneuploidies, which is the one that generally has the most severe impact on the health of the individual? A) 47, XXY B) 45, X C) 47, trisomy 21 D) 47, XXX

C) 47, trisomy 21

At a specific area of a chromosome, the sequence of nucleotides below is present where the chain opens to form a replication fork: 3' C C T A G G C T G C A A T C C 5' An RNA primer is formed starting at the underlined T (T) of the template. Which of the following represents the primer sequence? A) 5' A C G T T A G G 3' B) 5' G C C T A G G 3' C) 5' A C G U U A G G 3' D) 5' G C C U A G G 3'

C) 5' A C G U U A G G 3'

Which of the following pairs of base sequences could form a short stretch of a normal double helix of DNA? A) 5′-AGCT-3′ with 5′-TCGA-3′ B) 5′-GCGC-3′ with 5′-TATA-3′ C) 5′-ATGC-3′ with 5′-GCAT-3′ D) All of these pairs are correct.

C) 5′-ATGC-3′ with 5′-GCAT-3′

Use this model of a eukaryotic transcript to answer the following question. E = exon and I = intron 5′-UTR E1 I1 E2 I2 E3 I3 E4 UTR-3′ Which components of the previous molecule will also be found in mRNA in the cytosol? A) 5′-UTR I1 I2 I3 UTR-3′ B) 5′-E1 I1 E2 I2 E3 I3 E4-3′ C) 5′-UTR E1 E2 E3 E4 UTR-3′ D) 5′-E1 E2 E3 E4-3′

C) 5′-UTR E1 E2 E3 E4 UTR-3′

Starting with a fertilized egg (zygote), a series of six cell divisions would produce an early embryo with how many cells? A) 32 B) 16 C) 64 D) 12

C) 64

In analyzing the number of different bases in a DNA sample, which result would be consistent with the base-pairing rules? A) A + T = G + C B) A = G C) A + G = C + T D) A = C

C) A + G = C + T

Use the following map of four genes on a chromosome to answer the question. Between which two genes would you expect the highest frequency of recombination? A) A and E B) A and W C) A and G D) E and G

C) A and G

Which of the following statements is correct in describing the terms monohybrid cross and dihybrid cross? A) A monohybrid cross results in a 9:3:3:1 ratio, whereas a dihybrid cross gives a 3:1 ratio. B) A monohybrid cross is performed for one generation, whereas a dihybrid cross is performed for two generations. C) A dihybrid cross involves organisms that are heterozygous for two characters that are being studied, and a monohybrid cross involves organisms that are heterozygous for only one character being studied. D) A monohybrid cross involves a single parent, whereas a dihybrid cross involves two parents.

C) A dihybrid cross involves organisms that are heterozygous for two characters that are being studied, and a monohybrid cross involves organisms that are heterozygous for only one character being studied.

The genetic code is essentially the same for all organisms. From this, one can logically assume which of the following statements to be true? A) The same codons in different organisms translate into different amino acids. B) DNA was the first genetic material. C) A gene from an organism can theoretically be expressed by any other organism. D) Different organisms have different types of amino acids.

C) A gene from an organism can theoretically be expressed by any other organism.

Which of the following statements is most likely true of a protein that cotransports glucose and sodium ions into the intestinal cells of an animal? A) Glucose entering the cell down its concentration gradient provides energy for uptake of sodium ions against the electrochemical gradient. B) Sodium ions can move down their electrochemical gradient through the cotransporter whether or not glucose is present outside the cell. C) A substance that blocks sodium ions from binding to the cotransport protein will also block the transport of glucose. D) Sodium and glucose compete for the same binding site in the cotransporter.

C) A substance that blocks sodium ions from binding to the cotransport protein will also block the transport of glucose.

A particular triplet of bases in the coding sequence of DNA is AAA. The anticodon on the tRNA that binds the mRNA codon is ________. A) UUU B) TTT C) AAA D) UUA

C) AAA

What are the products of linear electron flow during the light reactions of photosynthesis? A) heat and fluorescence B) ATP and P700 C) ATP and NADPH D) ADP and NADP+

C) ATP and NADPH

Which of the following statements describes a central role that ATP plays in cellular metabolism? A) Hydrolysis of ATP provides an input of free energy for exergonic reactions. B) Its terminal phosphate bond is stronger than most covalent bonds in other biological macromolecules. C) ATP provides energy coupling between exergonic and endergonic reactions. D) Hydrolysis of the terminal phosphate group stores free energy that is used for cellular work.

C) ATP provides energy coupling between exergonic and endergonic reactions.

Which of the following statements correctly explains the fact that all seven of the pea plant traits studied by Mendel obeyed the principle of independent assortment? A) The diploid number of chromosomes in the pea plants was seven. B) None of the traits obeyed the law of segregation. C) All of the genes controlling the traits behaved as if they were on different chromosomes. D) All of the genes controlling the traits were located on the same chromosome.

C) All of the genes controlling the traits behaved as if they were on different chromosomes.

Refer to the following figure to answer the questions below. 3H2 + N2 ⇌ 2NH3 Which of the following is true for the reaction? A) Only the forward or reverse reactions can occur at one time. B) Hydrogen and nitrogen are the reactants of the reverse reaction. C) Ammonia is being formed and decomposed simultaneously. D) The reaction is nonreversible.

C) Ammonia is being formed and decomposed simultaneously.

Which of the following events accompanies absorption of energy by chlorophyll molecules of the reaction-center complex? A) A molecule of water is split. B) NADP+ is reduced to NADPH. C) An electron is excited. D) ATP is synthesized from the energy absorbed.

C) An electron is excited.

Density-dependent inhibition is a phenomenon in which crowded cells stop dividing at some optimal density and location. This phenomenon involves binding of a cell-surface protein to its counterpart on an adjoining cell's surface. A growth inhibiting signal is sent to both cells, preventing them from dividing. Certain external physical factors can affect this inhibition mechanism. Select the statement that makes a correct prediction about natural phenomena that could occur during the cell cycle to prevent cell growth. A) As cells become more numerous, they begin to squeeze against each other, restricting their size and ability to allow cell growth. B) As cells become more numerous, the protein kinases they produce begin to compete with each other until only one cell has the proteins necessary for growth. C) As cells become more numerous, the amount of required growth factors and nutrients per cell becomes insufficient to allow for cell growth. D) As cells become more numerous, more and more of them enter the synthesis part of the cell cycle and duplicate DNA to inhibit cell growth.

C) As cells become more numerous, the amount of required growth factors and nutrients per cell becomes insufficient to allow for cell growth.

Which of the following statements correctly describes a distinction between autotrophs and heterotrophs? A) Only heterotrophs have mitochondria. B) Only heterotrophs require oxygen. C) Autotrophs, but not heterotrophs, can nourish themselves beginning with CO2 and other nutrients that are inorganic. D) Cellular respiration is unique to heterotrophs.

C) Autotrophs, but not heterotrophs, can nourish themselves beginning with CO2 and other nutrients that are inorganic.

Which of the following statements is a correct distinction between autotrophs and heterotrophs? A) Only heterotrophs require chemical compounds from the environment. B) Cellular respiration is unique to heterotrophs. C) Autotrophs, but not heterotrophs, can nourish themselves beginning with CO2 and other nutrients that are inorganic. D) Only heterotrophs have mitochondria.

C) Autotrophs, but not heterotrophs, can nourish themselves beginning with CO2 and other nutrients that are inorganic.

The figure illustrates the energy states associated with the reaction A + B ↔ C + D. Which of the following represents the activation energy required for the enzyme-catalyzed reaction in the figure? A) D B) A C) B D) C

C) B

Theodor W. Engelmann illuminated a filament of algae with light that passed through a prism, thus exposing different segments of algae to different wavelengths of light. He added aerobic bacteria and then noted in which areas the bacteria congregated. He noted that the largest groups were found in the areas illuminated by the red and blue light. What did Engelmann conclude about the congregation of bacteria in the red and blue areas? A) Bacteria released excess carbon dioxide in these areas. B) Bacteria congregated in these areas due to an increase in the temperature of the red and blue light. C) Bacteria congregated in these areas because these areas had the most oxygen being released. D) Bacteria are attracted to red and blue light and thus these wavelengths are more reactive than other wavelengths.

C) Bacteria congregated in these areas because these areas had the most oxygen being released.

Suppose an experimenter becomes proficient with a technique that allows her to move DNA sequences within a prokaryotic genome. If a researcher moves the promoter for the lac operon to the region between the beta galactosidase (lacZ) gene and the permease (lacY) gene, which of the following results would be most likely? A) RNA polymerase will no longer transcribe permease. B) The three genes of the lac operon will be expressed normally. C) Beta galactosidase will not be produced. D) The operon will still transcribe the lacZ and lacY genes, but the mRNA will not be translated.

C) Beta galactosidase will not be produced.

How does 0.5 M sucrose (molecular mass 342) solution compare to 0.5 M glucose (molecular mass 180) solution? A) Sucrose has 171 mg/L, whereas glucose has 90 mg/L. B) Both have 6.02 × 1023 molecules. C) Both have 3.01 × 1023 molecules. D) Sucrose has 171 molecules, whereas glucose has 90.

C) Both have 3.01 × 1023 molecules.

Albinism is a recessive trait. A man and woman who both have normal pigmentation have one child out of three who has albinism (without melanin pigmentation). What are the genotypes of this child's parents? A) One parent must be heterozygous; the other parent can be homozygous dominant, homozygous recessive, or heterozygous. B) One parent must be homozygous for the recessive allele; the other parent can be homozygous dominant, homozygous recessive, or heterozygous. C) Both parents must be heterozygous. D) One parent must be homozygous dominant; the other parent must be heterozygous.

C) Both parents must be heterozygous.

Why might the alternative pathways of photosynthesis using C4 or CAM systems be described as metabolic compromises? A) Each one minimizes both water loss and rate of photosynthesis. B) C4 compromises on water loss and CAM compromises on photorespiration. C) Both pathways minimize photorespiration but also expend more ATP during carbon fixation. D) CAM plants allow more water loss, while C4 plants allow less CO2 into the plant.

C) Both pathways minimize photorespiration but also expend more ATP during carbon fixation.

Five dialysis bags, constructed of a type of membrane that is permeable to water and impermeable to sucrose, were filled with various concentrations of sucrose and then placed in separate beakers containing an initial concentration of 0.6 M sucrose solution. At 10-minute intervals, the bags were massed (weighed), and the percent change in mass of each bag was graphed. Which line in the graph represents the bag that contained a solution isotonic to the 0.6 M solution at the beginning of the experiment? A) A B) B C) C D) D

C) C

For the following questions, match the labeled component of the cell membrane in the figure with its description. Which component in the accompanying figure is a microfilament (actin filament) of the cytoskeleton? A) A B) B C) C D) D

C) C

Which of the following carbon molecules does not have the bond angle of 109.5°? A) CH4 B) C2H6 C) C2H4 D) C3H8

C) C2H4

Which of the summary statements below describes the results of the following reaction? C6H12O6 + 6 O2 → 6 CO2 + 6 H2O + Energy A) CO2 is reduced and O2 is oxidized. B) O2 is reduced and CO2 is oxidized. C) C6H12O6 is oxidized and O2 is reduced. D) O2 is oxidized and H2O is reduced.

C) C6H12O6 is oxidized and O2 is reduced.

A cell has enough available ATP to meet its needs for about 30 seconds. What is likely to happen when an athlete exhausts his or her ATP supply? A) ATP is transported into the cell from the circulatory system. B) He or she has to sit down and rest. C) Catabolic processes are activated that generate more ATP. D) Other cells take over, and the muscle cells that have used up their ATP cease to function.

C) Catabolic processes are activated that generate more ATP.

Both the volume and the surface area for three different cells were measured. These values are listed in the following table: Volume Surface Area Cell 1 9.3 μm3 26.5 μm2 Cell 2 12.2 μm3 37.1 μm2 Cell 3 17.6 μm3 40.6 μm2 Using data from the table above, select the best explanation for why that cell will be able to eliminate waste most efficiently? A) Cell 3, since it has the largest surface area, which will enable it to eliminate all of its wastes efficiently. B) Cell 1, since it has the smallest volume and will not produce as much waste as the other cells. C) Cell 2, since it has the highest ratio of surface area to volume, which facilitates the exchange of materials between a cell and its environment. D) Cell 3, because it is big enough to allow wastes to easily diffuse through the plasma membrane.

C) Cell 2, since it has the highest ratio of surface area to volume, which facilitates the exchange of materials between a cell and its environment.

Cells from advanced malignant tumors often have very abnormal chromosomes and an abnormal number of chromosomes. What might explain the association between malignant tumors and chromosomal abnormalities? A) Cancer cells are no longer density-dependent. B) Cancer cells are no longer anchorage-dependent. C) Cell cycle checkpoints are not in place to stop cells with chromosome abnormalities. D) Transformation introduces new chromosomes into cells.

C) Cell cycle checkpoints are not in place to stop cells with chromosome abnormalities.

Which of the following effects might be caused by reduced or very little active telomerase activity? A) Cells maintain normal functioning. B) Cells may become cancerous. C) Cells age and begin to lose function. D) Telomere lengthens in germ cells.

C) Cells age and begin to lose function.

Codons are three-base sequences in mRNA that specify the addition of a single amino acid to the growing protein chain during translation. How do eukaryotic codons and prokaryotic codons compare? A) Prokaryotic codons usually specify different amino acids than those of eukaryotes. B) The translation of codons is mediated by tRNAs in eukaryotes, but translation requires no intermediate molecules such as tRNAs in prokaryotes. C) Codons are a nearly universal language among all organisms. D) Prokaryotic codons usually contain different bases than those of eukaryotes.

C) Codons are a nearly universal language among all organisms.

Which of the following processes would allow the detection of alternative splicing of transcripts from a given gene? A) Compare the sequences of different primary transcripts made from the given gene. B) Compare the DNA sequence of the given gene to that of a similar gene in a related organism. C) Compare the sequences of different mRNAs made from the given gene. D) Measure the relative rates of transcription of the given gene compared to that of a gene known to be constitutively spliced.

C) Compare the sequences of different mRNAs made from the given gene.

Which of the following processes occurs when homologous chromosomes cross over in meiosis I? A) Two sister chromatids exchange identical pieces of DNA. B) Maternal alleles are "corrected" to be like paternal alleles and vice versa. C) Corresponding segments of non-sister chromatids are exchanged. D) Two sister chromatids get tangled, resulting in one re-sequencing its DNA.

C) Corresponding segments of non-sister chromatids are exchanged.

Which of the following occurrences describes how recombination between linked genes comes about? A) Nonrecombinant chromosomes break and then rejoin with one another. B) Linked genes travel together at anaphase. C) Crossovers between these genes result in chromosomal exchange. D) Independent assortment sometimes fails.

C) Crossovers between these genes result in chromosomal exchange.

Students conducted an experiment to determine the effect of light intensity on the rate of photosynthesis. They punched 40 leaf disks from spinach leaves and used a syringe partially filled with water to pull the gases from the leaf disks so that all leaf disks sunk to the bottom of the syringe. Ten leaf disks from the syringe were placed in each of four cups and covered with 50 ml of the solutions as indicated below. All leaf disks were resting on the bottom of the cups when the experiment began. The volume of liquid in each cup and the temperature of the solutions were held constant. All cups were placed 0.5 meters from the designated light source. A large beaker of water was placed between the light and the cups to act as a heat sink to prevent a change in temperature. At the end of 10 minutes, the number of disks floating in each cup was recorded. Trial Grams of baking soda (CO2source) Wattage of light bulb Disks floating at 10 minutes 1 0.5 25 3 2 0.5 50 5 3 0.5 75 9 4 0 75 0 Use your knowledge of the mechanism of photosynthesis and the data presented in the chart to determine which of the statements is a correct explanation for the students' data. A) Cup 1 had a low rate of photosynthesis because 0.5 grams of baking soda did not provide a sufficient amount of CO2. B) Cup 3 had the same rate of photosynthesis as Cup 1 because they had the same ratio of disks floating to wattage of light. C) Cup 4 had the lowest rate of photosynthesis because it had the least CO2. D) Cup 2 had the highest rate of photosynthesis because it had the highest ratio of disks floating to wattage of light.

C) Cup 4 had the lowest rate of photosynthesis because it had the least CO2.

Eukaryotic chromosomes are composed of which of the following macromolecules? A) DNA and phospholipids B) DNA and RNA C) DNA and proteins D) DNA only

C) DNA and proteins

The central rule of molecular biology states that ________. A) RNA is transcribed into protein B) DNA is translated into protein C) DNA is transcribed into RNA, which is translated into protein D) DNA is translated into RNA, which is transcribed into protein

C) DNA is transcribed into RNA, which is translated into protein

In DNA replication, the resulting daughter molecules contain one strand of the original parental DNA and one new strand. What is the explanation for this phenomenon? A) DNA replication is not conservative. B) RNA synthesis is conservative. C) DNA replication is semiconservative. D) DNA replication is conservative.

C) DNA replication is semiconservative.

Which of the following best summarizes the relationship between dehydration reactions and hydrolysis? A) Dehydration reactions eliminate water from membranes; hydrolysis reactions add water to membranes. B) Dehydration reactions and hydrolysis reactions assemble polymers from monomers. C) Dehydration reactions assemble polymers; hydrolysis reactions break polymers apart. D) Hydrolysis reactions create polymers, and dehydration reactions create monomers.

C) Dehydration reactions assemble polymers; hydrolysis reactions break polymers apart.

Which of the following processes occur during meiosis but not mitosis? A) Haploid cells multiply into more haploid cells. B) Haploid cells fuse to form diploid cells. C) Diploid cells form haploid cells. D) A diploid cell combines with a haploid cell.

C) Diploid cells form haploid cells.

What is the correct chromosomal condition for one daughter nucleus at telophase of mitosis? A) D B) B C) E D) C

C) E

Which statement about the beak size of finches on the island of Daphne Major during prolonged drought is true? A) Each bird evolved a deeper, stronger beak as the drought persisted. B) Each bird that survived the drought produced only offspring with deeper, stronger beaks than seen in the previous generation. C) Each bird's survival was strongly influenced by the depth and strength of its beak as the drought persisted. D) The frequency of the strong-beak alleles increased in each bird as the drought persisted.

C) Each bird's survival was strongly influenced by the depth and strength of its beak as the drought persisted.

Refer to the information and figure below to answer the following questions. A certain (hypothetical) organism is diploid, has either blue or orange wings as the consequence of one of its genes on chromosome 12, and has either long or short antennae as the result of a second gene on chromosome 19, as shown in the figure. A female with a paternal set of one orange and one long gene chromosome and a maternal set comprised of one blue and one short gene chromosome would be expected to produce which of the following types of eggs after meiosis? A) All eggs will have maternal types of gene combinations. B) Half the eggs will have maternal and half will have paternal combinations. C) Each egg has a one-fourth chance of having either blue long, blue short, orange long, or orange short combinations. D) All eggs will have paternal types of gene combinations.

C) Each egg has a one-fourth chance of having either blue long, blue short, orange long, or orange short combinations.

A gene for the MN blood group has codominant alleles Mand N. If both children in a family are of blood type M, which of the following situations is possible? A) Each parent must be type M. B) Both children are heterozygous for this gene. C) Each parent is either M or MN. D) Neither parent can have the N allele.

C) Each parent is either M or MN.

________ is a regulatory mechanism in which the end product of a metabolic pathway inhibits an enzyme that catalyzes an early step in the pathway. A) Cooperative inhibition B) Metabolic inhibition C) Feedback inhibition D) Allosteric inhibition

C) Feedback inhibition

Which of the following correctly describes chemical equilibrium? A) Concentrations of products are higher than the concentrations of the reactants. B) There are equal concentrations of reactants and products, and the reactions have stopped. C) Forward and reverse reactions continue with no net effect on the concentrations of the reactants and products. D) There are equal concentrations of products and reactants while forward and reverse reactions continue.

C) Forward and reverse reactions continue with no net effect on the concentrations of the reactants and products.

Which of the following is a correct association? A) kinase activity and the addition of a tyrosine B) phosphodiesterase activity and the removal of phosphate groups C) GTPase activity and hydrolysis of GTP to GDP D) adenylyl cyclase activity and the conversion of cAMP to AMP

C) GTPase activity and hydrolysis of GTP to GDP

Use the following information to answer the question. A plantlike organism on the planet Pandora can have three recessive genetic traits: bluish leaves, due to an allele (a) of geneA; a feathered stem, due to an allele (b) of gene B; and hollow roots due to an allele (c) of gene C. The three genes are linked and recombine. A geneticist performed a testcross with an organism that had been found to be heterozygous for the three recessive traits, and she was able to identify progeny of the following phenotypic distribution (+ = wild type): The greatest distance among the three genes is between a and c. What does this mean? A) Gene c is between a and b. B) Gene a is between b and c. C) Genes are in the order: a-b-c. D) Gene a is not recombining with c.

C) Genes are in the order: a-b-c.

Which of the following produces and modifies polysaccharides that will be secreted? A) lysosome B) mitochondrion C) Golgi apparatus D) peroxisome

C) Golgi apparatus

Which of the following are compounds? A) O2 and CH4 B) H2O, O2, and CH4 C) H2O and CH4, but not O2 D) H2O and O2

C) H2O and CH4, but not O2

Which of the following structural characteristics is most critical for the association between histones and DNA? A) Histones are small proteins. B) There are at least five different histone proteins in every eukaryote. C) Histones are positively charged. D) Histones are highly conserved (that is, histones are very similar in every eukaryote).

C) Histones are positively charged.

Which of these is an example of inductive reasoning? A) If two species are members of the same genus, they are more alike than each of them could be to a different genus. B) If protists are all single-celled, then they are incapable of aggregating. C) Hundreds of individuals of a species have been observed and all are photosynthetic; therefore, the species is photosynthetic. D) These organisms live in sunny regions. Therefore, they are using photosynthesis.

C) Hundreds of individuals of a species have been observed and all are photosynthetic; therefore, the species is photosynthetic.

DNA was isolated from three different cell types of the same organism, the relative DNA content for each type was determined, and the results were plotted on the graph shown in the figure below. Refer to the graph to answer the following questions. A bar graph shows DNA content (n) from 0 to 8. Three samples have the following amounts of DNA: Sample I, 2 units; sample II, 4 units, sample III, 1 unit. Which sample of DNA might represent an animal cell in the G2 phase of the cell cycle prior to meiosis? A) I B) both I and II C) II D) III

C) II

Particular receptor tyrosine kinases (RTKs) that promote excessive cell division are found at high levels in various cancer cells. HER2 is an RTK that is present at excessively high levels in some breast cancer cells. Herceptin is a protein that binds to HER2 and inhibits cell division. Herceptin may be an effective treatment for breast cancer treatment under which of the following conditions? A) If the patient has excessive levels of other RTKs in cancer cells. B) If injection of HER2 in the patient's cancer cells stimulates cell division. C) If the patient's cancer cells have excessive levels of HER2. D) If the patient's genome codes for the HER2.

C) If the patient's cancer cells have excessive levels of HER2.

Which of the following statements correctly describes osmosis? A) In osmosis, solutes move across a membrane from areas of lower water concentration to areas of higher water concentration. B) Osmosis is an energy-demanding or "active" process. C) In osmosis, water moves across a membrane from areas of lower solute concentration to areas of higher solute concentration. D) Osmosis only takes place in red blood cells.

C) In osmosis, water moves across a membrane from areas of lower solute concentration to areas of higher solute concentration.

Which of the following scenarios describes an example of epistasis? A) In cacti, there are several genes for the type of spines. B) Recessive genotypes for each of two genes (aabb) results in an albino corn snake. C) In rabbits and many other mammals, one genotype (ee) prevents any fur color from developing. D) In Drosophila (fruit flies), white eyes can be due to anX-linked gene or to a combination of other genes.

C) In rabbits and many other mammals, one genotype (ee) prevents any fur color from developing.

What is the most likely mechanism by which testosterone functions inside a cell? A) It coordinates a phosphorylation cascade that reduces spermatogenesis. B) It acts as a steroid signal receptor that activates ion channel proteins in the plasma membrane. C) It binds with a receptor protein that enters the nucleus and activates expression of specific genes. D) It acts as a signal receptor that activates tyrosine kinases.

C) It binds with a receptor protein that enters the nucleus and activates expression of specific genes.

How does the enzyme telomerase meet the challenge of replicating the ends of linear chromosomes? A) It causes specific double-strand DNA breaks that result in blunt ends on both strands. B) It adds a single 5' cap structure that resists degradation by nucleases. C) It catalyzes the lengthening of telomeres, compensating for the shortening that could occur during replication without telomerase activity. D) It adds numerous GC pairs, which resist hydrolysis and maintain chromosome integrity.

C) It catalyzes the lengthening of telomeres, compensating for the shortening that could occur during replication without telomerase activity.

If a cell has completed meiosis I and the first cytokinesis, and is just beginning meiosis II, which of the following is an appropriate description of its genetic contents? A) It has one-fourth the DNA and one-half the chromosomes as the parent cell. B) It has half the chromosomes but twice the DNA of the parent cell. C) It has half the amount of DNA as the cell that began meiosis. D) It is genetically identical to another cell formed from the same meiosis I event.

C) It has half the amount of DNA as the cell that began meiosis.

In liver cells, the inner mitochondrial membranes are about five times the area of the outer mitochondrial membranes. What purpose must this serve? A) It allows for an increased rate of the citric acid cycle. B) It increases the surface for substrate-level phosphorylation. C) It increases the surface for oxidative phosphorylation. D) It allows for an increased rate of glycolysis

C) It increases the surface for oxidative phosphorylation.

Which of the following statements about diffusion is true? A) It is an active process in which molecules move from a region of lower concentration to a region of higher concentration. B) It requires an expenditure of energy by the cell. C) It is a passive process in which molecules move from a region of higher concentration to a region of lower concentration. D) It is very rapid over long distances.

C) It is a passive process in which molecules move from a region of higher concentration to a region of lower concentration.

Which of the following statements accurately describes the structure of a eukaryotic chromosome? A) It has different numbers of genes in different cell types of an organism. B) It is constructed as a series of nucleosomes wrapped around two DNA molecules. C) It is a single linear molecule of double-stranded DNA plus proteins. D) It is composed of a single strand of DNA.

C) It is a single linear molecule of double-stranded DNA plus proteins.

Which of the following statements correctly describes a ribozyme? A) It is an enzyme that catalyzes the association between the large and small ribosomal subunits. B) It is a catalyst that uses RNA as a substrate. C) It is an RNA with catalytic activity. D) It is an enzyme that synthesizes RNA as part of the transcription process.

C) It is an RNA with catalytic activity.

Within a double-stranded DNA molecule, adenine forms hydrogen bonds with thymine, and cytosine forms hydrogen bonds with guanine. What is the significance of the structural arrangement? A) It allows variable width of the double helix. B) It determines the type of protein produced. C) It permits complementary base pairing. D) It determines the tertiary structure of a DNA molecule.

C) It permits complementary base pairing.

In the nematode C. elegans, ced-9 prevents apoptosis in a normal cell in which of the following ways? A) Ced-9 remains inactive until it is signaled by ced-3 and other caspases. B) Ced-9 cleaves to produce ced-3 and ced-4. C) It prevents the caspase activity of ced-3 and ced-4. D) Ced-9 prevents blebbing by its action on the cell membrane.

C) It prevents the caspase activity of ced-3 and ced-4.

What is the function of the nuclear pore complex found in eukaryotes? A) It synthesizes secreted proteins. B) It synthesizes the proteins required to copy DNA and make mRNA. C) It regulates the movement of proteins and RNAs into and out of the nucleus. D) It assembles ribosomes from raw materials that are synthesized in the nucleus.

C) It regulates the movement of proteins and RNAs into and out of the nucleus.

Which of the following statements best describes the primary role played by oxygen in cellular respiration? A) It oxidizes glucose to form two molecules of pyruvate. B) It serves as an acceptor for carbon, forming CO2 in the citric acid cycle. C) It serves as the final acceptor for electrons from the electron transport chain. D) It yields energy in the form of ATP as it is passed down the electron transport chain.

C) It serves as the final acceptor for electrons from the electron transport chain.

How does the transcription of structural genes in an inducible operon occur? A) It stops when the pathway's product is present. B) It starts when the pathway's product is present. C) It starts when the pathway's substrate is present. D) It occurs continuously in the cell.

C) It starts when the pathway's substrate is present.

Which of the following statements is true regarding the complexity of biological systems? A) Understanding the chemical structure of DNA reveals how it directs the functioning of a living cell. B) An ecosystem displays complex properties of the biotic component only. C) Knowing the function of a component of a living system can provide insights into the structure and organization of the living system. D) An understanding of the interactions between different components within a living system is an approach towards understanding reductionism.

C) Knowing the function of a component of a living system can provide insights into the structure and organization of the living system.

Recall that in the experiment, caterpillars born in the spring looked like flowers, and caterpillars born in the summer looked like twigs. What is the most likely selective advantage for this difference in body shape? A) Looking like their food sources allows the caterpillars to move through their environment more efficiently. B) Development into the adult moth form is faster for caterpillars shaped like twigs than like flowers. C) Looking like their food source lets the caterpillars blend into their surroundings, reducing predation. D) Looking like their food source will increase the caterpillars' feeding efficiency; this would increase their growth rate and survival rate.

C) Looking like their food source lets the caterpillars blend into their surroundings, reducing predation.

In 1668, Francesco Redi performed a series of experiments on spontaneous generation. He began by putting similar pieces of meat into eight identical jars. Four jars were left open to the air, and four were sealed. He then did the same experiment with one variation: Instead of sealing four of the jars completely, he covered them with gauze (the gauze excluded the flies while allowing the meat to be exposed to air). In both experiments, he monitored the jars and recorded whether or not maggots (young flies) appeared in the meat. What hypothesis was being tested in the initial experiment with open versus sealed jars? A) Spontaneous generation can occur only if meat is exposed to air. B) The type of meat used affects the likelihood of spontaneous generation. C) Maggots do not arise spontaneously, but from eggs laid by adult flies. D) Spontaneous generation is more likely during the long days of summer.

C) Maggots do not arise spontaneously, but from eggs laid by adult flies.

In a metabolic pathway, succinate dehydrogenase catalyzes the conversion of succinate to fumarate. The reaction is inhibited by malonic acid, a substance that resembles succinate but cannot be acted upon by succinate dehydrogenase. Increasing the amount of succinate molecules to those of malonic acid reduces the inhibitory effect of malonic acid. What role does malonic acid play with respect to succinate dehydrogenase? A) Malonic acid is a noncompetitive inhibitor. B) Malonic acid is an allosteric regulator. C) Malonic acid is a competitive inhibitor. D) Malonic acid blocks the binding of fumarate.

C) Malonic acid is a competitive inhibitor.

Who performed classic experiments that supported the semiconservative model of DNA replication? A) Watson and Crick B) Hershey and Chase C) Meselson and Stahl D) Franklin and Wilkins

C) Meselson and Stahl

Which of the following statements is true of metabolism in its entirety in all organisms? A) Metabolism uses all of an organism's resources. B) Metabolism manages the increase of entropy in an organism. C) Metabolism consists of all the energy transformation reactions in an organism. D) Metabolism depends on a constant supply of energy from food.

C) Metabolism consists of all the energy transformation reactions in an organism.

In his transformation experiments, what phenomenon did Griffith observe? A) Infecting mice with nonpathogenic strains of bacteria makes them resistant to pathogenic strains. B) Mice infected with a pathogenic strain of bacteria can spread the infection to other mice. C) Mixing a heat-killed pathogenic strain of bacteria with a living nonpathogenic strain can convert some of the living cells into the pathogenic form. D) Mixing a heat-killed nonpathogenic strain of bacteria with a living pathogenic strain makes the pathogenic strain nonpathogenic.

C) Mixing a heat-killed pathogenic strain of bacteria with a living nonpathogenic strain can convert some of the living cells into the pathogenic form.

The genetic code is redundant. What is meant by this statement? A) The genetic code is different for different domains of organisms. B) The genetic code is universal (the same for all organisms). C) More than one codon can specify the addition of the same amino acid. D) A single codon can specify the addition of more than one amino acid.

C) More than one codon can specify the addition of the same amino acid.

Which of the following statements about the cytoskeleton is true? A) Although microtubules are common within a cell, actin filaments are rarely found outside of the nucleus. B) Chemicals that block the assembly of the cytoskeleton would have little effect on a cell's response to external stimuli. C) Movement of cilia and flagella is the result of motor proteins causing microtubules to move relative to each other. D) The cytoskeleton of eukaryotes is a static structure most resembling scaffolding used at construction sites.

C) Movement of cilia and flagella is the result of motor proteins causing microtubules to move relative to each other.

Why did all of the F1 offspring of Mendel's classic pea cross always look like one of the two parental varieties? A) Each allele affected phenotypic expression. B) The traits blended together during fertilization. C) One allele was dominant. D) No genes interacted to produce the parental phenotype.

C) One allele was dominant.

The figure shows the absorption spectrum for chlorophyll a and the action spectrum for photosynthesis. Why are they different? A) Aerobic bacteria take up oxygen, which changes the measurement of the rate of photosynthesis. B) Green and yellow wavelengths inhibit the absorption of red and blue wavelengths. C) Other pigments absorb light in addition to chlorophyll a. D) Oxygen given off during photosynthesis interferes with the absorption of light.

C) Other pigments absorb light in addition to chlorophyll a.

Paper chromatography is a technique used to separate molecules based upon their size and solubility in a particular solvent. If pigments from a particular species of plant are extracted and subjected to paper chromatography, which of the following results is most likely? A) Paper chromatography would produce a single band of pigment that is characteristic of that particular plant. B) Paper chromatography would separate the pigments into two bands, one that appears blue and one that appears red. C) Paper chromatography would separate the pigments into several bands that appear green or yellow/orange. D) Paper chromatography would separate the pigments into two bands that appear green.

C) Paper chromatography would separate the pigments into several bands that appear green or yellow/orange.

Which of the following statements is true of signal molecules? A) In response to some G protein-mediated signals, a special type of lipid molecule associated with the plasma membrane is cleaved to form IP3 and calcium. B) When signal molecules first bind to receptor tyrosine kinases, the receptors phosphorylate a number of nearby molecules. C) Protein kinase A activation is one possible result of signal molecules binding to G protein-coupled receptors. D) In most cases, signal molecules interact with the cell at the plasma membrane, enter the cell, and eventually enter the nucleus.

C) Protein kinase A activation is one possible result of signal molecules binding to G protein-coupled receptors.

Proton pumps are used in various ways by members of every domain of organisms: Bacteria, Archaea, and Eukarya. What does this fact most probably mean? A) The high concentration of protons in the ancient atmosphere must have necessitated a pump mechanism. B) Cells of each domain evolved proton pumps independently when oceans became more acidic. C) Proton gradients across a membrane were used by cells that were the common ancestor of all three domains of life. D) Proton pumps are necessary to all cell membranes.

C) Proton gradients across a membrane were used by cells that were the common ancestor of all three domains of life.

Which of the following processes occurs during transcription? A) mRNA attaches to ribosomes B) proteins are synthesized C) RNA is synthesized D) DNA is replicated

C) RNA is synthesized

Which of the following statements best describes the termination of transcription in prokaryotes? A) RNA polymerase transcribes through the polyadenylation signal, causing proteins to associate with the transcript and cut it free from the polymerase. B) Once transcription has initiated, RNA polymerase transcribes until it reaches the end of the chromosome. C) RNA polymerase transcribes through the terminator sequence, causing the polymerase to separate from the DNA and release the transcript. D) RNA polymerase transcribes through a stop codon, causing the polymerase to stop advancing through the gene and release the mRNA.

C) RNA polymerase transcribes through the terminator sequence, causing the polymerase to separate from the DNA and release the transcript.

Why is a scientific topic best discussed by people of varying points of view, from different subdisciplines, and representing diverse cultures? A) This is a way of ensuring that everyone gets the same results. B) Scientists can coordinate with others to conduct experiments in similar ways. C) Robust and critical discussion between diverse groups improves scientific thinking. D) People need to exchange their ideas with other disciplines and cultures because everyone has a right to an opinion in science.

C) Robust and critical discussion between diverse groups improves scientific thinking.

Genetic variation leads to genetic diversity in populations and is the raw material for evolution. Biological systems have multiple processes, such as reproduction, that affect genetic variation. They are evolutionarily conserved and shared by various organisms. Which of the following statements best represents the connection between reproduction and evolution? A) Plants that use sexual reproduction are rare since this type of reproduction in plants does not contribute to genetic diversity. B) Since prokaryotic organisms reproduce asexually, there is no mechanism for them to add genetic diversity for evolution. C) Sexual reproduction increases genetic variation because random mutations can be shuffled between organisms. D) In order to increase genetic diversity for evolution in sexually reproducing organisms, mutations must occur in the zygote after fertilization.

C) Sexual reproduction increases genetic variation because random mutations can be shuffled between organisms.

Movement of the chromosomes during anaphase would be most affected by a drug that prevents which of the following events in mitosis and cell division? A) Formation of a cleavage furrow B) Nuclear envelope breakdown C) Shortening of microtubules D) Elongation of microtubules

C) Shortening of microtubules

Which of the following statements is TRUE? A) All life requires the same essential elements. B) A patient suffering from a goiter should not consume seafood. C) Some naturally occurring elements are toxic to organisms. D) Carbon, hydrogen, oxygen, and calcium are the most abundant elements of living matter.

C) Some naturally occurring elements are toxic to organisms.

What is meant by the description "antiparallel" regarding the two strands that make up the DNA double helix? A) Base pairings create unequal spacing between the two DNA strands. B) One strand contains only purines and the other contains only pyrimidines. C) The 5' to 3' direction of one strand runs counter to the 5' to 3' direction of the other strand. D) The double helix structure of DNA creates nonparallel strands.

C) The 5' to 3' direction of one strand runs counter to the 5' to 3' direction of the other strand.

Which statement is true of all atoms that are anions? A) The atom has more protons than electrons. B) The atom has more neutrons than protons. C) The atom has more electrons than protons. D) The atom has fewer protons than does a neutral atom of the same element.

C) The atom has more electrons than protons.

In a healthy eukaryotic cell, the rate of DNA repair is typically equal to the rate of DNA mutation. When the rate of repair lags behind the rate of mutation, what is a possible fate of the cell? A) DNA replication will proceed more quickly. B) DNA replication will continue by a new mechanism. C) The cell can be transformed into a cancerous cell. D) RNA may be used instead of DNA as inheritance material.

C) The cell can be transformed into a cancerous cell.

Which of the following statements describes a cell that is considered to be differentiated? A) The cell loses connections to the surrounding cells. B) The cell appears to be different from the surrounding cells. C) The cell produces proteins specific to a particular cell type. D) The cell replicates by the process of mitosis.

C) The cell produces proteins specific to a particular cell type.

Which of the following statements describes the chromosomal makeup of each daughter cell after telophase of meiosis I? A) The cells are diploid, and the chromosomes are each composed of two chromatids. B) The cells are haploid, and the chromosomes are each composed of a single chromatid. C) The cells are haploid, and the chromosomes are each composed of two chromatids. D) The cells are diploid, and the chromosomes are each composed of a single chromatid.

C) The cells are haploid, and the chromosomes are each composed of two chromatids.

Which of the following statements regarding gene linkage is correct? A) All of the traits that Mendel studied—seed color, pod shape, flower color, and others—are due to genes linked on the same chromosome. B) Linked genes are found on different chromosomes. C) The closer two genes are on a chromosome, the lower the probability that a crossover will occur between them. D) The observed frequency of recombination of two genes that are far apart from each other has a maximum value of 100%.

C) The closer two genes are on a chromosome, the lower the probability that a crossover will occur between them.

he solutions in the arms of a U-tube are separated at the bottom of the tube by a selectively permeable membrane. The membrane is permeable to sodium chloride but not to glucose. Side A is filled with a solution of 0.4 M glucose and 0.5 M sodium chloride (NaCl), and side B is filled with a solution containing 0.8 M glucose and 0.4 M sodium chloride. Initially, the volume in both arms is the same. A U-shaped tube with water-permeable membrane at the center bottom of the U. The left side, A, contains 0.4 M glucose and 0.5 M sodium chloride (NaCl). The right side, B, contains 0.8 M glucose and 0.4 M NaCl. The two sides are level with each other. In the U-tube experiment illustrated above, which of the following statements correctly describes side B at equilibrium? A) The concentration of NaCl and glucose will decrease, and the water level will increase. B) The concentration of NaCl will increase, the concentration of glucose will decrease, and the water level will decrease. C) The concentration of NaCl will increase, the concentration of glucose will decrease, and the water level will increase. D) The concentration of NaCl and glucose will increase, and the water level will decrease.

C) The concentration of NaCl will increase, the concentration of glucose will decrease, and the water level will increase.

Individuals with the disorder xeroderma pigmentosum are hypersensitive to sunlight, and mutations to the DNA in their skin cells are left uncorrected. Why are the mutations not corrected in individuals with this disorder? A) The disorder causes mitosis to stop during metaphase. B) The disorder makes cells unable to replicate DNA. C) The disorder causes cells to be unable to repair thymine dimers. D) The disorder makes cells unable to form chromosomes.

C) The disorder causes cells to be unable to repair thymine dimers.

Exposing inner mitochondrial membranes to ultrasonic vibrations will disrupt the membranes. However, the fragments will reseal "inside out." The little vesicles that result can still transfer electrons from NADH to oxygen and synthesize ATP. Which of the following statements about these inside-out membrane vesicles is true? A) ATP will be produced from ADP and (P)i in the interior of the vesicle. B) The inside of the vesicles will become alkaline when NADH is added. C) The inside of the vesicles will become acidic when NADH is added. D) The vesicles will pump protons out of the interior of the vesicle to the exterior using energy from ATP hydrolysis.

C) The inside of the vesicles will become acidic when NADH is added.

Which of the following statements correctly describes the difference between the leading and the lagging strands of DNA during DNA replication? A) The leading strand is synthesized by adding nucleotides to the 3' end of the growing strand, and the lagging strand is synthesized by adding nucleotides to the 5' end. B) The leading strand is synthesized at twice the rate of the lagging strand. C) The leading strand is synthesized in the same direction as the movement of the replication fork, and the lagging strand is synthesized in the opposite direction. D) The lagging strand is synthesized continuously, whereas the leading strand is synthesized in short fragments that are ultimately stitched together.

C) The leading strand is synthesized in the same direction as the movement of the replication fork, and the lagging strand is synthesized in the opposite direction.

What happens when electrons are passed from one atom to a more electronegative atom? A) The more electronegative atom is oxidized, and energy is consumed. B) The more electronegative atom is oxidized, and energy is released. C) The more electronegative atom is reduced, and energy is released. D) The more electronegative atom is reduced, and energy is consumed.

C) The more electronegative atom is reduced, and energy is released.

When electrons flow along the electron transport chains of mitochondria, which of the following changes occurs? A) The electrons gain free energy. B) ATP synthase pumps protons by active transport. C) The pH of the matrix increases. D) NAD+ is oxidized.

C) The pH of the matrix increases.

A researcher introduces double-stranded RNA into a culture of mammalian cells and can identify its location or that of its smaller subsections experimentally, using a fluorescent probe. When she finds that the introduced strand separates into single-stranded RNAs, what other evidence of this single-stranded RNA piece's activity would she find? A) The cell's translation ability is entirely shut down. B) She can measure the degradation rate of the remaining single strand. C) The rate of accumulation of the polypeptide encoded by the target mRNA is reduced. D) The amount of miRNA is multiplied by its replication.

C) The rate of accumulation of the polypeptide encoded by the target mRNA is reduced.

Which of the following statements accurately describes the differences between DNA replication in prokaryotes and DNA replication in eukaryotes? A) Prokaryotic chromosomes have a single origin of replication, whereas eukaryotic chromosomes have many. B) Prokaryotes produce Okazaki fragments during DNA replication, but eukaryotes do not. C) The rate of elongation during DNA replication is slower in prokaryotes than in eukaryotes. D) Prokaryotic chromosomes have histones, whereas eukaryotic chromosomes do not.

C) The rate of elongation during DNA replication is slower in prokaryotes than in eukaryotes.

If you were to add one of the eight citric acid cycle intermediates to the culture medium of yeast growing in the laboratory, what do you think would happen to the rates of ATP and carbon dioxide production? A) The rate of ATP production would increase, while carbon dioxide production would decrease. B) The rate of ATP production would decrease, while carbon dioxide production would increase. C) The rates of ATP production and carbon dioxide production would both increase. D) The rates of ATP production and carbon dioxide production would both decrease.

C) The rates of ATP production and carbon dioxide production would both increase.

What would be observed by live-cell fluorescence microscopy immediately after HIV entry if HIV is endocytosed first, and then later fuses with the endocytotic vesicle membrane? A) A spot of red fluorescence will remain outside the cell after delivering the viral capsid. B) A spot of red fluorescence will diffuse in the infected cell's cytoplasm. C) The red fluorescent dye-labeled lipids will appear in the infected cell's interior. D) A spot of red fluorescence will be visible on the infected cell's plasma membrane, marking the site of membrane fusion and HIV entry.

C) The red fluorescent dye-labeled lipids will appear in the infected cell's interior.

During meiosis, a defect occurs in a cell that results in the failure of spindle microtubules binding at the kinetochores. Which of the following statements describes the most likely result of such a defect? A) New microtubules with more effective binding capabilities to kinetochores will be synthesized to compensate for the defect. B) Excessive cell divisions will occur resulting in cancerous tumors and an increase in the chromosome numbers known as polyploidy. C) The resulting cells will not receive the correct number of chromosomes in the gametes, a condition known as aneuploidy. D) The defect will be bypassed in order to ensure normal chromosome distribution in the new cells.

C) The resulting cells will not receive the correct number of chromosomes in the gametes, a condition known as aneuploidy.

Hormones are chemical substances produced in one organ that are released into the bloodstream and affect the function of a target organ. Which of the following conditions is required for the target organ to respond to a particular hormone? A) The target organ must be the same as the organ that produced the hormone. B) The target organ must have the opposite mating type of the organ that produced the hormone. C) The target organ must have receptors that recognize and bind the hormone molecule. D) Cells in the target organ must modify their plasma membranes to allow the hormone to enter the cytoplasm.

C) The target organ must have receptors that recognize and bind the hormone molecule.

The solutions in the two arms of this U-tube are separated by a membrane that is permeable to water and glucose but not to sucrose. Side A is half-filled with a solution of 2 M sucrose and 1 M glucose. Side B is half-filled with 1 M sucrose and 2 M glucose. Initially, the liquid levels on both sides are equal. A U-shaped tube with water-permeable membrane at the center bottom of the U. The left side, A, is filled with 2 M sucrose and 1 M glucose. The right side, B, is filled with 1 M sucrose and 2 M glucose. The two sides are level with each other. Which of the following will be true when the system illustrated above reaches equilibrium? A) The concentration of sucrose on side A will be greater than the concentration of sucrose on side B. B) The water levels will be unchanged. C) The water level will be higher in side A than in side B. D) The water level will be higher in side B than in side A.

C) The water level will be higher in side A than in side B.

What would be a likely consequence of a mutation in plants that results in a photorespiration deficiency? A) Cells would carry on the Calvin cycle at a much slower rate. B) Photosynthetic efficiency would be reduced at low light intensities. C) There would be more cellular damage from harmful products of the light reactions of photosynthesis. D) Less oxygen would be produced by photosynthesis.

C) There would be more cellular damage from harmful products of the light reactions of photosynthesis.

Exposure of zebrafish nuclei to cytosol isolated from eggs at metaphase of mitosis resulted in phosphorylation of NEP55 and L68 proteins by cyclin-dependent kinase 2. NEP55 is a protein of the inner nuclear membrane, and L68 is a protein of the nuclear lamina. What is the most likely role of phosphorylation of these proteins in the process of mitosis? A) They assist in the migration of centrosomes to opposite sides of the nucleus. B) They are involved in chromosome condensation. C) They are involved in the disassembly of the nuclear envelope. D) They enable the attachment of the spindle microtubules to kinetochore regions of the centromere.

C) They are involved in the disassembly of the nuclear envelope.

Which of the following statements correctly describes the reason that closely linked genes are typically inherited together? A) The number of genes in a cell is greater than the number of chromosomes. B) Alleles are paired together during meiosis. C) They are located close together on the same chromosome. D) Genes align that way during metaphase I of meiosis.

C) They are located close together on the same chromosome.

Several organisms, primarily protists, have what are called intermediate mitotic organization. These protists are intermediate in what sense? A) They reproduce by binary fission in their early stages of development and by mitosis when they are mature. B) They use mitotic division but only have circular chromosomes. C) They maintain a nuclear envelope during division. D) None of them form spindles.

C) They maintain a nuclear envelope during division.

Which of the following processes do normal proto-oncogenes typically exhibit? A) They normally suppress tumor growth. B) They are underexpressed in cancer cells. C) They stimulate normal cell growth and division. D) They enhance tumor growth.

C) They stimulate normal cell growth and division.

The loss of water from a plant by transpiration cools the leaf. Movement of water in transpiration requires both adhesion to the conducting walls and wood fibers of the plant and cohesion of the molecules to each other. A scientist wanted to increase the rate of transpiration of a crop species to extend its range into warmer climates. The scientist substituted a nonpolar solution with an atomic mass similar to that of water for hydrating the plants. What do you expect the scientist's data will indicate from this experiment? A) The rate of transpiration will be slightly lower with the nonpolar substance as the plant will not have evolved with the nonpolar compound. B) Transpiration rates will increase as nonpolar compounds undergo adhesion and cohesion with wood fibers more readily than water. C) Transpiration rates will fall to zero as nonpolar compounds do not have the properties necessary for adhesion and cohesion. D) The rate of transpiration will be the same for both water and the nonpolar substance.

C) Transpiration rates will fall to zero as nonpolar compounds do not have the properties necessary for adhesion and cohesion.

HIV's genome of RNA includes the code for reverse transcriptase (RT), an enzyme that acts early in infection to synthesize a DNA genome off of an RNA template. The HIV genome also codes for protease (PR), an enzyme that acts later in infection by cutting long viral polyproteins into smaller, functional proteins. Both RT and PR represent potential targets for antiretroviral drugs. Drugs called nucleoside analogs (NA) act against RT, whereas drugs called protease inhibitors (PI) act against PR. Which of the following treatment options would most likely avoid the evolution of drug-resistant HIV (assuming no drug interactions or side effects)? A) Use high doses of NA and a PI at the same time for a period not to exceed one day. B) Use a single PI, but slowly increase the dosage over the course of a week. C) Use moderate doses of NA and two different PIs at the same time for several months. D) Use a series of NAs, one at a time, and change about once a week.

C) Use moderate doses of NA and two different PIs at the same time for several months.

Can the atomic mass of an element vary? A) Yes. Adding or losing protons will change the atomic mass without forming a different element. B) No, it is fixed; otherwise a new element will be formed. C) Yes. Adding or losing neutrons will change the atomic mass without forming a different element. D) Yes. Adding or losing electrons will substantially change the atomic mass.

C) Yes. Adding or losing neutrons will change the atomic mass without forming a different element.

A single base substitution mutation is likely to have a less deleterious effect when the base change exhibits which of the following results? A) an amino acid substitution at the active site of an enzyme B) a stop codon C) a codon that specifies the same amino acid as the original codon D) an amino acid substitution that alters the tertiary structure of the protein

C) a codon that specifies the same amino acid as the original codon

Which of the following statements best describes the characteristics of siRNA? A) a portion of rRNA that allows it to bind to several ribosomal proteins in forming large or small subunits B) a double-stranded RNA that is formed by cleavage of hairpin loops in a larger precursor C) a double-stranded RNA, one of whose strands can complement and inactivate a sequence of mRNA D) a single-stranded RNA that can, where it has internal complementary base pairs, fold into cloverleaf patterns

C) a double-stranded RNA, one of whose strands can complement and inactivate a sequence of mRNA

An earthquake decimates a ground-squirrel population, killing 98% of the squirrels. The surviving population happens to have broader stripes, on average, than the initial population. If broadness of stripes is genetically determined, what effect has the ground-squirrel population experienced during the earthquake? A) disruptive selection B) directional selection C) a genetic bottleneck D) a founder event

C) a genetic bottleneck

For lipids to be fluid at room temperature, they should have ________. A) single bonds only B) a longer carbon chain C) a higher number of cis double bonds D) a higher number of glycerol molecules

C) a higher number of cis double bonds

The partial negative charge at one end of a water molecule is attracted to the partial positive charge of another water molecule. What is this attraction called? A) a van der Waals interaction B) an ionic bond C) a hydrogen bond D) a covalent bond

C) a hydrogen bond

A spaceship is designed to support animal life for a multiyear voyage to the outer planets of the solar system. Plants will be grown to provide oxygen and to recycle carbon dioxide. Since the spaceship will be too far from the sun for photosynthesis, an artificial light source will be needed. What wavelengths of light should be used to maximize plant growth with a minimum of energy expenditure? A) full-spectrum white light B) green light C) a mixture of blue and red light D) UV light

C) a mixture of blue and red light

A newly discovered unicellular organism isolated from acidic mine drainage is found to contain a cell wall, a plasma membrane, two flagella, and peroxisomes. Based just on this information, the organism is most likely ________. A) a motile bacterium B) a motile archaea C) a motile eukaryote D) a nonmotile eukaryote E) a nonmotile prokaryote

C) a motile eukaryote

Which of the following types of mutation would convert a proto-oncogene into an oncogene? A) a mutation that creates an unstable proto-oncogene mRNA B) a mutation that blocks transcription of the proto-oncogene C) a mutation that greatly increases the amount of the proto-oncogene protein D) a deletion of most of the proto-oncogene coding sequence

C) a mutation that greatly increases the amount of the proto-oncogene protein

Which of the following descriptions best fits the class of molecules known as nucleotides? A) a nitrogenous base and a sugar B) a nitrogenous base and a phosphate group C) a nitrogenous base, a phosphate group, and a sugar D) a sugar and a purine or pyrimidine

C) a nitrogenous base, a phosphate group, and a sugar

The most commonly occurring mutation in people with cystic fibrosis is a deletion of a single codon. What is the result of this type of mutation? A) a frameshift mutation B) a base-pair substitution C) a polypeptide missing an amino acid D) a nonsense mutation

C) a polypeptide missing an amino acid

A human cell containing 22 autosomes and a Y chromosome is A) an egg. B) a zygote. C) a sperm. D) a somatic cell of a male.

C) a sperm.

What is the name of the thermodynamic barrier that must be overcome before products are formed in a spontaneous reaction? A) the equilibrium point B) entropy C) activation energy D) free energy

C) activation energy

In humans, clear gender differentiation occurs not at fertilization, but after the second month of gestation. Which of the following statements describes the first event of this differentiation? A) formation of estrogens in female embryos B) formation of testosterone in male embryos C) activation of SRY in male embryos and masculinization of the gonads D) activation of SRY in females and feminization of the gonads

C) activation of SRY in male embryos and masculinization of the gonads

All female mammals have one active X chromosome per cell instead of two. What causes this to happen? A) activation of the BARR gene on one X chromosome, which then becomes inactive B) attachment of methyl (-CH3) groups to the X chromosome that will remain active C) activation of the XIST gene on the X chromosome that will become the Barr body D) inactivation of the XIST gene on the X chromosome derived from the male parent

C) activation of the XIST gene on the X chromosome that will become the Barr body

High levels of citric acid inhibit the enzyme phosphofructokinase, a key enzyme in glycolysis. Citric acid binds to the enzyme at a different location from the active site. This is an example of ________. A) positive feedback regulation B) competitive inhibition C) allosteric regulation D) the specificity of enzymes for their substrates

C) allosteric regulation

Elements found on the left side of the periodic table contain outer shells that are ________; these elements tend to form ________ in solution. A) almost full; cations B) almost empty; anions C) almost empty; cations D) almost full; anions

C) almost empty; cations

An obstetrician knows that one of her patients is a pregnant woman whose fetus is at risk for a serious disorder that is detectable biochemically in fetal cells. The obstetrician would most reasonably offer which of the following procedures to her patient? A) X-ray B) blood transfusion C) amniocentesis or CVS D) karyotyping of the woman's somatic cells

C) amniocentesis or CVS

Which of the following molecules is most similar in structure to ATP? A) an amino acid with three phosphate groups attached B) a DNA nucleotide C) an RNA nucleotide D) a pentose sugar

C) an RNA nucleotide

Which of the following types of reactions would decrease the entropy within a cell? A) catabolic reactions B) hydrolysis C) anabolic reactions D) digestion

C) anabolic reactions

Mendel's observation of the segregation of alleles in gamete formation has its basis in which of the following phases of cell division? A) metaphase II of meiosis B) anaphase II of meiosis C) anaphase I of meiosis D) prophase I of meiosis

C) anaphase I of meiosis

Which of the following molecules dramatically increases the rate of diffusion of water across cell membranes? A) gated ion channels B) the sodium-potassium pump C) aquaporins D) ATP

C) aquaporins

Thalidomide and L-dopa (see figure) are examples of pharmaceutical drugs that occur as enantiomers, or molecules that ________. A) are cis-trans isomers B) have identical three-dimensional shapes C) are mirror images of one another D) are mirror images of one another and have the same biological activity

C) are mirror images of one another

Which of the following is NOT true? Both chloroplasts and mitochondria _____. A) have their own DNA B) have multiple membranes C) are part of the endomembrane system D) are capable of reproducing themselves

C) are part of the endomembrane system

Many G protein-coupled receptors contain seven transmembrane α-helical domains. The amino end of the protein lies at the exterior of the plasma membrane. Loops of amino acids connect the helices either at the exterior surface or on the cytosolic surface of the membrane. The loop on the cytosolic side between helices 5 and 6 is usually substantially longer than the others. Where would you expect to find the carboxyl end of the protein? A) connected with the loop at H5 and H6 B) at the exterior surface C) at the cytosolic surface D) embedded in the phospholipid bilayer of the membrane

C) at the cytosolic surface

Many G protein-coupled receptors contain seven transmembrane α-helical domains. The amino end of the protein lies at the exterior of the plasma membrane. Loops of amino acids connect the helices either at the exterior surface or on the cytosolic surface of the membrane. The loop on the cytosolic side between helices 5 and 6 is usually substantially longer than the others. Where would a coupled G protein most likely interact with this receptor? A) at the exterior surface B) at the cytosolic surface C) connected with the loop at H5 and H6 D) embedded in the phospholipid bilayer of the membrane

C) at the loop between H5 and H6

The left to right order of elements in the periodic table is based on their ________. A) the number of neutrons B) electric charge of the atom C) atomic number D) atomic mass

C) atomic number

In the cells of many eukaryotic species, the nuclear envelope has to disappear to permit which of the following events in the cell cycle? A) condensation of the chromosomes B) separation of the centrosomes C) attachment of microtubules to kinetochores D) DNA synthesis

C) attachment of microtubules to kinetochores

Use the following information to answer the question. In a series of mapping experiments, the recombination frequencies for four different linked genes of Drosophila were determined as shown in the figure. Based on this information, what is the order of these genes on a chromosome map? A) n-rb-b-vg B) rb-cn-vg-b C) b-rb-cn-vg D) vg-cn-b-rb

C) b-rb-cn-vg

A flask containing photosynthetic green algae and a control flask containing water with no algae are both placed under a bank of lights that is set to cycle between 12 hours of light and 12 hours of dark. The dissolved oxygen concentrations in both flasks are monitored. Predict what the relative dissolved oxygen concentrations will be in the flask with algae compared to the control flask. The dissolved oxygen in the flask with algae will ________. A) be higher in the light, but the same in the dark B) always be lower C) be higher in the light, but lower in the dark D) always be higher

C) be higher in the light, but lower in the dark

A researcher introduces double-stranded RNA into a culture of mammalian cells and can identify its location or that of its smaller subsections experimentally, using a fluorescent probe. Some time later, she finds that the introduced strand separates into single-stranded RNAs, one of which is degraded. What does this enable the remaining strand to do? A) attach to histones in the chromatin B) bind to noncomplementary RNA sequences C) bind to complementary regions of target mRNAs D) activate other siRNAs in the cell

C) bind to complementary regions of target mRNAs

Which of the following statements describes an example of cooperativity associated with enzyme regulation? A) binding of an ATP molecule along with another substrate molecule in the active site of the enzyme B) one enzyme in a metabolic pathway passing its product to act as a substrate for the next enzyme in the pathway C) binding a substrate to one subunit of a tetramer stimulates faster binding of substrate to each of the other three subunits D) binding of the end product of a metabolic pathway to the first enzyme in the pathway to inhibit the enzyme

C) binding a substrate to one subunit of a tetramer stimulates faster binding of substrate to each of the other three subunits

Starch is composed of ________. A) unbranched amylopectin and unbranched amylose B) branched amylopectin and branched amylose C) branched amylopectin and unbranched amylose D) unbranched amylopectin and branched amylose

C) branched amylopectin and unbranched amylose

Caffeine is an inhibitor of phosphodiesterase. Therefore, the cells of a person who has recently consumed coffee would have increased levels of which of the following molecules? A) adenylyl cyclase B) activated G proteins C) cAMP D) phosphorylated proteins

C) cAMP

Which two functional groups are always found in amino acids? A) hydroxyl and carboxyl groups B) amino and sulfhydryl groups C) carboxyl and amino groups D) carbonyl and amino groups

C) carboxyl and amino groups

Which parts of the amino acids X and Y are involved in the formation of a peptide bond? X—Y A) amino group of X and carboxyl group of Y B) carboxyl group of X and side chain of Y C) carboxyl group of X and amino group of Y D) side chains of both X and Y

C) carboxyl group of X and amino group of Y

An animal cell lacking carbohydrates on the external surface of its plasma membrane would likely be impaired in which function? A) transporting ions against an electrochemical gradient B) establishing a diffusion barrier to charged molecules C) cell-cell recognition D) attaching the plasma membrane to the cytoskeleton

C) cell-cell recognition

Which organelle or structure is absent in plant cells? A) mitochondria B) microtubules C) centrosomes D) peroxisomes

C) centrosomes

Microevolutions occur when ________. A) individuals within all species vary in their phenotypic traits B) gene flow evenly transfers alleles between populations C) changes in allele frequencies in a population occur over generations D) a bird has a beak of a particular size that does not grow larger during a drought

C) changes in allele frequencies in a population occur over generations

Which structure below is independent/not part of the endomembrane system? A) nuclear envelope B) Golgi apparatus C) chloroplast D) plasma membrane

C) chloroplast

The drug cytochalasin B blocks the function of actin. Which of the following aspects of the animal cell cycle would be most disrupted by cytochalasin B? A) spindle attachment to kinetochores B) movement of chromosomes to the poles during anaphase C) cleavage furrow formation and cytokinesis D) spindle formation

C) cleavage furrow formation and cytokinesis

One difference between cancer cells and normal cells is that cancer cells A) are arrested at the S phase of the cell cycle. B) cannot function properly because they are affected by density-dependent inhibition. C) continue to divide even when they are tightly packed together. D) are unable to synthesize DNA.

C) continue to divide even when they are tightly packed together.

Which of the following molecules are involved in pattern formation during development? A) cAMP B) miRNAs C) cytoplasmic determinants D) transcription factors

C) cytoplasmic determinants

Based on Figure 7.18, which of these experimental treatments would increase the rate of sucrose transport into a plant cell? A) decreasing extracellular sucrose concentration B) decreasing cytoplasmic pH C) decreasing extracellular pH D) adding a substance that makes the membrane more permeable to hydrogen ions

C) decreasing extracellular pH

The elongation of the leading strand during DNA synthesis A) progresses away from the replication fork. B) occurs in the 3′ → 5′ direction. C) depends on the action of DNA polymerase. D) produces Okazaki fragments.

C) depends on the action of DNA polymerase.

Which of the following descriptions illustrates phenotype variation caused by environment? A) variation in horse coat color B) inheritance of body builder "physique" C) diet of caterpillars changes their morphology D) average beak depth during drought

C) diet of caterpillars changes their morphology

The individual with genotype AaBbCCDdEE can make many kinds of gametes. Which of the following correctly describes why this situation is possible? A) recurrent mutations form new alleles B) crossing over during prophase I leads to genetic variety C) different possible assortment of chromosomes into gametes occurs D) there is a tendency for dominant alleles to segregate together

C) different possible assortment of chromosomes into gametes occurs

Three-spined stickleback fish (Gasterosteus aculeatus) show substantial heritable variation in gill-raker length related to differences in their diets. Longer gill rakers appear to function better for capturing open-water prey, while shorter gill rakers function better for capturing shallow-water prey. Which of the following types of selection is most likely to be found in a large lake (open water in the middle and shallow water around the sides) with a high density of these fish? A) sexual selection B) stabilizing selection C) disruptive selection D) directional selection

C) disruptive selection

Rate of an enzyme-catalyzed reaction as a function of varying reactant concentration, with the concentration of enzyme constant. For the enzyme-catalyzed reaction shown in the figure, if the initial reactant concentration is 1.0 micromolar, which of these treatments will cause the greatest increase in the rate of the reaction? A) increasing the concentration of reactants to 10.0 micro molar, while reducing the concentration of enzyme by 1/2 B) cooling the reaction by 10C C) doubling the enzyme concentration D) doubling the activation energy needed

C) doubling the enzyme concentration

During which of the following processes do sister chromatids separate from each other? A) during meiosis II only B) during both mitosis and meiosis I C) during both mitosis and meiosis II D) during meiosis I only

C) during both mitosis and meiosis II

According to the fluid mosaic model of membrane structure, proteins of the membrane are mostly... A) randomly oriented in the membrane, with no fixed inside-outside polarity. B) confined to the hydrophobic interior of the membrane. C) embedded in a lipid bilayer. D) spread in a continuous layer over the inner and outer surfaces of the membrane.

C) embedded in a lipid bilayer.

A chemical reaction that has a positive ΔG is best described as ________. A) enthalpic B) exergonic C) endergonic D) spontaneous

C) endergonic

In chemiosmosis, what is the most direct source of energy that is used to convert ADP + (P)i to ATP? A) energy released as electrons flow through the electron transport chain B) energy released from substrate-level phosphorylation C) energy released from movement of protons through ATP synthase, down their electrochemical gradient D) energy released as electrons are transported across the inner mitochondrial membrane

C) energy released from movement of protons through ATP synthase, down their electrochemical gradient

A characteristic 9 + 2 arrangement of microtubules, consisting of nine doublets of microtubules surrounding a pair of single microtubules is associated with ________. A) bacterial flagella B) eukaryotic flagella, motile cilia, and nonmotile cilia C) eukaryotic flagella and motile cilia D) centrioles and basal bodies

C) eukaryotic flagella and motile cilia

CAM plants keep stomata closed in the daytime, thus reducing loss of water. They can do this because they ________. A) use photosystem I and photosystem II at night B) fix CO2 into sugars in the bundle-sheath cells C) fix CO2 into organic acids during the night D) fix CO2 into pyruvate in the mesophyll cells

C) fix CO2 into organic acids during the night

An original section of DNA has the base sequence AGCGTTACCGT. A mutation in this DNA strand results in the base sequence AGGCGTTACCGT. What type of mutation does this change represent? A) a silent mutation B) a missense mutation C) frameshift mutation D) a point mutation

C) frameshift mutation

Which of the following characteristics of eukaryotic telomeres cause them to replicate differently than the rest of the chromosome? A) DNA polymerase that cannot replicate the leading strand template to its 5' end B) the activity of telomerase enzyme C) gaps left at the 5' end of the lagging strand template D) gaps left at the 3' end of the lagging strand because of the need for a primer

C) gaps left at the 5' end of the lagging strand template

Which Hardy-Weinberg condition is affected by population size? A) selection B) gene flow C) genetic drift D) no mutation

C) genetic drift

Which of the following is not a polymer? A) DNA B) RNA C) glucose D) starch

C) glucose

The enzyme amylase can break glycosidic linkages between glucose monomers only if the monomers are in the form. Which of the following could amylase break down? A) starch, chitin, and cellulose B) glycogen and cellulose C) glycogen, starch, and amylopectin D) cellulose and chitin

C) glycogen, starch, and amylopectin

A man who is a dwarf with achondroplasia and normal vision marries a color-blind woman of normal height. The man's father was six feet tall, and both the woman's parents were of average height. Dwarfism caused by achondroplasia is autosomal dominant, and red-green color blindness is X-linked recessive. What proportion of their sons would be color blind and of normal height? A) one out of four B) all C) half D) none

C) half

Liquid water ________. A) is nonpolar B) has a specific heat lower than that of most other substances C) has a heat of vaporization higher than that of most other substances D) is less dense than ice

C) has a heat of vaporization higher than that of most other substances

In the formula for determining a population's genotype frequencies, the "2" in the term 2pq is necessary because ________. A) heterozygotes have two alleles B) the population is doubling in number C) heterozygotes can come about in two ways D) the population is diploid

C) heterozygotes can come about in two ways

Use the figure to answer the question. What type of bonding is responsible for maintaining the shape of the tRNA molecule shown in the figure? A) van der Waals interactions between hydrogen atoms B) peptide bonding between amino acids C) hydrogen bonding between base pairs D) ionic bonding between phosphates

C) hydrogen bonding between base pairs

Which of the following is broken when water evaporates? A) ionic bonds B) nonpolar covalent bonds C) hydrogen bonds D) polar covalent bonds

C) hydrogen bonds

Even though plants cells carry out photosynthesis, they still use their mitochondria for oxidation of pyruvate. Under what conditions will plant cell mitochondria be active in this process? A) in photosynthesizing cells in the light and in other tissues in the dark B) in photosynthetic cells in the light, while photosynthesis occurs concurrently C) in all cells, with or without light D) in nonphotosynthesizing cells only

C) in all cells, with or without light

In autotrophic bacteria, where is chlorophyll located? A) in chloroplast membranes B) in the cell wall C) in infolded regions of the plasma membrane D) in the nucleoid

C) in infolded regions of the plasma membrane

If plant cells are grown on media containing radioactively labeled thymine for one generation, radioactively labeled macromolecules will be detected in which of the following? A) only in the nucleus and mitochondria B) only in the nucleus C) in the nucleus, mitochondria, and chloroplasts D) only in the nucleus and chloroplasts

C) in the nucleus, mitochondria, and chloroplasts

In a plant cell, DNA may be found _____. A) only in the nucleus B) only in the nucleus and chloroplasts C) in the nucleus, mitochondria, and chloroplasts D) in the nucleus, mitochondria, chloroplasts, and peroxisomes

C) in the nucleus, mitochondria, and chloroplasts

A researcher found a method she could use to manipulate and quantify phosphorylation and methylation in embryonic cells in culture. One of her colleagues suggested she try increased methylation of cytosine (C) nucleotides in the DNA of promoters of a mammalian system. Which of the following results would she most likely see? A) activation of histone tails for enzymatic function B) higher levels of transcription of certain genes C) inactivation of the selected genes D) decreased chromatin condensation

C) inactivation of the selected genes

If a pharmaceutical company wished to design a drug to maintain low blood sugar levels, one approach might be to design a compound that does which of the following? A) stimulates G protein activity in liver cells B) activates adenylyl cyclase in liver cells C) increases phosphodiesterase activity in liver cells D) increases glycogen phosphorylase activity in liver cells

C) increases phosphodiesterase activity in liver cells

The membranes of winter wheat are able to remain fluid when it is extremely cold by ________. A) decreasing the number of hydrophobic proteins in the membrane B) increasing the proportion of glycolipids in the membrane C) increasing the proportion of unsaturated phospholipids in the membrane D decreasing the percentage of cholesterol molecules in the membrane

C) increasing the proportion of unsaturated phospholipids in the membrane

Lipid-soluble signaling molecules, such as aldosterone, cross the membranes of all cells but affect only target cells because... A. only target cells have enzymes that break down aldosterone. B. only in target cells is testosterone able to initiate the phosphorylation cascade leading to activated transcription factor. C. intracellular receptors are present only in target cells. D. only target cells retain the appropriate DNA segments.

C) intracellular receptors are present only in target cells.

The label on a container of margarine lists "hydrogenated vegetable oil" as the major ingredient. Hydrogenated vegetable oil ________. A) has more "kinks" in the fatty acid chains B) has fewer trans fatty acids C) is solid at room temperature D) is less likely to clog arteries

C) is solid at room temperature

The sodium-potassium pump is called an electrogenic pump because it ________. A) generates voltage across the membrane B) decreases the voltage difference across the membrane C) is used to drive the transport of glucose against a concentration gradient D) pumps equal quantities of Na+ and K+ across the membrane in opposite directions

C) is used to drive the transport of glucose against a concentration gradient

What are scaffolding proteins? A) proteins that enter the nucleus of a cell to regulate transcription B) relay proteins that orient receptors and their ligands in appropriate directions to facilitate complex formation C) large molecules to which several relay proteins attach to facilitate cascade effects D) microtubule arrays that allow lipid-soluble hormones to get from the cell membrane to the nuclear pores

C) large molecules to which several relay proteins attach to facilitate cascade effects

The experimental approach taken in current biological investigations presumes that ________. A) a life force ultimately controls the activities of living organisms, and this life force cannot be studied by physical or chemical methods B) simple organic compounds can be synthesized in the laboratory from inorganic precursors, but complex organic compounds like carbohydrates and proteins can be synthesized only by living organisms C) living organisms can be understood in terms of the same physical and chemical laws that can be used to explain all natural phenomena D) living organisms are composed of the same elements present in nonliving things, plus a few special trace elements found only in living organisms or their products

C) living organisms can be understood in terms of the same physical and chemical laws that can be used to explain all natural phenomena

In a chemical reaction, the element 13Al will most preferably ________. A) lose five electrons and become positively charged B) gain five electrons and become negatively charged C) lose three electrons and become positively charged D) gain three electrons and become positively charged

C) lose three electrons and become positively charged

Apoptosis involves all but which of the following? A) cell-signaling pathways B) fragmentation of the DNA C) lysis of the cell D) digestion of cellular contents by scavenger cells

C) lysis of the cell

Which of the following molecules are required for the process of translation? A) mRNA, tRNA, and DNA B) mRNA, DNA, and rRNA C) mRNA, tRNA, and rRNA D) mRNA, tRNA, DNA, and rRNA

C) mRNA, tRNA, and rRNA

The voltage across a membrane is called the ________. A) osmotic potential B) chemical gradient C) membrane potential D) electrochemical gradient

C) membrane potential

Energy released by the electron transport chain is used to pump H+ ions into which location in eukaryotic cells? A) cytoplasm adjacent to the mitochondrial outer membrane B) mitochondrial matrix C) mitochondrial intermembrane space D) mitochondrial inner membrane

C) mitochondrial intermembrane space

Researchers investigating the mechanism of vesicular transport assembled a cell-free system that included microtubule tracks, vesicles, and ATP. However, they observed no movement of transport of vesicles in this system. What were they missing? A) contractile microfilaments B) an axonfv C) motor proteins D) intermediate filaments

C) motor proteins

Which of the following membrane activities requires energy from ATP hydrolysis? A) movement of glucose molecules into a bacterial cell from a medium containing a higher concentration of glucose than inside the cell B) movement of carbon dioxide out of a paramecium C) movement of Na+ ions from a lower concentration in a mammalian cell to a higher concentration in the extracellular fluid D) facilitated diffusion of chloride ions across the membrane through a chloride channel

C) movement of Na+ ions from a lower concentration in a mammalian cell to a higher concentration in the extracellular fluid

When your body temperature rises on a hot day, the neural and hormonal mechanisms activate sweating. Evaporation of sweat leads to cooling of the body surface. This is an example of ________. A) chemical cycling B) emergent properties C) negative feedback regulation D) positive feedback regulation

C) negative feedback regulation

A solution contains 0.0000001 (10-7) moles of hydrogen ions [H+] per liter. Which of the following best describes this solution? A) acidic: H+ donor B) acidic: H+ acceptor C) neutral D) basic: H+ acceptor

C) neutral

Hydrophobic substances such as vegetable oil are ________. A) polar substances that repel water molecules B) nonpolar substances that have an attraction for water molecules C) nonpolar substances that repel water molecules D) polar substances that have an affinity for water

C) nonpolar substances that repel water molecules

Absence of bicoid mRNA from a Drosophila egg leads to the absence of anterior larval body parts and mirror-image duplication of posterior parts. This is evidence that the product of the bicoid gene A) is a protein present in all head structures. B) is transcribed in the early embryo. C) normally leads to formation of head structures. D) normally leads to formation of tail structures.

C) normally leads to formation of head structures.

Which of the following lists represents the order of increasingly higher levels of organization of chromatin? A) nucleosome, looped domain, 30-nm chromatin fiber B) 30-nm chromatin fiber, nucleosome, looped domain C) nucleosome, 30-nm chromatin fiber, looped domain D) looped domain, 30-nm chromatin fiber, nucleosome

C) nucleosome, 30-nm chromatin fiber, looped domain

The relation between amino acid and polypeptide is similar to the relation between ________. A) triglycerides and steroids B) glycogen and glucose C) nucleotide and nucleic acid D) phospholipid and plasma membrane

C) nucleotide and nucleic acid

We can be sure that a mole of table sugar and a mole of vitamin C are equal in their A) volume. B) mass. C) number of molecules. D) number of atoms.

C) number of molecules.

Use the figure to answer the question. Refer to the metabolic pathway illustrated. If A, B, and C are all required for growth, a strain mutant for the gene encoding enzyme B would be able to grow on medium supplemented with which of the following nutrient(s)? A) nutrient A only B) nutrient B only C) nutrient C only D) nutrients A and C

C) nutrient C only

In receptor-mediated endocytosis, receptor molecules initially project to the outside of the cell. Where do they end up after endocytosis? A) on the inside surface of the vesicle B) on the outer surface of the nucleus C) on the outside of vesicles D) on the inside surface of the cell membrane

C) on the outside of vesicles

Which of the following statements describes a primary function of both alcohol fermentation and lactic acid fermentation? A) hydrolysis of ATP to ADP + (P)i B) reduction of NAD+ to NADH C) oxidation of NADH to NAD+ D) reduction of FAD to FADH2

C) oxidation of NADH to NAD+

Carbon dioxide (CO2) is released during which of the following stages of cellular respiration? A) glycolysis and the oxidation of pyruvate to acetyl CoA B) oxidative phosphorylation and fermentation C) oxidation of pyruvate to acetyl CoA and the citric acid cycle D) fermentation and glycolysis

C) oxidation of pyruvate to acetyl CoA and the citric acid cycle

The mechanism of photophosphorylation is most similar to which of the following processes? A) the Calvin cycle B) substrate-level phosphorylation in glycolysis C) oxidative phosphorylation in cellular respiration D) reduction of NADP+

C) oxidative phosphorylation in cellular respiration

In a single molecule of water, two hydrogen atoms are bonded to a single oxygen atom by ________. A) ionic bonds B) nonpolar covalent bonds C) polar covalent bonds D) hydrogen bonds

C) polar covalent bonds

At which phase of the cell cycle do centrioles begin to move apart in animal cells? A) anaphase B) metaphase C) prophase D) telophase

C) prophase

In mitochondria, exergonic redox reactions A) are the source of energy driving prokaryotic ATP synthesis. B) are coupled via phosphorylated intermediates to endergonic processes. C) provide the energy that establishes the proton gradient. D) reduce carbon atoms to carbon dioxide

C) provide the energy that establishes the proton gradient.

When a molecule of NAD+ (nicotinamide adenine dinucleotide) gains a hydrogen atom (not a proton), the molecule becomes ________. A) dehydrogenated B) oxidized C) reduced D) redoxed

C) reduced

To understand the chemical basis of inheritance, we must understand the molecular structure of DNA. This is an example of the application of which concept to the study of biology? A) evolution B) feedback regulation C) reductionism D) emergent properties

C) reductionism

The chemical reaction illustrated... A) joins two fatty acids together B) is a hydrolysis reaction C) results in a peptide bond D) links two polymers to form a monomer

C) results in a peptide bond

Elements 72Zn, 75As, and 74Ge have the ________. A) same number of protons and electrons B) same number of protons C) same number of neutrons D) same number of neutrons and electrons

C) same number of neutrons

Transcription in eukaryotes requires which of the following molecules in addition to RNA polymerase? A) aminoacyl-tRNA synthetase B) anticodons C) several transcription factors D) ribosomes and tRNA

C) several transcription factors

Meiosis II is similar to mitosis in that A) the daughter cells are diploid. B) DNA replicates before the division. C) sister chromatids separate during anaphase. D) homologous chromosomes synapse.

C) sister chromatids separate during anaphase.

The liver is involved in detoxification of many poisons and drugs. Which of the following structures is primarily involved in this process and, therefore, abundant in liver cells? A) nuclear envelope B) rough endoplasmic reticulum C) smooth endoplasmic reticulum D) Golgi apparatus

C) smooth endoplasmic reticulum

One of the buffers that contribute to pH stability in human blood is carbonic acid (H2CO3). Carbonic acid is a weak acid that, when placed in an aqueous solution, dissociates into a bicarbonate ion (HCO3-) and a hydrogen ion (H+). (See figure.) If the pH of blood drops, one would expect ________. A) the concentration of bicarbonate ions (HCO3-) to increase B) a decrease in the concentration of H2CO3 and an increase in the concentration of HCO3- C) the HCO3- to act as a base and remove excess H+ by the formation of H2CO3 D) the HCO3- to act as an acid and remove excess H+ by the formation of H2CO3

C) the HCO3- to act as a base and remove excess H+ by the formation of H2CO3

Whenever diploid populations are in Hardy-Weinberg equilibrium at a particular locus, ________. A) individuals within the population are evolving B) two alleles are present in equal proportions C) the allele's frequency should not change from one generation to the next D) natural selection, gene flow, and genetic drift are acting equally to change an allele's frequency

C) the allele's frequency should not change from one generation to the next

Most CO2 from catabolism is released during... A) electron transport B) glycolysis C) the citric acid cycle D) lactate fermentation

C) the citric acid cycle.

Which of the following statements defines a genome? A) a karyotype B) the complete set of a species' polypeptides C) the complete set of an organism's genes and other DNA sequences D) the complete set of an organism's polypeptides

C) the complete set of an organism's genes and other DNA sequences

In bacteria, DNA will be found in ________. A) mitochondria B) ribosomes C) the nucleoid D) a membrane-enclosed nucleus

C) the nucleoid

How does the primary transcript in the nucleus of a eukaryotic cell compare to the functional mRNA? A) the primary transcript is the same size as the mRNA B) the primary transcript is smaller than the mRNA C) the primary transcript is larger than the mRNA D) both the primary transcript and mRNA contain introns

C) the primary transcript is larger than the mRNA

Which of the following statements best describes the addition rule of probability? A) the probability that two or more independent events will occur simultaneously B) the likelihood that a trait is due to two or more meiotic events C) the probability that either one of two independent events will occur D) the probability of producing two or more heterozygous offspring

C) the probability that either one of two independent events will occur

Which of the following is an example of post-transcriptional control of gene expression? A) the addition of methyl groups to cytosine bases of DNA B) gene amplification contributing to cancer C) the removal of introns and alternative splicing of exons D) the binding of transcription factors to a promoter

C) the removal of introns and alternative splicing of exons

Some bacteria are metabolically active in hot springs because A) they are able to maintain a lower internal temperature. B) high temperatures make catalysis unnecessary. C) their enzymes have high optimal temperatures. D) their enzymes are completely insensitive to temperature.

C) their enzymes have high optimal temperatures.

People who are lactose intolerant cannot extract energy from milk because ________. A) lactose is too big to be digested by the enzymes B) they are missing the bacteria that can digest lactose C) they are missing an enzyme D) milk is fermented to a by-product, which cannot be digested

C) they are missing an enzyme

The functioning of enhancers is an example of A) a eukaryotic equivalent of prokaryotic promoter functioning. B) post-translational control that activates certain proteins. C) transcriptional control of gene expression. D) the stimulation of translation by initiation factors.

C) transcriptional control of gene expression.

Somatic cells of roundworms have four individual chromosomes per cell. How many chromosomes would you expect to find in an ovum from a roundworm? A) eight B) four C) two D) a diploid number

C) two

Which of the following is a major difference between RNA and DNA? A) type of purines B) type of phosphate C) type of sugar D) type of glycosidic bond

C) type of sugar

The following question refers to the pedigree chart in the figure for a family, some of whose members exhibit the dominant trait, W. Affected individuals are indicated by a dark square or circle. What is the genotype of individual II-5? A) WW B) ww or Ww C) ww D) Ww

C) ww

If we continue to follow the cell lineage from question 4, then the DNA content of a single cell at metaphase of meiosis II will be A) 2x. B) 0.5x. C) x. D) 0.25x.

C) x.

Feather color in budgies is determined by two different genes, Y for pigment on the outside of the feather, and Bfor pigment on the inside of the feather. YYBB, YyBB, orYYBb is green; yyBB or yyBb is blue; YYbb or Yybb is yellow; and yybb is white. Two blue budgies were crossed. Over the years, they produced 22 offspring, five of which were white. What are the most likely genotypes for the two blue budgies? A) yyBB and yyBb B) yyBB and yyBB C) yyBb and yyBb D) yyBb and yybb

C) yyBb and yyBb

Starting with one molecule of glucose, glycolysis results in the net production of which of the following sets of energy-containing products? A. 6 CO2, 2 pyruvate, and 2 ATP B. 2 NAD+, 2 pyruvate, and 2 ATP C. 2 NADH, 2 pyruvate, and 2 ATP D. 4 NADH, 2 pyruvate, and 4 ATP

C. 2 NADH, 2 pyruvate, and 2 ATP

Motor proteins require which of the following structures of molecules to function in the movement of chromosomes towards the poles of the mitotic spindle? A. Intact cohesin B. Intact centromeres C. ATP as an energy source D. a microtubule-organizing center

C. ATP as an energy source

Which of the following graphs most likely describes the effect of pH on the function of the enzyme catalase in human cells? Note: The x-axis is pH and the y-axis is enzyme activity.

D)

In a Hardy-Weinberg population with two alleles, A and a, that are in equilibrium, the frequency of the allele a is 0.3. What is the frequency of individuals that are homozygous for this allele? A) 9.0 B) 0.9 C) 0.49 D) 0.09

D) 0.09

In a Hardy-Weinberg population with two alleles, A and a, that are in equilibrium, the frequency of allele a is 0.1. What is the frequency of individuals with AA genotype? A) 0.42 B) 0.20 C) 0.32 D) 0.81

D) 0.81

The following question refers to the pedigree chart in the figure for a family, some of whose members exhibit the dominant trait, W. Affected individuals are indicated by a dark square or circle. 1 ww white square with 2 Ww black circle. II: offspring: 2 black square; 3 black circle, 4 black square III: 1 black square, offspring of II 2 and white circle. IV: offspring of II 4 and white circle: 3 black square mates with 4 white circle. What is the probability that individual III-1 is Ww? A) 1/4 B) 2/4 C) 3/4 D) 1

D) 1

Use the following information to answer the question. A plantlike organism on the planet Pandora can have three recessive genetic traits: bluish leaves, due to an allele (a) of geneA; a feathered stem, due to an allele (b) of gene B; and hollow roots due to an allele (c) of gene C. The three genes are linked and recombine. A geneticist performed a testcross with an organism that had been found to be heterozygous for the three recessive traits, and she was able to identify progeny of the following phenotypic distribution (+ = wild type): Which of the progeny phenotypes will require recombination between genes A and B? A) 2, 3, 5, and 7 B) 2, 4, 5, and 8 C) 1, 3, 6, and 7 D) 1, 2, 5, and 6

D) 1, 2, 5, and 6

How much of 0.5 M glucose (molecular mass 180) is needed to provide 100 mg of glucose? A) 10 mL B) 0.11 mL C) 100 mL D) 1.11 mL

D) 1.11 mL

A man has extra digits (six fingers on each hand and six toes on each foot). His wife and their daughter have the normal number of digits (five fingers on each hand and five toes on each foot.) Having extra digits is a dominant trait. The couple's second child has extra digits. What is the probability that their next (third) child will have extra digits? A) 1/8 B) 3/4 C) 1/16 D) 1/2

D) 1/2

Albinism is a recessive trait. A man and woman both show normal pigmentation, but both have one parent who has albinism (without melanin pigmentation). What is the probability that their first child will have albinism? A) 1 B) 1/2 C) 0 D) 1/4

D) 1/4

The pattern of inheritance can be predicted from data if one is given the parent or offspring genotypes or phenotypes. Two organisms, with genotypes BbDD and BBDd, are mated. Assuming independent assortment of the B/b and D/d genes, determine the genotypic ratios in offspring that would occur. A) 1/2 BBDD, 1/2 bbdd B) 1/4 BBDD, 1/2 BbDd, 1/4 bbdd C) 9/16 BBDD, 3/16 BbDD, 3/16 BBDd, 1/16 bbdd D) 1/4 BBDD, 1/4 BbDD, 1/4 BBDd, 1/4 BbDd

D) 1/4 BBDD, 1/4 BbDD, 1/4 BBDd, 1/4 BbDd

If a cell at metaphase of mitosis contains 20 sister chromatids, how many chromosomes will be present in a G1 cell? A) 5 B) 20 C) 40 D) 10

D) 10

Cinnabar eye color is a sex-linked, recessive characteristic in fruit flies. If a female having cinnabar eyes is crossed with a wild-type male, what percentage of the F1 males will have cinnabar eyes? A) 0% B) 25% C) 50% D) 100%

D) 100%

If a cell is able to synthesize 30 ATP molecules for each molecule of glucose completely oxidized to carbon dioxide and water, approximately how many ATP molecules can the cell synthesize for each molecule of pyruvate oxidized to carbon dioxide and water? A) 16 B) 8 C) 25 D) 12.5

D) 12.5

Thymine makes up 28% of the nucleotides in a sample of DNA from an organism. Approximately what percentage of the nucleotides in this sample will be guanine? A) 8% B) 42% C) 55% D) 22%

D) 22%

In carbohydrates, the ratio of hydrogen (H) to oxygen (O) is ________. A) 3:1 B) 1:1 C) 4:1 D) 2:1

D) 2:1

If the DNA content of a diploid cell in the G1 phase of the cell cycle is x, then the DNA content of the same cell at metaphase of meiosis I will be A) 0.25x. B) x. C) 0.5x. D) 2x.

D) 2x.

Use the following information to answer the question. A plantlike organism on the planet Pandora can have three recessive genetic traits: bluish leaves, due to an allele (a) of geneA; a feathered stem, due to an allele (b) of gene B; and hollow roots due to an allele (c) of gene C. The three genes are linked and recombine. A geneticist performed a testcross with an organism that had been found to be heterozygous for the three recessive traits, and she was able to identify progeny of the following phenotypic distribution (+ = wild type): If recombination frequency is equal to distance in map units, what is the approximate distance between genes A and B? A) 6 map units B) 15 map units C) 30 map units D) 3 map units

D) 3 map units

Quaking aspen trees can send out underground stems for asexual reproduction. Sexual reproduction is not as common, but when it does happen, the haploid gametes have 19 chromosomes. How many chromosomes are in the cells of the underground stems? A) 9 B) 10 C) 19 D) 38

D) 38

How many electrons does one atom of carbon share to complete its valence shell? A) 2 B) 8 C) 3 D) 4

D) 4

The citric acid cycle. For each mole of glucose (C6H12O6) oxidized by cellular respiration, how many moles of CO2 are released in the citric acid cycle (see the accompanying figure)? A) 2 B) 6 C) 32 D) 4

D) 4

Arrange the following in order from most general to most specific. 1 natural selection 2. microevolution 3. intrasexual selection 4. evolution 5. sexual selection A) 4, 1, 2, 3, 5 B) 1, 4, 2, 5, 3 C) 4, 2, 1, 3, 5 D) 4, 2, 1, 5, 3

D) 4, 2, 1, 5, 3

Which molecule has both hydrophilic and hydrophobic properties and is found in plasma membranes? A) 12 B) 14 C) 1 D) 5

D) 5

Referring to the figure, what bases will be added to the primer as DNA replication proceeds? A) 5' C,A,G,C,A,G,A 3' B) 3' T,C,T,G,C,T,G 5' C) 3' G,T,C,G,T,C,T 5' D) 5' A,G,A,C,G,A,C 3'

D) 5' A,G,A,C,G,A,C 3'

Using Figure 17.6, identify a 5′ → 3′ sequence of nucleotides in the DNA template strand for an mRNA coding for the polypeptide sequence Phe-Pro-Lys. A) 5-AAACCCUUU-3 B) 5-UUUCCCAAA-3 C) 5-GAACCCCTT-3 D) 5-CTTCGGGAA-3

D) 5-CTTCGGGAA-3

Use the figure to answer the question. The figure represents tRNA that recognizes and binds a particular amino acid (in this instance, phenylalanine). Which codon on the mRNA strand codes for this amino acid? A) 3′-GUG-5′ B) 5′-UGG-3′ C) 5′-GUA-3′ D) 5′-UUC-3′

D) 5′-UUC-3′

In an analysis of the nucleotide composition of a molecule of DNA, which of the following combinations of base pairs will be found? A) A = G and C = T B) G + C = T + A C) A = C D) A + C = G + T

D) A + C = G + T

In humans, ABO blood types refer to glycoproteins in the membranes of red blood cells. There are three alleles for this autosomal gene: IA, IB, and i. The IA allele codes for the A glycoprotein, The IB allele codes for the B glycoprotein, and the i allele doesn't code for any membrane glycoprotein. IA and IB are codominant, and i is recessive to both IA and IB. People with type A blood have the genotypes IAIA or IAi, people with type B blood areIBIB or IBi, people with type AB blood are IAIB, and people with type O blood are ii. If a woman with type AB blood marries a man with type O blood, which of the following blood types could their children possibly have? A) AB and O B) A, B, and O C) A, B, AB, and O D) A and B

D) A and B

Which of the following statements is consistent with the second law of thermodynamics? A) A gain of free energy in a system is always associated with conversion of energy from one form to another. B) Without an input of energy, the entropy of an organism would tend to decrease over time. C) Every energy transformation performed by an organism decreases the entropy of the universe. D) A constant input of energy is required to maintain the high level of cellular organization.

D) A constant input of energy is required to maintain the high level of cellular organization.

Which of the following can be attributed to water's high specific heat? A) Sugar dissolves in hot tea faster than in iced tea. B) Oil and water do not mix well. C) Ice floats on water. D) A lake heats up more slowly than the air around it.

D) A lake heats up more slowly than the air around it.

What are the products of cyclic electron flow during the light reactions of photosynthesis? A) ADP and NADP+ B) heat and fluorescence C) ATP and NADPH D) ATP

D) ATP

Which of the following is the most correct interpretation of the figure? A) Energy from catabolism can be used directly for performing cellular work. B) ADP + (P)i are a set of molecules that store energy for catabolism. C) (P)i acts as a shuttle molecule to move energy from ATP to ADP. D) ATP is a molecule that acts as an intermediary to store energy for cellular work.

D) ATP is a molecule that acts as an intermediary to store energy for cellular work.

Which of the following is true during a typical cAMP-mediated signal transduction event? A) The second messenger is the last part of the system to be activated. B) The second messenger amplifies the hormonal response by attracting more hormones to the cell being affected. C) A hormone activates the second messenger by directly binding to it. D) Adenylyl cyclase is activated after the hormone binds to the cell and before phosphorylation of proteins occurs.

D) Adenylyl cyclase is activated after the hormone binds to the cell and before phosphorylation of proteins occurs.

A black guinea pig crossed with a guinea pig with albinism produced 12 black offspring. When the albino was crossed with a second black animal, six blacks and six albinos were obtained. What is the best explanation for this genetic situation? A) Albinism and black are codominant. B) Albinism is a recessive trait; black is codominant. C) Albinism is a dominant trait; black is incompletely dominant. D) Albinism is a recessive trait; black is a dominant trait.

D) Albinism is a recessive trait; black is a dominant trait.

Which of the following statements is true? A) A typical human liver cell has one set of chromosomes B) mRNA is the only type of RNA found in the living system C) Organisms interact but do not affect their environment D) All forms of life employ the same genetic code

D) All forms of life employ the same genetic code

The figure illustrates the energy states associated with the reaction A + B ↔ C + D. Which of the following represents the activation energy required for the non-enzyme-catalyzed reaction in the figure? A) D B) A C) B D) C

D) C

Visualize the structural formula of each of the following hydrocarbons. Which hydrocarbon has a double bond in its carbon skeleton? A) C3H8 B) C2H2 C) C2H6 D) C2H4

D) C2H4

How do cells use the ATP cycle illustrated in the figure? A) Cells use the cycle to recycle energy released by ATP hydrolysis. B) Cells use the cycle primarily to generate heat. C) Cells use the cycle to recycle ADP, phosphate, and the energy released by ATP hydrolysis. D) Cells use the cycle to recycle ADP and phosphate.

D) Cells use the cycle to recycle ADP and phosphate.

For the following questions, match the labeled component of the cell membrane in the figure with its description. Which component in the accompanying figure is hydrophilic? A) A B) B C) C D) D

D) D

In the late 1950s, Meselson and Stahl grew bacteria in a medium containing "heavy" (radioactive) nitrogen (13N) and then transferred them to a medium containing 14N (non-radioactive). Which of the results in the figure would be expected after one round of DNA replication in the presence of 14N? A) A B) B C) C D) D

D) D

Which component is not directly involved in translation? A) tRNA B) GTP C) ribosomes D) DNA

D) DNA

Which of the following facts did Hershey and Chase make use of in trying to determine whether DNA or protein is the genetic material? A) DNA contains purines, whereas protein includes pyrimidines. B) DNA contains sulfur, whereas protein does not. C) DNA contains nitrogen, whereas protein does not. D) DNA contains phosphorus, whereas protein does not.

D) DNA contains phosphorus, whereas protein does not.

Suppose you are provided with an actively dividing culture of E. coli bacteria to which radioactive thymine has been added. What would happen if a cell replicates once in the presence of this radioactive base? A) Neither of the two daughter cells would be radioactive. B) One of the daughter cells, but not the other, would have radioactive DNA. C) All four bases of the DNA would be radioactive. D) DNA in both daughter cells would be radioactive.

D) DNA in both daughter cells would be radioactive.

A biochemist isolates, purifies, and combines in a test tube a variety of molecules needed for DNA replication. When she adds some DNA to the mixture, replication occurs, but each DNA molecule consists of a normal strand paired with numerous segments of DNA a few hundred nucleotides long. What has she probably left out of the mixture? A) primase B) DNA polymerase C) Okazaki fragments D) DNA ligase

D) DNA ligase

The lagging strand is characterized by a series of short segments of DNA (Okazaki fragments) that will be joined together to form a finished lagging strand. The experiments that led to the discovery of Okazaki fragments gave evidence for which of the following ideas? A) Bacterial replication is fundamentally different from eukaryotic replication. The key should not be way longer than the distractors. B) DNA is the genetic material. C) DNA is a polymer consisting of four monomers: adenine, thymine, guanine, and cytosine. D) DNA polymerase is an enzyme that synthesizes leading and lagging strands during replication only in one direction.

D) DNA polymerase is an enzyme that synthesizes leading and lagging strands during replication only in one direction.

Some photosynthetic organisms contain chloroplasts that lack photosystem II, yet are able to survive. Which of the following approaches would be the best way to detect the lack of photosystem II in these organisms? A) Determine the action spectrum for photosynthesis in these organisms. B) Determine whether they consume CO2 in the dark. C) Determine whether they have thylakoid membranes. D) Determine whether they produce O2 in the light.

D) Determine whether they produce O2 in the light.

Which of the following statements is a logical consequence of the second law of thermodynamics? A) If the entropy of a system decreases, there must be a corresponding decrease in the entropy of the universe. B) If there is an increase in the energy of a system, there must be a corresponding decrease in the energy of the rest of the universe. C) If the entropy of a system increases, there must be a corresponding decrease in the entropy of the universe. D) Each chemical reaction in an organism must increase the total entropy of the universe.

D) Each chemical reaction in an organism must increase the total entropy of the universe.

Which of the following statements is true of a species that has a chromosome number of 2n = 16? A) The species is diploid with 32 chromosomes per cell. B) A gamete from this species has four chromosomes. C) The species has 16 sets of chromosomes per cell. D) Each diploid cell has eight homologous pairs of chromosomes.

D) Each diploid cell has eight homologous pairs of chromosomes.

Sutherland discovered that the signaling molecule epinephrine is responsible for which of the following events? A) brings about a decrease in levels of cAMP as a result of bypassing the plasma membrane B) causes lower blood glucose by binding to liver cells C) interacts directly with glycogen phosphorylase D) elevates cytosolic concentrations of cyclic AMP

D) Elevating cytosolic concentrations of cyclic AMP.

Which of the following statements correctly describes the primary difference between enhancers and proximal control elements? A) Enhancers are DNA sequences; proximal control elements are proteins. B) Enhancers are transcription factors; proximal control elements are DNA sequences. C) Enhancers improve transcription; proximal control elements inhibit transcription. D) Enhancers are located considerable distances from the promoter; proximal control elements are close to the promoter.

D) Enhancers are located considerable distances from the promoter; proximal control elements are close to the promoter.

Which of the following statements about enzyme function is true? A) Enzyme function is independent of physical and chemical environmental factors such as pH and temperature. B) Enzyme function is generally increased if the three-dimensional structure or conformation of an enzyme is altered. C) Enzymes increase the rate of chemical reactions by providing activation energy to the substrate. D) Enzymes increase the rate of chemical reactions by lowering activation energy barriers.

D) Enzymes increase the rate of chemical reactions by lowering activation energy barriers.

A mutation that disrupts the ability of an animal cell to add polysaccharide modifications to proteins would most likely cause defects in which of the following organelles or structures? A) mitochondria and Golgi apparatus B) nuclear pores and secretory vesicles C) nuclear matrix and extracellular matrix D) Golgi apparatus and extracellular matrix

D) Golgi apparatus and extracellular matrix

The immediate energy source that drives ATP synthesis by ATP synthase during oxidative phosphorylation is the A) flow of electrons down the electron transport chain. B) oxidation of glucose and other organic compounds. C) transfer of phosphate to ADP. D) H+ concentration gradient across the membrane holding ATP synthase.

D) H+ concentration gradient across the membrane holding ATP synthase.

A sexually reproducing animal has two unlinked genes, one for head shape (H) and one for tail length (T). Its genotype is HhTt. Which of the following genotypes is possible in a gamete from this organism? A) HhTt B) T C) Hh D) HT

D) HT

Which of the following statements accurately describes one characteristic of histones? A) Histones are found in mammals, but not in other animals or in plants or fungi. B) Each nucleosome consists of two molecules of histone H1. C) The carboxyl end of each histone extends outward from the nucleosome and is called a "histone tail." D) Histone H1 is not present in the nucleosome bead; instead, it draws the nucleosomes together.

D) Histone H1 is not present in the nucleosome bead; instead, it draws the nucleosomes together.

Which of the following molecular characteristics cause histones to bind tightly to DNA? A) Histones are negatively charged, and DNA is positively charged. B) Both histones and DNA are strongly hydrophobic. C) Histones are covalently linked to the DNA. D) Histones are positively charged, and DNA is negatively charged.

D) Histones are positively charged, and DNA is negatively charged.

DNA was isolated from three different cell types of the same organism, the relative DNA content for each type was determined, and the results were plotted on the graph shown in the figure below. Refer to the graph to answer the following questions. Which sample of DNA might represent an animal cell in the G2 phase of the cell cycle prior to meiosis? A) both I and II B) III C) I D) II

D) II

In the figure, which number represents DNA synthesis? A) IV B) I C) III D) II

D) II

Refer to the life cycles illustrated in the figure below to answer the following questions. Which of the life cycles is typical for most fungi and some protists? A) I and II B) I only C) III only D) II only

D) II only

In the figure, MPF reaches its highest concentration during this stage. A) IV B) I C) II D) III

D) III

Refer to the life cycles illustrated in the figure below to answer the following questions. Which of the life cycles is typical for plants and some algae? A) II only B) I only C) I and III D) III only

D) III only

Which sentence best describes the logic of scientific inquiry? A) If my observations are accurate, they will support my hypothesis. B) If I generate a testable hypothesis, tests and observations will support it. C) If my prediction is correct, it will lead to a testable hypothesis. D) If my hypothesis is correct, I can expect certain test results.

D) If my hypothesis is correct, I can expect certain test results.

Which of the following statements is correct in comparing sexual and asexual reproduction? A) Asexual reproduction, but not sexual reproduction, is characteristic of only plants and fungi. B) In asexual reproduction, offspring are produced by fertilization without meiosis. C) Asexual reproduction produces only haploid offspring. D) In sexual reproduction, individuals transmit half of their nuclear genes to each of their offspring.

D) In sexual reproduction, individuals transmit half of their nuclear genes to each of their offspring.

Why does testosterone, a lipid-soluble signaling molecule that crosses the membranes of all cells, affect only target cells? A) Only target cells retain the appropriate genes regulated by testosterone. B) Only target cells possess the cytosolic enzymes that transduce the signal from testosterone to adenylyl cyclase. C) Only in target cells is testosterone able to initiate the phosphorylation cascade leading to activated transcription factor. D) Intracellular receptors for testosterone are present only in target cells.

D) Intracellular receptors for testosterone are present only in target cells.

Why is carbon so important in biology? A) It is a common element on Earth. B) It bonds to only a few other elements. C) It has very little electronegativity, making it a good electron donor. D) It can form a variety of carbon skeletons and host functional groups.

D) It can form a variety of carbon skeletons and host functional groups.

There is a mutation that is found in E. coli in the repressor that results in a molecule known as a super-repressor because it represses the lac operon permanently. Which of the following characteristics would you expect to observe in such a mutant? A) It cannot bind to the operator. B) It cannot make a functional repressor. C) It makes a repressor that binds CAP. D) It cannot bind to the inducer.

D) It cannot bind to the inducer.

Which of the following is a primary function of the active site of an enzyme? A) It is activated by the presence of the end product of the metabolic pathway in which the enzyme is involved. B) It binds noncompetitive inhibitors of the enzyme. C) It binds allosteric regulators of the enzyme. D) It catalyzes the reaction associated with the enzyme.

D) It catalyzes the reaction associated with the enzyme.

Which of the following is not true of a codon? A) It is the basic unit of the genetic code. B) It never codes for more than one amino acid. C) It may code for the same amino acid as another codon. D) It extends from one end of a tRNA molecule.

D) It extends from one end of a tRNA molecule.

Which of the following statements about the evolution of life on Earth, from simple prokaryote-like cells to multicellular eukaryotic organisms, is true? A) It has occurred in accordance with the laws of thermodynamics and resulted in a substantial increase in the total energy in the universe. B) By resulting in such diversity and complexity of life, it is an exception to the second law of thermodynamics. C) It has occurred in accordance with the laws of thermodynamics and resulted in a substantial increase in the entropy of the planet. D) It has occurred in accordance with the laws of thermodynamics and resulted in a substantial decrease in the entropy of the planet.

D) It has occurred in accordance with the laws of thermodynamics and resulted in a substantial decrease in the entropy of the planet.

Which of the following functions does the product of the p53 gene carry out? A) It causes cells to reduce expression of genes involved in DNA repair. B) It slows down the rate of DNA replication by interfering with the binding of DNA polymerase. C) It allows cells to pass on mutations due to DNA damage. D) It inhibits the cell cycle.

D) It inhibits the cell cycle.

For a chemotherapeutic drug to be useful for treating cancer cells, which of the following is most desirable? A) It is safe enough to limit all apoptosis. B) It does not alter metabolically active cells. C) It interferes with cells entering G0. D) It interferes with rapidly dividing cells.

D) It interferes with rapidly dividing cells.

Which of the following statements correctly describes the effect a nonsense mutation would have on a gene? A) It changes an amino acid in the encoded protein. B) It alters the reading frame of the mRNA. C) It has no effect on the amino acid sequence of the encoded protein. D) It introduces a premature stop codon into the mRNA.

D) It introduces a premature stop codon into the mRNA.

Characterization of the complete three-dimensional structure of a newly purified protein suggests that it catalyzes the breakdown of a large substrate. The protein consists of a single polypeptide chain. It has a large pocket that appears to be the binding site for the substrate and a smaller indentation that appears to be the binding site for a regulatory molecule. What do these structural observations suggest about the mechanism by which the activity of this protein is likely regulated? A) It is probably an enzyme that is regulated by competitive inhibition. B) It is probably an enzyme that is regulated by cooperativity. C) It is probably a multi-subunit enzyme that is regulated by allosteric regulation. D) It is probably an enzyme that is regulated by noncompetitive inhibition.

D) It is probably an enzyme that is regulated by noncompetitive inhibition.

In a Drosophila experiment, a cross was made between homozygous wild-type females and yellow-bodied males. All of the resulting F1s were phenotypically wild type. However, adult flies of the F2 generation (resulting from matings of the F1s) had the characteristics shown in the figure. How is the mutant allele for yellow body inherited? A) It is dominant. B) It is incompletely dominant. C) It is codominant. D) It is recessive.

D) It is recessive.

What happens to the free energy released as electrons are passed from photosystem II to photosystem I through a series of electron carriers? A) It is used to phosphorylate NAD+ to NADPH. B) It excites electrons of the reaction center in photosystem I. C) It is used to synthesize ATP through substrate-level phosphorylation. D) It is used to establish and maintain a proton gradient.

D) It is used to establish and maintain a proton gradient.

How does the toxin of Vibrio cholerae cause profuse diarrhea? A) It modifies a ligand-gated ion channel. B) It signals IP3 to act as a second messenger for the release of calcium. C) It modifies adenylyl cyclase and triggers excess formation of cAMP. D) It modifies a G protein involved in regulating salt and water secretion.

D) It modifies a G protein involved in regulating salt and water secretion.

Which of the following statements summarizes the metabolic results of photorespiration? A) It produces oxygen and ATP and consumes carbon dioxide. B) It produces oxygen and consumes ATP and carbon dioxide. C) It produces ATP and consumes oxygen and carbon dioxide. D) It produces carbon dioxide and consumes ATP and oxygen.

D) It produces carbon dioxide and consumes ATP and oxygen.

A particular organism has 46 chromosomes in its karyotype. Which of the following statements is correct regarding this organism? A) It reproduces sexually. B) It must be human. C) It must be an animal. D) It produces gametes with 23 chromosomes.

D) It produces gametes with 23 chromosomes.

If an animal cell suddenly lost the ability to produce GTP, what might happen to its signaling system? A) It would be able to carry out reception and transduction but would not be able to respond to a signal. B) It would not be able to activate receptor tyrosine kinases. C) It would use ATP instead of GTP to activate G proteins on the cytoplasmic side of the plasma membrane. D) It would not be able to activate G proteins on the cytoplasmic side of the plasma membrane.

D) It would not be able to activate G proteins on the cytoplasmic side of the plasma membrane.

Why are males more often affected by sex-linked traits than females? A) Male hormones such as testosterone often alter the effects of mutations on the X chromosome. B) X chromosomes in males generally have more mutations than X chromosomes in females. C) Female hormones such as estrogen often compensate for the effects of mutations on the X chromosome. D) Males are hemizygous for the X chromosome.A recessive allele on the X chromosome is responsible for red-green color blindness in humans. A woman with normal vision whose father is color blind marries a color-blind male. What is the probability that this couple's first son will be color blind?

D) Males are hemizygous for the X chromosome.A recessive allele on the X chromosome is responsible for red-green color blindness in humans.

Which of the following statements is generally true of aneuploidies in newborns? A) An aneuploidy resulting in the deletion of a chromosome segment is less serious than a duplication. B) Human aneuploidy usually conveys an adaptive advantage in humans. C) A monosomy is more frequent than a trisomy. D) Monosomy X is the only viable monosomy known to occur in humans.

D) Monosomy X is the only viable monosomy known to occur in humans.

In 1986, a nuclear power accident in Chernobyl, USSR (now Ukraine), led to high radiation levels for miles surrounding the plant. The high levels of radiation caused elevated mutation rates in the surviving organisms, and evolutionary biologists have been studying rodent populations in the Chernobyl area ever since. Based on your understanding of evolutionary mechanisms, which of the following most likely occurred in the rodent populations following the accident? A) Mutation caused genetic drift and decreased fitness. B) Mutations caused major changes in rodent physiology over time. C) Mutation caused the fixation of new alleles. D) Mutation led to increased genetic variation.

D) Mutation led to increased genetic variation.

Following glycolysis and the citric acid cycle, but before the electron transport chain and oxidative phosphorylation, the carbon skeleton of glucose has been broken down to CO2 with some net gain of ATP. Most of the energy from the original glucose molecule at that point in the process, however, is stored in the form of which of the following molecules? A) pyruvate B) glucose C) acetyl-CoA D) NADH

D) NADH

What is an adaptive advantage of recombination between linked genes? A) Recombination must occur or genes will not assort independently. B) The forces on the cell during meiosis II result in recombination. C) Recombination is required for independent assortment. D) New allele combinations are acted upon by natural selection.

D) New allele combinations are acted upon by natural selection.

In E. coli, there is a mutation in a gene called dnaB that alters the helicase that normally acts at the origin of replication. Which of the following events would you expect to occur as a result of this mutation? A) Replication will occur via RNA polymerase alone. B) Replication will require a DNA template from another source. C) Additional proofreading will occur. D) No replication fork will be formed.

D) No replication fork will be formed.

Which of the following would likely diffuse through the lipid bilayer of a plasma membrane most rapidly? A) sucrose B) Na+ C) an amino acid D) O2

D) O2

Following a scientific method, which of the following is the correct order of steps? A) Observation → Analysis → Hypothesis → Conclusion → Communicate results → Experiment B) Experiment → Hypothesis → Observation → Analysis → Conclusion → Communicate results C) Observation → Hypothesis → Experiment → Communicate results → Analysis → Conclusion D) Observation → Hypothesis → Experiment → Analysis → Conclusion → Communicate results

D) Observation → Hypothesis → Experiment → Analysis → Conclusion → Communicate results

Duchenne muscular dystrophy is a serious condition caused by a recessive allele of a gene on the human X chromosome. The patients have muscles that weaken over time because they have absent or decreased dystrophin, a muscle protein. They rarely live past their 20s. How likely is it for a woman to have this condition? A) Only if a woman is XXX could she have this condition. B) One-fourth of the daughters of an affected man would have this condition. C) Women can never have this condition. D) One-half of the daughters of an affected father and a carrier mother could have this condition.

D) One-half of the daughters of an affected father and a carrier mother could have this condition.

A phenotypically normal prospective couple seeks genetic counseling because the man knows that he has a translocation of a portion of his chromosome 4, which has been exchanged with a portion of his chromosome 12. Although his translocation is balanced, he and his wife want to know the probability that his sperm will be abnormal. What is your prognosis regarding his sperm? A) Half will be normal, and the rest will have the father's translocation. B) None will carry the translocation. C) All will carry the same translocation as the father. D) One-quarter will carry the two normal chromosomes, 4 and 12, one-quarter will have only the two translocation chromosomes and no normal chromosomes 4 and 12, and half will have one normal and one translocated chromosome.

D) One-quarter will carry the two normal chromosomes, 4 and 12, one-quarter will have only the two translocation chromosomes and no normal chromosomes 4 and 12, and half will have one normal and one translocated chromosome.

In which of the following ways do plant hormones differ from hormones in animals? A) Animal hormones typically travel from the hormone producing cell to an adjacent responding cell through gap junctions. B) Plant hormones commonly travel through the soil from one plant to another. C) Animal hormones are only local regulators. D) Plant hormones frequently travel through the air as a gas.

D) Plant hormones frequently travel through the air as a gas.

What would be the consequence of a mutation in a bacterial cell that produces a defective aminoacyl-tRNA synthetase that attaches a lysine instead of the normal phenylalanine to tRNAs with the anticodon AAA? A) None of the proteins in the cell will contain phenylalanine. B) The ribosome will skip a codon every time a UUU is encountered. C) The cell will compensate for the defect by attaching phenylalanine to tRNAs with lysine-specifying anticodons. D) Proteins in the cell will include lysine instead of phenylalanine at amino acid positions specified by the codon UUU.

D) Proteins in the cell will include lysine instead of phenylalanine at amino acid positions specified by the codon UUU.

Which of the following statements correctly describes some aspect of protein secretion from prokaryotic cells? A) The mechanism of protein secretion in prokaryotes is probably the same as that in eukaryotes. B) Prokaryotes cannot secrete proteins because they lack rough endoplasmic reticulum. C) Prokaryotes cannot secrete proteins because they lack an endomembrane system. D) Proteins secreted by prokaryotes are likely synthesized on ribosomes bound to the cytoplasmic surface of the plasma membrane.

D) Proteins secreted by prokaryotes are likely synthesized on ribosomes bound to the cytoplasmic surface of the plasma membrane.

Which of the following processes destroys RNA molecules in a cell if they have a sequence complementary to an introduced double-stranded RNA? A) RNA blocking B) RNA obstruction C) RNA disposal D) RNA interference

D) RNA interference

For a repressible operon to be transcribed, which of the following conditions must occur? A) RNA polymerase must not occupy the promoter, and the repressor must be inactive. B) RNA polymerase and the active repressor must be present. C) A corepressor must be present. D) RNA polymerase must bind to the promoter, and the repressor must be inactive.

D) RNA polymerase must bind to the promoter, and the repressor must be inactive.

Which of the following types of signaling is represented in the figure? A) autocrine B) paracrine C) hormonal D) synaptic

D) Synaptic

Telomere shortening puts a limit on the number of times a cell can divide. Research has shown that telomerase can extend the life span of cultured human cells. How might adding telomerase affect cellular aging? A) Telomerase will speed up the rate of cell proliferation. B) Telomerase would have no effect on cellular aging. C) Telomerase shortens telomeres, which delays cellular aging. D) Telomerase eliminates telomere shortening and retards aging.

D) Telomerase eliminates telomere shortening and retards aging.

To identify the molecule that accepts CO2, Calvin and Benson manipulated the carbon-fixation cycle by either cutting off CO2 or cutting off light from cultures of photosynthetic algae. They then measured the concentrations of various metabolites immediately following the manipulation. How would these experiments help identify the CO2 acceptor? A) The CO2 acceptor concentration would decrease when either the CO2 or light are cut off. B) The CO2 acceptor concentration would increase when either the CO2 or light are cut off. C) The CO2 acceptor concentration would decrease when the CO2 is cut off, but increase when the light is cut off. D) The CO2 acceptor concentration would increase when the CO2 is cut off, but decrease when the light is cut off.

D) The CO2 acceptor concentration would increase when the CO2 is cut off, but decrease when the light is cut off.

Use the figure to answer the question. The tRNA shown in the figure has its 3′ end projecting beyond its 5′ end. Which of the following processes will occur at this 3′ end? A) The excess nucleotides (ACCA) will be cleaved off at the ribosome. B) The small and large subunits of the ribosome will attach to it. C) The 5′ cap of the mRNA will become covalently bound. D) The amino acid binds covalently.

D) The amino acid binds covalently.

Which of the following is true when comparing an uncatalyzed reaction to the same reaction with a catalyst? A) The catalyzed reaction will be slower. B) The catalyzed reaction will have higher activation energy. C) The catalyzed reaction will consume all of the catalyst. D) The catalyzed reaction will have the same ∆G.

D) The catalyzed reaction will have the same ∆G.

Early observations of a cultured cell line indicated that the cells did not exhibit either density-dependent inhibition or anchorage dependence. What do these observations suggest about this cell line? A) The cells follow an altered series of cell cycle phases. B) The cells have nonfunctional MPF. C) The cells are unable to form spindle microtubules. D) The cells show characteristics of tumors.

D) The cells show characteristics of tumors.

Which of the following conditions is most likely to cause the lactose operon to be transcribed? A) There is glucose but no lactose in the cell. B) The cAMP level is high and the lactose level is low. C) There is more glucose in the cell than lactose. D) The cyclic AMP and lactose levels are both high within the cell.

D) The cyclic AMP and lactose levels are both high within the cell.

Living organisms increase in complexity as they grow, resulting in a decrease in the entropy of an organism. How does this relate to the second law of thermodynamics? A) Living organisms do not obey the second law of thermodynamics, which states that the entropy of an organism increases with each energy transformation. B) Living organisms are able to transform chemical energy into entropy. C) As a consequence of growth, the decrease in entropy of the organism is associated with a corresponding decrease in the entropy of the universe. D) The decrease in entropy is associated with growth of an organism. As a consequence of growth, organisms cause a greater increase in entropy in their environment than the decrease in entropy associated with their increased complexity.

D) The decrease in entropy is associated with growth of an organism. As a consequence of growth, organisms cause a greater increase in entropy in their environment than the decrease in entropy associated with their increased complexity.

Which of the following statements is a reasonable explanation for why unsaturated fatty acids help keep a membrane more fluid at lower temperatures? A) Unsaturated fatty acids are more nonpolar than saturated fatty acids. B) The double bonds block interaction among the hydrophilic head groups of the lipids. C) Unsaturated fatty acids have a higher cholesterol content, which prevents adjacent lipids from packing tightly. D) The double bonds form kinks in the fatty acid tails, preventing adjacent lipids from packing tightly.

D) The double bonds form kinks in the fatty acid tails, preventing adjacent lipids from packing tightly.

In 1668, Francesco Redi performed a series of experiments on spontaneous generation. He began by putting similar pieces of meat into eight identical jars. Four jars were left open to the air, and four were sealed. He then did the same experiment with one variation: Instead of sealing four of the jars completely, he covered them with gauze (the gauze excluded the flies while allowing the meat to be exposed to air). In both experiments, he monitored the jars and recorded whether or not maggots (young flies) appeared in the meat. In both experiments, flies appeared in all of the open jars and only in the open jars. Which one of the following statements is correct? A) The experiment was inconclusive because it did not run long enough. B) The experiment supports the hypothesis that spontaneous generation occurs in rotting meat. C) The experiment was inconclusive because Redi used only one kind of meat. D) The experiment supports the hypothesis that maggots arise only from eggs laid by adult flies.

D) The experiment supports the hypothesis that maggots arise only from eggs laid by adult flies.

Which of the following is an example of qualitative data? A) The six pairs of robins hatched an average of three chicks each. B) The temperature decreased from 20°C to 15°C. C) The contents of the stomach are mixed every 20 seconds. D) The fish swam in a zigzag motion.

D) The fish swam in a zigzag motion.

When Thomas Hunt Morgan crossed his red-eyed F1 generation flies to each other, the F2generation included both red- and white-eyed flies. Remarkably, all the white-eyed flies were male. What was the explanation for this result? A) The gene involved is located on the Y chromosome. B) The gene involved is located on an autosome, but only in males. C) Other male-specific factors influence eye color in flies. D) The gene involved is located on the X chromosome.

D) The gene involved is located on the X chromosome.

Suppose an experimenter becomes proficient with a technique that allows her to move DNA sequences within a prokaryotic genome. If a researcher moves the repressor gene (lacI), along with its promoter, to a position at some several thousand base pairs away from its normal position, which of the following results would be expected? A) The lac operon will be expressed continuously. B) The repressor will no longer bind to the inducer. C) The repressor will no longer bind to the operator. D) The lac operon will function normally.

D) The lac operon will function normally.

Which of the following statements correctly describes the difference between the leading strand and the lagging strand in DNA replication? A) The leading strand is synthesized in the 3' → 5' direction in a discontinuous fashion, while the lagging strand is synthesized in the 5' → 3' direction in a continuous fashion. B) The leading strand requires an RNA primer, whereas the lagging strand does not. C) There are different DNA polymerases involved in elongation of the leading strand and the lagging strand. D) The leading strand is synthesized continuously in the 5' → 3' direction, while the lagging strand is synthesized discontinuously in the 5' → 3' direction.

D) The leading strand is synthesized continuously in the 5' → 3' direction, while the lagging strand is synthesized discontinuously in the 5' → 3' direction.

In an experimental situation, a student researcher inserts an mRNA molecule into a eukaryotic cell after she has removed its 5′ cap and poly-A tail. Which of the following processes would you expect her to find to have occurred? A) The mRNA attaches to a ribosome and is translated, but more slowly. B) The mRNA is quickly converted into a ribosomal subunit. C) The cell adds a new poly-A tail to the mRNA. D) The molecule is digested by enzymes because it is not protected at the 5′ end.

D) The molecule is digested by enzymes because it is not protected at the 5′ end.

A beaker contains 100 milliliters (mL) of NaOH solution at pH = 13. A technician carefully pours into the beaker 10 mL of HCl at pH = 1. Which of the following statements correctly describes the result of this mixing? A) The concentration of Na+ ions will rise. B) The pH of the beaker's contents will be neutral. C) The pH of the beaker's contents will increase. D) The pH of the beaker's contents will decrease.

D) The pH of the beaker's contents will decrease.

Which statement correctly identifies the result that the optimum pH for amylase function is 7? A) At pH 9, the enzyme is denatured and will lose its function, but not its structure. B) The pH with the lowest absorbance values would indicate the optimum pH for amylase since this pH does not affect the structure or function of the protein. C) At pH 4, the structure of the enzyme will be altered, and the enzyme would not be able to catalyze the reaction. D) The pH with the highest absorbance values would indicate the optimum pH for amylase since this pH does not affect the structure or function of the protein.

D) The pH with the highest absorbance values would indicate the optimum pH for amylase since this pH does not affect the structure or function of the protein.

A patient was involved a serious accident and lost a large quantity of blood. In an attempt to replenish body fluids, distilled water—equal to the volume of blood lost—is added to the blood directly via one of his veins. What will be the most probable result of this transfusion? A) The patient's red blood cells will shrivel up because the blood has become hypotonic compared to the cells. B) The patient's red blood cells will burst because the blood has become hypertonic compared to the cells. C) The patient's red blood cells will shrivel up because the blood has become hypertonic compared to the cells. D) The patient's red blood cells will swell and possibly burst because the blood has become hypotonic compared to the cells.

D) The patient's red blood cells will swell and possibly burst because the blood has become hypotonic compared to the cells.

How do phospholipids interact with water molecules? A) The polar heads avoid water; the nonpolar tails attract water (because water is polar and opposites attract). B) Phospholipids do not interact with water because water is polar and lipids are nonpolar. C) Phospholipids dissolve in water. D) The polar heads interact with water; the nonpolar tails do not.

D) The polar heads interact with water; the nonpolar tails do not.

In bacteria, there are 61 mRNA codons that specify an amino acid, but only 45 tRNAs. Which of the following statements explains this fact? A) Some tRNAs have anticodons that recognize four or more different codons. B) Many codons are never used, so the tRNAs that recognize them are dispensable. C) The DNA codes for all 61 tRNAs, but some are then destroyed. D) The rules for base pairing between the third base of a codon and tRNA are flexible.

D) The rules for base pairing between the third base of a codon and tRNA are flexible.

Which of the following statements would explain a testcross involving F1 dihybrid flies in which more parental-type offspring than recombinant-type offspring are produced? A) The two genes are linked but on different chromosomes. B) Both of the characters are controlled by more than one gene. C) Recombination did not occur in the cell during meiosis. D) The two genes are closely linked on the same chromosome.

D) The two genes are closely linked on the same chromosome.

What is the function of the pigment molecules in a light-harvesting complex in the thylakoid membranes? A) They synthesize ATP from ADP and (P)i. B) They split water and release oxygen from the reaction-center chlorophyll. C) They transfer electrons to NADP+. D) They absorb and transfer light energy to the reaction-center chlorophyll.

D) They absorb and transfer light energy to the reaction-center chlorophyll.

Which of the following functions are characteristic of general transcription factors in eukaryotes? A) They usually lead to a high level of transcription even without additional specific transcription factors. B) They bind to sequences just after the start site of transcription. C) They inhibit RNA polymerase binding to the promoter and begin transcribing. D) They bind to other proteins or to the TATA box.

D) They bind to other proteins or to the TATA box.

Which of the following statements about anabolic pathways is true? A) They release energy by degrading polymers to monomers. B) They are usually spontaneous chemical reactions. C) They decrease the entropy of the organism and its environment. D) They consume energy to build up polymers from monomers.

D) They consume energy to build up polymers from monomers.

Which of the following is true of unsaturated fats? A) They generally solidify at room temperature. B) They are more common in animals than in plants. C) They contain more hydrogen than do saturated fats having the same number of carbon atoms. D) They have double bonds in their fatty acid chains.

D) They have double bonds in their fatty acid chains.

Which of the following statements correctly describes cis-trans isomers? A) They have the same chemical properties. B) They have different molecular formulas. C) They have an asymmetric carbon that makes them mirror images. D) They have variations in arrangement around a double bond.

D) They have variations in arrangement around a double bond.

Which of the following statements is true about buffer solutions? A) They maintain a constant pH of 7 B) They fluctuate in pH when either acids or bases are added to them. C) They maintain a constant pH when acids are added to them but not when bases are added to them. D) They maintain a relatively constant pH when either acids or bases are added to them.

D) They maintain a relatively constant pH when either acids or bases are added to them.

Several organisms, primarily protists, have what are called intermediate mitotic organization. What is the most probable hypothesis about these intermediate forms of cell division? A) They represent a form of cell reproduction that must have evolved completely separately from those of other organisms. B) They rely on totally different proteins for the processes they undergo. C) They may be more closely related to plant forms that also have unusual mitosis. D) They show some but not all of the evolutionary steps toward complete mitosis.

D) They show some but not all of the evolutionary steps toward complete mitosis.

For the following questions, match the key event of meiosis with the stages listed below. I. Prophase I V. Prophase II II. Metaphase I VI. Metaphase II III. Anaphase I VII. Anaphase II IV. Telophase I VIII. Telophase II 13.11. Centromeres of sister chromatids disjoin and chromatids separate. A) III B) V C) IV D) VII

D) VII

Red-green color blindness is a sex-linked recessive trait in humans. Two people with normal color vision have a color-blind son. What are the genotypes of the parents? A) XNXN and XnY B) XnXn and XnY C) XNXN and XNY D) XNXn and XNY

D) XNXn and XNY

All the organisms on your campus make up... A) an ecosystem. B) a population. C) a taxonomic domain. D) a community.

D) a community.

When nucleotides polymerize to form a nucleic acid, ________. A) hydrogen bonds form between the bases of two nucleotides B) covalent bonds form between the bases of two nucleotides C) a hydrogen bond forms between the sugar of one nucleotide and the phosphate of a second D) a covalent bond forms between the sugar of one nucleotide and the phosphate of a second

D) a covalent bond forms between the sugar of one nucleotide and the phosphate of a second

Which of the following factors would tend to increase membrane fluidity? A) a relatively high protein content in the membrane B) a greater proportion of saturated phospholipids C) a lower temperature D) a greater proportion of unsaturated phospholipids

D) a greater proportion of unsaturated phospholipids

Which of the following is an example of potential rather than kinetic energy? A) water rushing over Niagara Falls B) light flashes emitted by a firefly C) a crawling beetle foraging for food D) a molecule of glucose

D) a molecule of glucose

Cilia and flagella bend because of ________. A) contraction by myosin B) conformational changes in ATP that thrust microtubules laterally C) a motor protein called radial spokes D) a motor protein called dynein

D) a motor protein called dynein

In which of the following environments would there be the greatest need for osmoregulation? A) a plant being grown hydroponically in a watery mixture of designated nutrients B) a red blood cell surrounded by plasma C) an animal connective tissue cell bathed in isotonic body fluid D) a salmon moving from a river into an ocean

D) a salmon moving from a river into an ocean

For a species with a haploid number of 23 chromosomes, how many different combinations of maternal and paternal chromosomes are possible for the gametes based on the independent assortment of chromosomes during meiosis? A) 23 B) about 1,000 C) 46 D) about 8 million

D) about 8 million

Protein phosphorylation is commonly involved with all of the following except A) regulation of transcription by signaling molecules. B) activation of receptor tyrosine kinases. C) activation of protein kinase molecules. D) activation of G protein-coupled receptors.

D) activation of G protein-coupled receptors.

Mendel's law of independent assortment has its basis in which of the following events of meiosis I? A) the division of cells at telophase B) synapsis of homologous chromosomes C) crossing over of homologous pairs of chromosomes D) alignment of pairs of homologous chromosomes along the middle of the cell

D) alignment of pairs of homologous chromosomes along the middle of the cell

Which of the following individuals will inherit an X-linked allele from a man who carries it? A) half of his daughters B) All of his sons C) all of his children D) all of his daughters

D) all of his daughters

Which chemical group is most likely to be responsible for an organic molecule behaving as a base? A) hydroxyl B) carbonyl C) phosphate D) amino

D) amino

Phosphorylation cascades involving a series of protein kinases are useful for cellular signal transduction because they ________. A) counter the harmful effects of phosphatases B) always lead to the same cellular response C) are species specific D) amplify the original signal many times

D) amplify the original signal many times

All of the following are part of a prokaryotic cell EXCEPT _____. A) a cell wall B) a plasma membrane C) ribosomes D) an endoplasmic reticulum

D) an endoplasmic reticulum

Which of the following statements correctly describes the normal tonicity conditions for typical plant and animal cells? The animal cell is in ________. A) an isotonic solution, and the plant cell is in a hypertonic solution B) a hypertonic solution, and the plant cell is in an isotonic solution C) a hypotonic solution, and the plant cell is in an isotonic solution D) an isotonic solution, and the plant cell is in a hypotonic solution

D) an isotonic solution, and the plant cell is in a hypotonic solution

During which phase of mitosis do the chromatids become chromosomes? A) metaphase B) telophase C) prophase D) anaphase

D) anaphase

During which of the following phases of meiosis do homologous chromosomes separate? A) prophase I B) anaphase II C) mitosis D) anaphase I

D) anaphase I

During which of the following phases of meiosis do centromeres split and sister chromatids migrate to opposite poles of the cell? A) telophase II B) telophase I C) anaphase I D) anaphase II

D) anaphase II

Which axis in the embryo does the protein product of the bicoid gene in Drosophila determine? A) anterior-lateral axis B) posterior-ventral axis C) posterior-dorsal axis D) anterior-posterior axis

D) anterior-posterior axis

Research indicates that ibuprofen, a drug used to relieve inflammation and pain, is a mixture of two enantiomers; that is, molecules that ________. A) differ in the location of their double bonds B) have identical chemical formulas but differ in the branching of their carbon skeletons C) differ in the arrangement of atoms around their double bonds D) are mirror images of each other

D) are mirror images of each other

Starch and cellulose ________. A) are structural components of the plant cell wall B) are used for energy storage in plants and animals C) are cis and trans isomers of each other D) are polymers of glucose

D) are polymers of glucose

The figure shows the pedigree for a family. Dark-shaded symbols represent individuals with one of the two major types of colon cancer. Numbers under the symbols are the individual's age at the time of diagnosis. Males are represented by squares, females by circles. From this pedigree, this trait seems to be inherited ________. A) as an autosomal recessive B) from mothers C) as a result of epistasis D) as an autosomal dominant

D) as an autosomal dominant

Which of the following could be responsible for atherosclerosis and should be eliminated from diet for health reasons? A) butter B) olive oil C) liver and kidney organ meat D) butter, liver, and kidney organ meat

D) butter, liver, and kidney organ meat

The element present in all organic molecules is ________. A) nitrogen B) hydrogen C) oxygen D) carbon

D) carbon

Which of the following terms most precisely describes the cellular process of breaking down large molecules into smaller ones? A) anabolism (anabolic pathways) B) dehydration Answered C) metabolism D) catabolism (catabolic pathways)

D) catabolism (catabolic pathways)

HIV is the virus that causes AIDS. In the mid-1990s, researchers discovered an enzyme in HIV called protease. Once the enzyme's structure was known, researchers began looking for drugs that would fit into the active site and block it. If this strategy for stopping HIV infections were successful, it would be an example of what phenomenon? A) denaturation B) noncompetitive inhibition C) allosteric regulation D) competitive inhibition

D) competitive inhibition

Saturated fats ________. A) are generally liquid at room temperature B) have multiple double bonds in the carbon chains of their fatty acids C) are more common in plants than in animals D) contain more hydrogen than unsaturated fats that consist of the same number of carbon atoms

D) contain more hydrogen than unsaturated fats that consist of the same number of carbon atoms

Which of the following methods is utilized by eukaryotes to control their gene expression that is different from the type of control found in bacteria? A) control of RNA splicing B) transcriptional control C) control of chromatin remodeling D) control of both RNA splicing and chromatin remodeling

D) control of both RNA splicing and chromatin remodeling

A carbon atom is most likely to form what kind of bond(s) with other atoms? A) ionic bonds, covalent bonds, and hydrogen bonds B) hydrogen C) ionic D) covalent

D) covalent

Which bond or interaction would be difficult to disrupt when compounds are put into water? A) hydrogen bonds B) ionic and hydrogen bonds C) ionic bonds D) covalent bonds between carbon atoms

D) covalent bonds between carbon atoms

The bulldog ant has a diploid number of two chromosomes. Therefore, following meiosis, each daughter cell will have a single chromosome. In addition to mutations, how might genetic diversity be generated in this species? A) nothing else B) independent assortment only C) crossing over only D) crossing over and random fertilization

D) crossing over and random fertilization

What two components constitute an active MPF? A) cyclin and tubulin B) ATP synthetase and a protease C) a growth factor and mitotic factor D) cyclin and a cyclin-dependent kinase

D) cyclin and a cyclin-dependent kinase

In a very large population, a quantitative trait has the following distribution pattern. If there is no gene flow, the curve shifts to the left or to the right, and the population size consequently increases over successive generations, which of the following is most likely occurring? A) genetic drift B) disruptive selection C) immigration or emigration D) directional selection

D) directional selection

In those parts of equatorial Africa where the malaria parasite is most common, the sickle-cell allele constitutes 20% of the β hemoglobin alleles in the human gene pool. In the United States, the parasite that causes malaria is not present, but it is present in African-Americans whose ancestors were from equatorial Africa. What should be happening to the sickle-cell allele in the United States, and what should be happening to it in equatorial Africa? A) disruptive selection; stabilizing selection B) directional selection; disruptive selection C) stabilizing selection; disruptive selection D) directional selection; stabilizing selection

D) directional selection; stabilizing selection

Which of the following are maternal effect genes that control the orientation of the egg and thus the Drosophila embryo? A) morphogens B) homeotic genes C) segmentation genes D) egg-polarity genes

D) egg-polarity genes

Hydrangea plants of the same genotype are planted in a large flower garden. Some of the plants produce blue flowers and others pink flowers. This can be best explained by which of the following? A) the alleles are codominant B) the knowledge that multiple alleles are involved C) the allele for blue hydrangea is completely dominant over the allele for pink hydrangea D) environmental factors such as soil pH affect the phenotype

D) environmental factors such as soil pH affect the phenotype

DNA methylation and histone acetylation are examples of which of the following processes? A) genetic mutation B) chromosomal rearrangements C) translocation D) epigenetic phenomena

D) epigenetic phenomena

Gene S controls the sharpness of spines in a type of cactus. Cacti with the dominant allele, S, have sharp spines, whereas homozygous recessive ss cacti have dull spines. At the same time, a second gene, N, determines whether or not cacti have spines. Homozygous recessive nn cacti have no spines at all. The relationship between genes S and N is an example of which of the following inheritance patterns? A) pleiotropy B) incomplete dominance C) codominance D) epistasis

D) epistasis

Choose the pair of terms that correctly completes this sentence: Catabolism is to anabolism as ________ is to ________. A) free energy; entropy B) work; energy C) exergonic; spontaneous D) exergonic; endergonic

D) exergonic; endergonic

Organic molecules with only hydrogens and five carbon atoms cannot ________. A) have a branching carbon skeleton B) have different combinations of double bonds between carbon atoms C) have different positions of double bonds between carbon atoms D) form enantiomers

D) form enantiomers

Mitochondrial DNA is primarily involved in coding for proteins needed for protein complexes of the electron transport chain and ATP synthase. Therefore, mutations in mitochondrial genes would most affect which of the following processes? A) the storage of urine in the urinary bladder B) the movement of oxygen into erythrocytes C) DNA synthesis in cells of the immune system D) generation of ATP in muscle cells

D) generation of ATP in muscle cells

In colorectal cancer, several genes must be mutated for a cell to develop into a cancer cell. Which of the following kinds of genes would you expect to be mutated? A) genes coding for enzymes that act in the colon B) genes of the bacteria, which are abundant in the colon C) genes that are especially susceptible to mutation D) genes involved in control of the cell cycle

D) genes involved in control of the cell cycle

Phospholipids and triglycerides both ________. A) have three fatty acids B) have a phosphate C) contain serine or some other organic compound D) have a glycerol backbone

D) have a glycerol backbone

In the formula for determining a population's genotype frequencies, the "pq" in the term 2pq is necessary because ________. A) the population is doubling in number B) the population is diploid C) heterozygotes can come about in two ways D) heterozygotes have two alleles

D) heterozygotes have two alleles

Humans can digest starch but not cellulose because ________. A) starch monomers are joined by covalent bonds, and cellulose monomers are joined by ionic bonds B) Starch is softer than cellulose C) the monomer of starch is glucose, while the monomer of cellulose is galactose D) humans have enzymes that can hydrolyze the α-glycosidic linkages of starch but not the β-glycosidic linkages of cellulose

D) humans have enzymes that can hydrolyze the α-glycosidic linkages of starch but not the β-glycosidic linkages of cellulose

In which reactions of cellular respiration and fermentation does substrate-level phosphorylation occur? A) only in glycolysis B) only in the citric acid cycle C) only in the electron transport chain D) in both glycolysis and the citric acid cycle

D) in both glycolysis and the citric acid cycle

Carbon dioxide is split to form oxygen gas and carbon compounds in which of the following metabolic pathways? A) during photosynthesis B) during respiration C) during photosynthesis and respiration D) in neither photosynthesis nor respiration

D) in neither photosynthesis nor respiration

In which of the following organisms did the process of photosynthesis most likely originate? A) three separate times during evolution B) in fungi C) in plants D) in prokaryotes

D) in prokaryotes

A controlled experiment ________. A) is repeated many times to ensure that the results are accurate B) includes at least two groups, one differing from the other by two or more variables C) includes one group for which the scientist controls all variables D) includes at least two groups, one of which does not receive the experimental treatment

D) includes at least two groups, one of which does not receive the experimental treatment

A controlled experiment is one that... A) is repeated many times to ensure that the results are accurate B) includes at least two groups, one differing from the other by two or more variables C) includes one group for which the scientist controls all variables D) includes at least two groups, one of which does not receive the experimental treatment

D) includes at least two groups, one of which does not receive the experimental treatment

Radish flowers may be red, purple, or white. A cross between a red-flowered plant and a white-flowered plant yields all-purple offspring. The flower color trait in radishes is an example of which of the following inheritance patterns? A) sex linkage B) a multiple allelic system C) codominance D) incomplete dominance

D) incomplete dominance

As the [H3O+] of the solution decreases, the [OH-] ________. A) decreases and thus the pH decreases B) increases and thus pH decreases C) decreases and thus the pH increases D) increases and thus pH increases

D) increases and thus pH increases

When taken up by a cell, which of the following molecules binds to a repressor so that the repressor no longer binds to the operator? A) repressor B) corepressor C) promoter D) inducer

D) inducer

German scientist Carl Correns found that the inheritance of variegated color on the leaves of certain plants was determined only by the maternal parent. What phenomenon explains this pattern of inheritance? A) genomic imprinting B) ribosome structure C) sex linkage D) inheritance of plastid genes

D) inheritance of plastid genes

Binding of a signaling molecule to which type of receptor leads directly to a change in the distribution of substances on opposite sides of the membrane? A) intracellular receptor B) phosphorylated receptor tyrosine kinase dimer C) G protein-coupled receptor D) ligand-gated ion channel

D) ligand-gated ion channel

One primary advantage of light microscopy over electron microscopy is that ________. A) light microscopy provides higher contrast than electron microscopy B) light microscopy provides for higher magnification than electron microscopy C) light microscopy provides for higher resolution than electron microscopy D) light microscopy allows the visualization of dynamic processes in living cells

D) light microscopy allows the visualization of dynamic processes in living cells

Cells will phagocytize asbestos, but are not able to degrade it. As a result, asbestos fibers accumulate in _____. A) mitochondria B) ribosomes C) peroxisomes D) lysosomes

D) lysosomes

Which of the following molecular structures contain codons? A) tRNA B) rRNA C) a protein D) mRNA

D) mRNA

During which of the following processes do homologous pairs of chromosomes align adjacent to one another at the metaphase plate of a cell? A) metaphase of mitosis B) telophase II of meiosis C) metaphase II of meiosis D) metaphase I of meiosis

D) metaphase I of meiosis

Which of the following is the smallest structure that would most likely be visible with a standard (not super-resolution) research-grade light microscope? A) ribosome B) virus C) microtubule D) mitochondrion

D) mitochondrion

Which organelle is the primary site of ATP synthesis in eukaryotic cells? A) Golgi apparatus B) lysosome C) peroxisome D) mitochondrion

D) mitochondrion

A molecule with the chemical formula C6H12O6 is probably a ________. A) polysaccharide B) fatty acid C) nucleic acid D) monosaccharide

D) monosaccharide

Genetic variation ________. A) is created by the direct action of natural selection B) tends to be reduced when diploid organisms produce gametes C) arises in response to changes in the environment D) must be present in a population before natural selection can act upon the population

D) must be present in a population before natural selection can act upon the population

Which of these provides evidence of the common ancestry of all life? A) structure of the nucleus B) structure of cilia C) structure of chloroplasts D) near universality of the genetic code

D) near universality of the genetic code

Bonds between two atoms that are equally electronegative are ________. A) polar covalent bonds B) hydrogen bonds C) ionic bonds D) nonpolar covalent bonds

D) nonpolar covalent bonds

G1 is associated with which of the following cellular events? A) break down of the nuclear membrane B) the beginning of mitosis C) DNA replication D) normal growth and cell function

D) normal growth and cell function

Sulfur is in the same column of the periodic table as oxygen, but has electronegativity similar to carbon. Compared to water molecules, molecules of H2S will ________. A) have a greater tendency to form hydrogen bonds with each other B) have a higher capacity to absorb heat for the same change in temperature C) have greater cohesion to other molecules of H2S D) not form hydrogen bonds with each other

D) not form hydrogen bonds with each other

The spontaneous loss of amino groups from adenine in DNA results in hypoxanthine, an uncommon base, opposite thymine. What combination of proteins could repair such damage? A) telomerase, helicase, single-strand binding protein B) telomerase, primase, DNA polymerase C) DNA ligase, replication fork proteins, adenylyl cyclase D) nuclease, DNA polymerase, DNA ligase

D) nuclease, DNA polymerase, DNA ligase

The higher the proportion of loci that are "fixed" in a population, the lower are that population's ________. A) chromosome number B) average heterozygosity C) nucleotide variability D) nucleotide variability and average heterozygosity

D) nucleotide variability and average heterozygosity

Mendel continued some of his experiments into the F2 or F3 generation in order to ________. A) observe whether or not the dominant trait would reappear B) obtain a larger number of offspring on which to base statistics C) distinguish which alleles were segregating D) observe whether or not a recessive trait would reappear

D) observe whether or not a recessive trait would reappear

Stanley Miller's 1953 experiments supported the hypothesis that ________. A) the conditions on early Earth were conducive to the origin of life B) life on Earth arose from simple organic molecules, with energy from lightning and volcanoes C) life on Earth arose from simple inorganic molecules D) organic molecules can be synthesized abiotically under conditions that may have existed on early Earth

D) organic molecules can be synthesized abiotically under conditions that may have existed on early Earth

In mechanism, photophosphorylation is most similar to... A. carbon fixation. B. substrate-level phosphorylation in glycolysis. C. the Calvin cycle. D. oxidative phosphorylation in cellular respiration.

D) oxidative phosphorylation in cellular respiration.

The activity of adenylyl cyclase is essentially the opposite of which of the following enzymes? A) phosphorylase B) protein kinase C) protein phosphatase D) phosphodiesterase

D) phosphodiesterase

A heat-killed, phosphorescent (light-emitting) strain of bacteria is mixed with a living, non-phosphorescent strain. Further observations of the mixture show that some of the living cells are now phosphorescent. Which of the following observations would provide the best evidence that the ability to phosphoresce is a heritable trait? A) evidence that DNA was passed from the heat-killed strain to the living strain B) especially bright phosphorescence in the living strain C) evidence that protein passed from the heat-killed strain to the living strain D) phosphorescence in descendants of the living cells

D) phosphorescence in descendants of the living cells

The difference between pinocytosis and receptor-mediated endocytosis is that ________. A) pinocytosis increases the surface area of the plasma membrane, whereas receptor-mediated endocytosis decreases the plasma membrane surface area B) pinocytosis brings only water molecules into the cell, but receptor-mediated endocytosis brings in other molecules as well C) pinocytosis can concentrate substances from the extracellular fluid, but receptor-mediated endocytosis cannot D) pinocytosis is nonselective in the molecules it brings into the cell, whereas receptor-mediated endocytosis offers more selectivity

D) pinocytosis is nonselective in the molecules it brings into the cell, whereas receptor-mediated endocytosis offers more selectivity

Which of the following structures form cytoplasmic channels that connect adjacent plant cells through the cell walls? A) tight junctions B) gap junctions C) desmosomes D) plasmodesmata

D) plasmodesmata

What makes lipids/fats hydrophobic? A) the carboxyl group at one end of the molecule B) the glycerol moiety C) their long carbon skeleton D) presence of relatively nonpolar C—H bonds

D) presence of relatively nonpolar C—H bonds

When chiasmata can first be seen in cells using a microscope, which of the following processes has most likely occurred? A) the separation of homologs B) meiosis II C) anaphase II D) prophase I

D) prophase I

Which of the following processes generates a proton-motive force in mitochondria? A) the flow of protons through ATP synthase down their concentration gradient B) the reduction of NAD+ by the first electron carrier in the electron transport chain C) lowering of pH in the mitochondrial matrix D) pumping of hydrogen ions from the mitochondrial matrix across the inner membrane and into the intermembrane space

D) pumping of hydrogen ions from the mitochondrial matrix across the inner membrane and into the intermembrane space

Which of the following processes is directly associated with photosystem I? A) generation of molecular oxygen B) extraction of hydrogen electrons from the splitting of water C) passing electrons to NADP+ D) receiving electrons from the thylakoid membrane electron transport chain

D) receiving electrons from the thylakoid membrane electron transport chain

Pseudohypertrophic muscular dystrophy is a human disorder that causes gradual deterioration of the muscles. Only boys are affected, and they are always born to phenotypically normal parents. Due to the severity of the disease, the boys die in their teens. Is this disorder likely to be caused by a dominant or recessive allele? Is the inheritance of this trait sex-linked or autosomal? A) recessive, autosomal B) incomplete dominant, sex-linked C) dominant, sex-linked D) recessive, sex-linked

D) recessive, sex-linked

In cattle, roan coat color (mixed red and white hairs) occurs in the heterozygous (CRCW) offspring of red (CRCR) and white (CWCW) homozygotes. Which of the following crosses would produce offspring in the ratio of 1 red:2 roan:1 white? A) red × roan B) red × white C) white × roan D) roan × roan

D) roan × roan

It became apparent to Watson and Crick after completion of their model that the DNA molecule could carry a vast amount of hereditary information. Which of the following characteristics of DNA is responsible for this? A) phosphate-sugar backbones B) side groups of nitrogenous bases C) complementary pairing of bases D) sequence of bases

D) sequence of bases

The reason for differences in the sets of proteins expressed in a nerve and a pancreatic cell of the same individual is that nerve and pancreatic cells contain different ________. A) promoters B) regulatory sequences C) genes D) sets of regulatory proteins

D) sets of regulatory proteins

In eukaryotic cells, transcription cannot begin until A) the DNA introns are removed from the template. B) the two DNA strands have completely separated and exposed the promoter. C) the 5 caps are removed from the mRNA. D) several transcription factors have bound to the promoter.

D) several transcription factors have bound to the promoter.

Adult male humans generally have deeper voices than do adult female humans, which is the direct result of higher levels of testosterone causing growth of the larynx. If the fossil records of apes and humans alike show a trend toward decreasing larynx size in adult females and increasing larynx size in adult males, then ________. A) selection was acting more directly upon genotype than upon phenotype B) intrasexual selection seems to have occurred in both species C) stabilizing selection was occurring in these species concerning larynx size D) sexual dimorphism was evolving over time in these species

D) sexual dimorphism was evolving over time in these species

An atom has four electrons in its valence shell. What types of covalent bonds is it capable of forming? A) double bonds only B) single bonds only C) single and double only D) single, double, or triple

D) single, double, or triple

Which of the following phenotypes is an example of polygenic inheritance? A) pink flowers in snapdragons B) white and purple flower color in peas C) the ABO blood group in humans D) skin pigmentation in humans

D) skin pigmentation in humans

Most Swiss starlings produce four to five eggs in each clutch. Starlings producing fewer or more than this have reduced fitness. Which of the following terms best describes this situation? A) directional selection B) disruptive selection C) sexual selection D) stabilizing selection

D) stabilizing selection

Rank, from low to high, the pH of blood, stomach acid, and urine. A) urine, blood, stomach acid B) blood, urine, and stomach acid C) stomach acid, blood, and urine D) stomach acid, urine, blood

D) stomach acid, urine, blood

Where in a plant cell does the Calvin cycle take place? A) interior of the thylakoid (thylakoid space) B) outer membrane of the chloroplast C) thylakoid membrane D) stroma of the chloroplast

D) stroma of the chloroplast

Which one of these processes describes bottleneck effect? A) alleles transferred to the next generation in portions that differ from previous generation B) transfer of alleles in and out of a population due to movement of fertile individuals C) chance events that change allele frequency D) sudden change in environments that alters gene frequency of a population

D) sudden change in environments that alters gene frequency of a population

Which of the following processes occurs in meiosis but not in mitosis? A) alignment of chromosomes at the equator B) condensation of chromosomes C) chromosome replication D) synapsis of chromosomes

D) synapsis of chromosomes

A cell with a predominance of smooth endoplasmic reticulum is likely specialized to ________. A) store large quantities of water B) actively secrete large quantities of protein C) import and export large quantities of protein D) synthesize large quantities of lipids

D) synthesize large quantities of lipids

Most cells cannot harness heat to perform work because A) heat can never be used to do work. B) cells do not have much thermal energy; they are relatively cool. C) heat does not involve a transfer of energy. D) temperature is usually uniform throughout a cell.

D) temperature is usually uniform throughout a cell.

During which of the following metabolic processes is most of the CO2 from the catabolism of glucose is released? A) electron transport B) glycolysis C) oxidation of pyruvate to acetyl-CoA D) the citric acid cycle

D) the citric acid cycle

What component of amino acid structure varies among different amino acids? A) the presence of a central C atom B) the glycerol molecule that forms the backbone of the amino acid C) the long carbon-hydrogen tails of the molecule D) the components of the R group

D) the components of the R group

Consider two solutions: solution X has a pH of 4; solution Y has a pH of 7. From this information, we can reasonably conclude that ________. A) solution Y has no free hydrogen ions (H+) B) the concentration of hydrogen ions in solution Y is 1000 times as great as the concentration of hydrogen ions in solution X C) the concentration of hydrogen ions in solution X is 3 times as great as the concentration of hydrogen ions in solution Y D) the concentration of hydrogen ions in solution X is 1000 times as great as the concentration of hydrogen ions in solution Y

D) the concentration of hydrogen ions in solution X is 1000 times as great as the concentration of hydrogen ions in solution Y

Soon after the island of Hawaii rose above the sea surface (somewhat less than one million years ago), the evolution of life on this new island should have been most strongly influenced by ________. A) sexual selection B) habitat differentiation C) a genetic bottleneck D) the founder effect

D) the founder effect

The inability of organisms to evolve anything that could be an advantage reflects ________. A) the consequences of inbreeding B) the consequences of random mutations C) the inability to compromise D) the limits of choices of genes within a species

D) the limits of choices of genes within a species

Tay-Sachs disease is a human genetic abnormality that results in cells accumulating and becoming clogged with very large, complex, undigested lipids. Which cellular organelle is most likely defective in this condition? A) the Golgi apparatus B) the rough endoplasmic reticulum C) the smooth endoplasmic reticulum D) the lysosome

D) the lysosome

Which of the following processes is the first event to take place in translation in eukaryotes? A) binding of the larger ribosomal subunit to smaller ribosomal subunits B) base pairing of activated methionine-tRNA to AUG of the messenger RNA C) the ribosome reaches a stop codon D) the small subunit of the ribosome recognizes and attaches to the 5′ cap of mRNA

D) the small subunit of the ribosome recognizes and attaches to the 5′ cap of mRNA

Organic chemistry is currently defined as A) the study of hydrocarbons. B) the study of natural (as opposed to synthetic) compounds. C) the study of compounds made only by living cells. D) the study of carbon compounds.

D) the study of carbon compounds.

Which of the following characteristics is directly related to the coding of a single amino acid during the process of translation? A) the amino acetyl tRNA synthase B) the complementarity of DNA and RNA C) the base sequence of the tRNA D) the three-base sequence of mRNA

D) the three-base sequence of mRNA

Where are ATP synthase complexes located in plant cells? A) thylakoid membrane only B) inner mitochondrial membrane only C) thylakoid membrane and plasma membrane D) thylakoid membrane and inner mitochondrial membrane

D) thylakoid membrane and inner mitochondrial membrane

Gene expression is often assayed by measuring the level of mRNA produced from a gene. What level of the control of gene expression can by analyzed by this type of assay? A) translational control B) alternative splicing C) replication control D) transcriptional control

D) transcriptional control

Which of the following would be most appropriate method to observe and measure the size of ribosomes in a eukaryotic cell? A) standard light microscopy B) scanning electron microscopy C) a hand lens (magnifying glass) D) transmission electron microscopy

D) transmission electron microscopy

How many electron pairs are shared between carbon atoms in a molecule that has the formula C2H4? A) four B) three C) one D) two

D) two

Systems biology is mainly an attempt to.... A) build high-throughput machines for the rapid acquisition of biological data. B) simplify complex problems by reducing the system into smaller, less complex units. C) analyze genomes from different species. D) understand the behavior of entire biological systems by studying interactions among its component parts.

D) understand the behavior of entire biological systems by studying interactions among its component parts.

Which of the following is a hydrophobic material? A) table salt B) paper C) sugar D) wax

D) wax

Under what conditions does the trp repressor block transcription of the trp operon? A) when the repressor binds to the inducer B) when the repressor is not bound to tryptophan C) when the repressor is not bound to the operator D) when the repressor binds to tryptophan

D) when the repressor binds to tryptophan

The mathematical expression for the change in free energy of a system is ΔG = ΔH - TΔS. Which of the following statements is correct? A) ΔS is the change in enthalpy, a measure of randomness. B) ΔH is the change in entropy, the energy available to do work. C) T is the temperature in degrees Celsius. D) ΔG is the change in free energy.

D) ΔG is the change in free energy.

A young dog has never had much energy. He is brought to a veterinarian for help, and she decides to conduct several diagnostic tests. She discovers that the dog's mitochondria can use only fatty acids and amino acids for respiration, and his cells produce more lactate than normal. Of the following, which is the best explanation of the dog's condition? A. His cells contain something that inhibits oxygen use in his mitochondria. B. His cells have a defective electron transport chain, so glucose goes to lactate instead of to acetyl CoA. C. His cells cannot move NADH from glycolysis into the mitochondria. D. His mitochondria lack the transport protein that moves pyruvate across the outer mitochondrial membrane.

D. His mitochondria lack the transport protein that moves pyruvate across the outer mitochondrial membrane.

The reducing power for Calvin cycle reactions is provided by which of the following molecules? A. NADP+ B. NADH C. ATP D. NADPH

D. NADPH

A person on a strict diet and exercise regimen lost 7 kg (about 15 pounds) of body fat in just two weeks. What is the most likely way that the lost fat left the body?

It was released as CO2 and H2O.

What functional group can pick up protons and raise pH of surrounding solution?

NH2 - amino group

Which molecule contains a functional group known as the "energy currency of living organisms"?

Phosphate

In addition to activating or inhibiting enzymes through allosteric regulation, what other means does a cell use to control enzymatic activity?

localization of enzymes into specific organelles or membranes

Water is one of the end products of aerobic respiration. What is the source of the oxygen atom used in formation of the water?

molecular oxygen (O2)


Conjuntos de estudio relacionados

OSHA: Extension Cords and Power Tools

View Set

Parliamentary and Presidential Democracies

View Set

PRACTICE EXAM OF TEXAS LIFE & HEALTH INSURANCE. "SECTION 2- TEXAS LIFE LAW"

View Set

Authoritarian states: Mao's rise to power 1949

View Set

Complementary and Integrative Health

View Set